%% ******************************************************************************** %% AUTHOR: Raj Dahya %% CREATED: November 2020 %% EDITED: — %% TYPE: Notizen %% TITLE: Lösungen zu diversen Aufgaben im Kurs %% DOI: — %% DEPARTMENT: Fakultät for Mathematik und Informatik %% INSTITUTE: Universität Leipzig %% ******************************************************************************** %% ******************************************************************************** %% DOCUMENT STRUCTURE: %% ~~~~~~~~~~~~~~~~~~~ %% %% - root.tex; %% | %% ---- parameters.tex; %% | %% ---- srclocal/index.tex; %% | %% ---- src/setup-type.tex; %% | %% ---- src/setup-packages.tex; %% | %% ---- src/setup-parameters.tex; %% | %% ---- src/setup-macros.tex; %% | %% ---- src/setup-environments.tex; %% | %% ---- src/setup-layout.tex; %% | %% ---- srclocal/setup-localmacros.tex; %% | %% ---- front/index.tex; %% | %% ---- front/title.tex; %% | %% ---- front/foreword.tex; %% | %% ---- front/contents.tex; %% | %% ---- body/index.tex; %% | %% ---- body/uebung/ueb1.tex; %% | %% ---- body/uebung/ueb2.tex; %% | %% ---- body/uebung/ueb3.tex; %% | %% ---- body/uebung/ueb4.tex; %% | %% ---- body/uebung/ueb5.tex; %% | %% ---- body/uebung/ueb6.tex; %% | %% ---- body/uebung/ueb7.tex; %% | %% ---- body/uebung/ueb8.tex; %% | %% ---- body/uebung/ueb9.tex; %% | %% ---- body/uebung/ueb10.tex; %% | %% ---- body/uebung/ueb11.tex; %% | %% ---- body/ska/ska4.tex; %% | %% ---- body/ska/ska5.tex; %% | %% ---- body/ska/ska6.tex; %% | %% ---- body/quizzes/quiz1.tex; %% | %% ---- body/quizzes/quiz2.tex; %% | %% ---- body/quizzes/quiz3.tex; %% | %% ---- body/quizzes/quiz4.tex; %% | %% ---- body/quizzes/quiz5.tex; %% | %% ---- body/quizzes/quiz6.tex; %% | %% ---- body/quizzes/quiz7.tex; %% | %% ---- body/quizzes/quiz8.tex; %% | %% ---- body/quizzes/quiz9.tex; %% | %% ---- body/quizzes/quiz10.tex; %% | %% ---- body/quizzes/quiz11.tex; %% | %% ---- back/index.tex; %% | %% ---- ./back/quelle.bib; %% %% DOCUMENT-RANDOM-SEED: 5637845 %% ******************************************************************************** %% ******************************************************************************** %% FILE: root.tex %% ******************************************************************************** %% ******************************************************************************** %% FILE: parameters.tex %% ******************************************************************************** %% ******************************************************************************** %% FILE: srclocal/index.tex %% ******************************************************************************** \makeatletter %% ******************************************************************************** %% FILE: src/setup-type.tex %% ******************************************************************************** \documentclass[ 12pt, a4paper, oneside, openright, center, chapterbib, crosshair, fleqn, headcount, headline, indent, indentfirst=false, portrait, phonetic, oldernstyle, onecolumn, sfbold, upper, ]{scrbook} %% ******************************************************************************** %% FILE: src/setup-packages.tex %% ******************************************************************************** \PassOptionsToPackage{T2A,OT1}{fontenc} % T1,OT1,T2A,OT2 \PassOptionsToPackage{utf8}{inputenc} % utf8 \PassOptionsToPackage{british,english,ngerman,russian}{babel} \PassOptionsToPackage{ english, ngerman, russian, capitalise, }{cleveref} \PassOptionsToPackage{ bookmarks=true, bookmarksopen=false, bookmarksopenlevel=0, bookmarkstype=toc, colorlinks=false, raiselinks=true, hyperfigures=true, }{hyperref} \PassOptionsToPackage{ reset, left=1in, right=1in, top=20mm, bottom=20mm, heightrounded, }{geometry} \PassOptionsToPackage{ framemethod=TikZ, }{mdframed} \PassOptionsToPackage{normalem}{ulem} \PassOptionsToPackage{ amsmath, thmmarks, }{ntheorem} \PassOptionsToPackage{table}{xcolor} \PassOptionsToPackage{ all, color, curve, frame, import, knot, line, movie, rotate, textures, tile, tips, web, xdvi, }{xy} \usepackage{amsfonts} \usepackage{amsmath} \usepackage{amssymb} \usepackage{ntheorem} % <— muss nach den ams* Packages vorkommen!! \usepackage{array} \usepackage{babel} \usepackage{bbding} \usepackage{bbm} \usepackage{calc} \usepackage{sectsty} \usepackage{titlesec} \usepackage{fancyhdr} \usepackage{footmisc} \usepackage{geometry} \usepackage{graphicx} \usepackage{ifpdf} \usepackage{ifthen} \usepackage{ifnextok} \usepackage{longtable} \usepackage{multicol} \usepackage{multirow} \usepackage{nameref} \usepackage{nowtoaux} \usepackage{paralist} \usepackage{enumerate} %% nach [paralist] \usepackage{pgf} \usepackage{pgfplots} \usepackage{proof} \usepackage{refcount} \usepackage{relsize} \usepackage{savesym} \usepackage{stmaryrd} \usepackage{subfigure} \usepackage{yfonts} %% <— Altgotische Fonts \usepackage{tikz} \usepackage{xy} \usepackage{undertilde} \usepackage{ulem} %% <– f\"ur besseren \underline-Befehl (\ul) \usepackage{xcolor} \usepackage{xspace} \usepackage{xstring} \usepackage{hyperref} \usepackage{cleveref} % must vor hyperref geladen werden. \pgfplotsset{compat=newest} \usetikzlibrary{ angles, arrows, automata, calc, decorations, decorations.pathmorphing, decorations.pathreplacing, math, positioning, patterns, quotes, snakes, } %% \var ≈ alter Befehl %% \xvar ≈ wie das neue Package \var interpretieren soll. \savesymbol{Diamond} \savesymbol{emptyset} \savesymbol{ggg} \savesymbol{int} \savesymbol{lll} \savesymbol{RectangleBold} \savesymbol{langle} \savesymbol{rangle} \savesymbol{hookrightarrow} \savesymbol{hookleftarrow} \savesymbol{Asterisk} \usepackage{mathabx} \usepackage{wasysym} \let\varemptyset=\emptyset \restoresymbol{x}{Diamond} \restoresymbol{x}{emptyset} \restoresymbol{x}{ggg} \restoresymbol{x}{int} \restoresymbol{x}{lll} \restoresymbol{x}{RectangleBold} \restoresymbol{x}{langle} \restoresymbol{x}{rangle} \restoresymbol{x}{hookrightarrow} \restoresymbol{x}{hookleftarrow} \restoresymbol{x}{Asterisk} \ifpdf \usepackage{pdfcolmk} \fi \usepackage{mdframed} %% Force-Import aus MnSymbol \DeclareFontFamily{U}{MnSymbolA}{} \DeclareFontShape{U}{MnSymbolA}{m}{n}{ <-6> MnSymbolA5 <6-7> MnSymbolA6 <7-8> MnSymbolA7 <8-9> MnSymbolA8 <9-10> MnSymbolA9 <10-12> MnSymbolA10 <12-> MnSymbolA12 }{} \DeclareFontShape{U}{MnSymbolA}{b}{n}{ <-6> MnSymbolA-Bold5 <6-7> MnSymbolA-Bold6 <7-8> MnSymbolA-Bold7 <8-9> MnSymbolA-Bold8 <9-10> MnSymbolA-Bold9 <10-12> MnSymbolA-Bold10 <12-> MnSymbolA-Bold12 }{} \DeclareSymbolFont{MnSyA}{U}{MnSymbolA}{m}{n} \DeclareMathSymbol{\lcirclearrowright}{\mathrel}{MnSyA}{252} \DeclareMathSymbol{\lcirclearrowdown}{\mathrel}{MnSyA}{255} \DeclareMathSymbol{\rcirclearrowleft}{\mathrel}{MnSyA}{250} \DeclareMathSymbol{\rcirclearrowdown}{\mathrel}{MnSyA}{251} \DeclareFontFamily{U}{MnSymbolC}{} \DeclareSymbolFont{MnSyC}{U}{MnSymbolC}{m}{n} \DeclareFontShape{U}{MnSymbolC}{m}{n}{ <-6> MnSymbolC5 <6-7> MnSymbolC6 <7-8> MnSymbolC7 <8-9> MnSymbolC8 <9-10> MnSymbolC9 <10-12> MnSymbolC10 <12-> MnSymbolC12% }{} \DeclareMathSymbol{\powerset}{\mathord}{MnSyC}{180} %% ******************************************************************************** %% FILE: src/setup-parameters.tex %% ******************************************************************************** \def\boolwahr{true} \def\boolfalsch{false} \def\boolleer{} \let\documenttwosided\boolfalsch \let\boolinappendix\boolfalsch \let\boolinmdframed\boolfalsch \let\eqtagset\boolfalsch \let\eqtaglabel\boolleer \let\eqtagsymb\boolleer \newcount\bufferctr \newcount\bufferreplace \newcounter{columnanzahl} \newlength\rtab \newlength\gesamtlinkerRand \newlength\gesamtrechterRand \newlength\ownspaceabovethm \newlength\ownspacebelowthm \setlength{\rtab}{0.025\textwidth} \setlength{\ownspaceabovethm}{0.5\baselineskip} \setlength{\ownspacebelowthm}{0.5\baselineskip} \setlength{\gesamtlinkerRand}{0pt} \setlength{\gesamtrechterRand}{0pt} \def\secnumberingpt{$\cdot$} \def\secnumberingseppt{.} \def\subsecnumberingseppt{} \def\thmnumberingpt{$\cdot$} \def\thmnumberingseppt{} \def\thmForceSepPt{.} \definecolor{leer}{gray}{1} \definecolor{hellgrau}{gray}{0.85} \definecolor{dunkelgrau}{gray}{0.5} \definecolor{maroon}{rgb}{0.6901961,0.1882353,0.3764706} \definecolor{dunkelgruen}{rgb}{0.015625,0.363281,0.109375} \definecolor{dunkelrot}{rgb}{0.5450980392,0,0} \definecolor{dunkelblau}{rgb}{0,0,0.5450980392} \definecolor{blau}{rgb}{0,0,1} \definecolor{newresult}{rgb}{0.6,0.6,0.6} \definecolor{improvedresult}{rgb}{0.9,0.9,0.9} \definecolor{hervorheben}{rgb}{0,0.9,0.7} \definecolor{starkesblau}{rgb}{0.1019607843,0.3176470588,0.8156862745} \definecolor{achtung}{rgb}{1,0.5,0.5} \definecolor{frage}{rgb}{0.5,1,0.5} \definecolor{schreibweise}{rgb}{0,0.7,0.9} \definecolor{axiom}{rgb}{0,0.3,0.3} %% ******************************************************************************** %% FILE: src/setup-macros.tex %% ******************************************************************************** %% **************************************************************** %% TEX: %% **************************************************************** \def\let@name#1#2{\expandafter\let\csname #1\expandafter\endcsname\csname #2\endcsname\relax} \DeclareRobustCommand\crfamily{\fontfamily{ccr}\selectfont} \DeclareTextFontCommand{\textcr}{\crfamily} \def\nichtzeigen#1{\phantom{#1}} %% **************************************************************** %% SPACING: %% **************************************************************** \def\ifthenelseleer#1#2#3{\ifthenelse{\equal{#1}{}}{#2}{#1#3}} \def\bedingtesspaceexpand#1#2#3{\ifthenelseleer{\csname #1\endcsname}{#3}{#2#3}} \def\voritemise{\leavevmode\nvraum{1}} \def\hraum{\null\hfill\null} \def\vraum{\null\vfill\null} \def\nvraum{\@ifnextchar\bgroup{\nvraum@c}{\nvraum@bes}} \def\nvraum@c#1{\vspace*{-#1\baselineskip}} \def\nvraum@bes{\vspace*{-\baselineskip}} \def\erlaubeplatz{\relax\ifmmode\else\@\xspace\fi} \def\entferneplatz{\relax\ifmmode\else\expandafter\@gobble\fi} %% **************************************************************** %% TAGS / BEZEICHNUNGEN / LABELLING: %% **************************************************************** \def\send@toaux#1{\@bsphack\protected@write\@auxout{}{\string#1}\@esphack} %% \rlabel{LABEL}[CTR]{CREF-SHORT}{CREF-LONG}{DISPLAYTEXT} \def\rlabel#1[#2]#3#4#5{#5\rlabel@aux{#1}[#2]{#3}{#4}{#5}} \def\rlabel@aux#1[#2]#3#4#5{% \send@toaux{\newlabel{#1}{{\@currentlabel}{\thepage}{{\unexpanded{#5}}}{#2.\csname the#2\endcsname}{}}}\relax% } %% \tag@rawscheme{CREF-SHORT}{CREF-LONG}[CTR]{LEFT-BRKT}{RIGHT-BRKT} [LABEL]{DISPLAYTEXT} \def\tag@rawscheme#1#2[#3]#4#5{\@ifnextchar[{\tag@rawscheme@{#1}{#2}[#3]{#4}{#5}}{\tag@rawscheme@{#1}{#2}[#3]{#4}{#5}[*]}} \def\tag@rawscheme@#1#2[#3]#4#5[#6]{\@ifnextchar\bgroup{\tag@rawscheme@@{#1}{#2}[#3]{#4}{#5}[#6]}{\tag@rawscheme@@{#1}{#2}[#3]{#4}{#5}[#6]{}}} \def\tag@rawscheme@@#1#2[#3]#4#5[#6]#7{% \ifthenelse{\equal{#6}{*}}{% \ifthenelse{\equal{#7}{\boolleer}}{\refstepcounter{#3}#4\csname the#3\endcsname#5}{#4#7#5}% }{% \refstepcounter{#3}#4% \ifthenelse{\equal{#7}{\boolleer}}{\rlabel{#6}[#3]{#1}{#2}{\csname the#3\endcsname}}{\rlabel{#6}[#3]{#1}{#2}{#7}}% #5% }% } %% \tag@scheme{CREF-SHORT}{CREF-LONG}[CTR] [LABEL]{DISPLAYTEXT} \def\tag@scheme#1#2[#3]{\tag@rawscheme{#1}{#2}[#3]{\upshape(}{\upshape)}} %% \eqtag[LABEL]{DISPLAYTEXT} \def\eqtag@post#1{\makebox[0pt][r]{#1}} \def\eqtag@pre{\tag@scheme{Eq}{Equation}[Xe]} \def\eqtag{\@ifnextchar[{\eqtag@}{\eqtag@[*]}} \def\eqtag@[#1]{\@ifnextchar\bgroup{\eqtag@@[#1]}{\eqtag@@[#1]{}}} \def\eqtag@@[#1]#2{\eqtag@post{\eqtag@pre[#1]{#2}}} \def\eqcref#1{\text{(\ref{#1})}} \def\ptcref#1{\ref{#1}} \def\punktlabel#1{\label{it:#1:\beweislabel}} \def\punktcref#1{\eqcref{it:#1:\beweislabel}} \def\crefit#1#2{\cref{#1}~\eqcref{it:#2:#1}} \def\Crefit#1#2{\Cref{#1}~\eqcref{it:#2:#1}} %% UNDER/OVERSET BEFEHLE \def\opfromto[#1]_#2^#3{\underset{#2}{\overset{#3}{#1}}} \def\textoverset#1#2{\overset{\text{#1}}{#2}} \def\textunderset#1#2{\underset{#2}{\text{#1}}} \def\crefoverset#1#2{\textoverset{\cref{#1}}{#2}} \def\Crefoverset#1#2{\textoverset{\Cref{#1}}{#2}} \def\crefunderset#1#2{\textunderset{#2}{\cref{#1}}} \def\Crefunderset#1#2{\textunderset{#2}{\Cref{#1}}} \def\eqcrefoverset#1#2{\textoverset{\eqcref{#1}}{#2}} \def\eqcrefunderset#1#2{\textunderset{#2}{\eqcref{#1}}} \def\mathclap#1{#1} \def\oberunterset#1{\@ifnextchar^{\oberunterset@oben{#1}}{\oberunterset@unten{#1}}} \def\oberunterset@oben#1^#2_#3{\underset{\mathclap{#3}}{\overset{\mathclap{#2}}{#1}}} \def\oberunterset@unten#1_#2^#3{\underset{\mathclap{#2}}{\overset{\mathclap{#3}}{#1}}} \def\breitunderbrace#1_#2{\underbrace{#1}_{\mathclap{#2}}} \def\breitoverbrace#1^#2{\overbrace{#1}^{\mathclap{#2}}} \def\breitunderbracket#1_#2{\underbracket{#1}_{\mathclap{#2}}} \def\breitoverbracket#1^#2{\overbracket{#1}^{\mathclap{#2}}} \def\generatenestedsecnumbering#1#2#3{% \expandafter\gdef\csname thelong#3\endcsname{% \expandafter\csname the#2\endcsname% \secnumberingpt% \expandafter\csname #1\endcsname{#3}% }% \expandafter\gdef\csname theshort#3\endcsname{% \expandafter\csname #1\endcsname{#3}% }% } \def\generatenestedthmnumbering#1#2#3{% \expandafter\gdef\csname the#3\endcsname{% \expandafter\csname the#2\endcsname% \thmnumberingpt% \expandafter\csname #1\endcsname{#3}% }% \expandafter\gdef\csname theshort#3\endcsname{% \expandafter\csname #1\endcsname{#3}% }% } %% **************************************************************** %% ALLG. MACROS: %% **************************************************************** \def\+#1{\addtocounter{#1}{1}} \def\setcounternach#1#2{\setcounter{#1}{#2}\addtocounter{#1}{-1}} \def\textsubscript#1{${}_{\textup{#1}}$} \def\rome#1{\overline{\underline{#1}}} \def\textTODO{\text{[{\large\textcolor{red}{More work needed!}}]}} \def\hlineEIGENpt{\hdashline[0.5pt/5pt]} \def\clineEIGENpt#1{\cdashline{#1}[0.5pt/5pt]} \def\forcepunkt#1{#1\IfEndWith{#1}{.}{}{.}} \def\lateinabkuerzung#1#2{% \expandafter\gdef\csname #1\endcsname{\emph{#2}\@ifnextchar.{\entferneplatz}{\erlaubeplatz}} } \def\deutscheabkuerzung#1#2{% \expandafter\gdef\csname #1\endcsname{{#2}\@ifnextchar.{\entferneplatz}{\erlaubeplatz}} } %% **************************************************************** %% MATHE %% **************************************************************** \def\matrix#1{\left(\begin{array}{#1}} \def\endmatrix{\end{array}\right)} \def\smatrix{\left(\begin{smallmatrix}} \def\endsmatrix{\end{smallmatrix}\right)} \def\multiargrekursiverbefehl#1#2#3#4#5#6#7#8{% \expandafter\gdef\csname#1\endcsname #2##1#4{\csname #1@anfang\endcsname##1#3\egroup} \expandafter\def\csname #1@anfang\endcsname##1#3{#5##1\@ifnextchar\egroup{\csname #1@ende\endcsname}{#7\csname #1@mitte\endcsname}} \expandafter\def\csname #1@mitte\endcsname##1#3{#6##1\@ifnextchar\egroup{\csname #1@ende\endcsname}{#7\csname #1@mitte\endcsname}} \expandafter\def\csname #1@ende\endcsname##1{#8} } \multiargrekursiverbefehl{svektor}{[}{;}{]}{\begin{smatrix}}{}{\\}{\\\end{smatrix}} \multiargrekursiverbefehl{vektor}{[}{;}{]}{\begin{matrix}{c}}{}{\\}{\\\end{matrix}} \multiargrekursiverbefehl{vektorzeile}{}{,}{;}{}{&}{}{} \multiargrekursiverbefehl{matlabmatrix}{[}{;}{]}{\begin{smatrix}\vektorzeile}{\vektorzeile}{;\\}{;\end{smatrix}} \def\cases[#1]#2{\left\{\begin{array}[#1]{#2}} \def\endcases{\end{array}\right.} \def\BeweisRichtung[#1]{\@ifnextchar\bgroup{\@BeweisRichtung@c[#1]}{\@BeweisRichtung@bes[#1]}} \def\@BeweisRichtung@bes[#1]{{\bfseries(#1).~}} \def\@BeweisRichtung@c[#1]#2#3{{\bfseries(#2#1#3).~}} \def\erzeugeBeweisRichtungBefehle#1#2{ \expandafter\gdef\csname #1text\endcsname##1##2{\BeweisRichtung[#2]{##1}{##2}} \expandafter\gdef\csname #1\endcsname{% \@ifnextchar\bgroup{\csname #1@\endcsname}{\csname #1text\endcsname{}{}}% } \expandafter\gdef\csname #1@\endcsname##1##2{% \csname #1text\endcsname{\punktcref{##1}}{\punktcref{##2}}% } } \erzeugeBeweisRichtungBefehle{hinRichtung}{$\Longrightarrow$} \erzeugeBeweisRichtungBefehle{herRichtung}{$\Longleftarrow$} \erzeugeBeweisRichtungBefehle{hinherRichtung}{$\Longleftrightarrow$} \def\cal#1{\mathcal{#1}} \def\brkt#1{\langle{}#1{}\rangle} \def\mathfrak#1{\mbox{\usefont{U}{euf}{m}{n}#1}} \def\kurs#1{\textit{#1}} \def\rectangleblack{\text{\RectangleBold}} \def\rectanglewhite{\text{\Rectangle}} \def\squareblack{\blacksquare} \def\squarewhite{\Box} %% ******************************************************************************** %% FILE: src/setup-environments.tex %% ******************************************************************************** %% ********************************************************************** %% CLEVEREF: ************************************************************ \def\crefname@full#1#2#3{\crefname{#1}{#2}{#3}\Crefname{#1}{#2}{#3}} \crefname@full{chapter}{Kapitel}{Kapitel} \crefname@full{section}{Abschnitt}{Abschnitte} \crefname@full{figure}{Fig.}{Fig.} \crefname@full{subfigure}{Fig.}{Fig.} \crefname@full{proof}{Beweis}{Beweise} \crefname@full{thm}{Theorem}{Theoreme} \crefname@full{satz}{Satz}{Sätze} \crefname@full{claim}{Behauptung}{Behauptungen} \crefname@full{lemm}{Lemma}{Lemmata} \crefname@full{cor}{Korollar}{Korollarien} \crefname@full{folg}{Folgerung}{Folgerungen} \crefname@full{prop}{Proposition}{Propositionen} \crefname@full{defn}{Definition}{Definitionen} \crefname@full{conv}{Konvention}{Konventionen} \crefname@full{fact}{Fakt}{Fakten} \crefname@full{rem}{Bemerkung}{Bemerkungen} \crefname@full{qstn}{Frage}{Fragen} \crefname@full{e.g.}{Beipsiel}{Beipsiele} %% **************************************************************** %% THEOREME: %% **************************************************************** \def\qedEIGEN#1{\@ifnextchar[{\qedEIGEN@c{#1}}{\qedEIGEN@bes{#1}}}%] \def\qedEIGEN@bes#1{% \parfillskip=0pt% % so \par doesnt push \square to left \widowpenalty=10000% % so we dont break the page before \square \displaywidowpenalty=10000% % ditto \finalhyphendemerits=0% % TeXbook exercise 14.32 \leavevmode% % \nobreak means lines not pages \unskip% % remove previous space or glue \nobreak% % don’t break lines \hfil% % ragged right if we spill over \penalty50% % discouragement to do so \hskip.2em% % ensure some space \null% % anchor following \hfill \hfill% % push \square to right #1% % the end-of-proof mark \par% } \def\qedEIGEN@c#1[#2]{% \parfillskip=0pt% % so \par doesnt push \square to left \widowpenalty=10000% % so we dont break the page before \square \displaywidowpenalty=10000% % ditto \finalhyphendemerits=0% % TeXbook exercise 14.32 \leavevmode% % \nobreak means lines not pages \unskip% % remove previous space or glue \nobreak% % don’t break lines \hfil% % ragged right if we spill over \penalty50% % discouragement to do so \hskip.2em% % ensure some space \null% % anchor following \hfill \hfill% % push \square to right {#1~{\smaller\bfseries\upshape (#2)}}% \par% } \def\qedVARIANT#1#2{ \expandafter\def\csname ennde#1Sign\endcsname{#2} \expandafter\def\csname ennde#1\endcsname{\@ifnextchar[{\qedEIGEN@c{#2}}{\qedEIGEN@bes{#2}}} %] } \qedVARIANT{OfProof}{$\squareblack$} \qedVARIANT{OfWork}{\rectangleblack} \qedVARIANT{OfSomething}{$\dashv$} \qedVARIANT{OnNeutral}{$\lozenge$} % \lozenge \bigcirc \blacklozenge \def\qedsymbol{\enndeOfProofSign} \def\proofSymbol{\enndeOfProofSign} \def\ra@pretheoremwork{ \setlength{\theorempreskipamount}{\ownspaceabovethm} } \def\rathmtransfer#1#2{ \expandafter\def\csname #2\endcsname{\csname #1\endcsname} \expandafter\def\csname end#2\endcsname{\csname end#1\endcsname} } \def\ranewthm#1#2#3[#4]{ %% FOR \BEGIN{THM} \theoremstyle{\current@theoremstyle} \theoremseparator{\current@theoremseparator} \theoremprework{\ra@pretheoremwork} \@ifundefined{#1@basic}{\newtheorem{#1@basic}[#4]{#2}}{\renewtheorem{#1@basic}[#4]{#2}} %% FOR \BEGIN{THM}[...] \theoremstyle{\current@theoremstyle} \theoremseparator{\thmForceSepPt} \theoremprework{\ra@pretheoremwork} \@ifundefined{#1@withName}{\newtheorem{#1@withName}[#4]{#2}}{\renewtheorem{#1@withName}[#4]{#2}} %% FOR \BEGIN{THM*} \theoremstyle{nonumberplain} \theoremseparator{\thmForceSepPt} \theoremprework{\ra@pretheoremwork} \@ifundefined{#1@star@basic}{\newtheorem{#1@star@basic}[Xdisplaynone]{#2}}{\renewtheorem{#1@star@basic}[Xdisplaynone]{#2}} %% FOR \BEGIN{THM*}[...] \theoremstyle{nonumberplain} \theoremseparator{\thmForceSepPt} \theoremprework{\ra@pretheoremwork} \@ifundefined{#1@star@withName}{\newtheorem{#1@star@withName}[Xdisplaynone]{#2}}{\renewtheorem{#1@star@withName}[Xdisplaynone]{#2}} %% GENERATE ENVIRONMENTS: \umbauenenv{#1}{#3}[#4] \umbauenenv{#1@star}{#3}[Xdisplaynone] %% TRANSFER *-DEFINITION \rathmtransfer{#1@star}{#1*} } \def\umbauenenv#1#2[#3]{% %% \BEGIN{THM}... \expandafter\def\csname #1\endcsname{\relax% \@ifnextchar[{\csname #1@\endcsname}{\csname #1@\endcsname[*]}% } %% \BEGIN{THM}[ANFANG]... \expandafter\def\csname #1@\endcsname[##1]{\relax% \@ifnextchar[{\csname #1@@\endcsname[##1]}{\csname #1@@\endcsname[##1][*]}% } %% \BEGIN{THM}[ANFANG][SCHLUSS] \expandafter\def\csname #1@@\endcsname[##1][##2]{% \ifx*##1% \def\enndeOfBlock{\csname end#1@basic\endcsname} \csname #1@basic\endcsname% \else% \def\enndeOfBlock{\csname end#1@withName\endcsname} \csname #1@withName\endcsname[##1]% \fi% \def\makelabel####1{% \gdef\beweislabel{####1}% \label{\beweislabel}% }% \ifx*##2% \def\enndeSymbol{\qedEIGEN{#2}} \else% \def\enndeSymbol{\qedEIGEN{#2}[##2]} \fi } %% \END{THM} \expandafter\gdef\csname end#1\endcsname{\enndeSymbol\enndeOfBlock} } %% NEWTHEOREM EINSTELLUNGSOPTIONEN: %% F\"UR \theoremstyle %% plain Emulates original LATEX defin, except uses param \theorem...skipamount. %% break Header followed by line break. %% change Header, Number and Text are interchanged, without a line break. %% changebreak =change, but with a line break after Header. %% margin Number in left margin, without a line break. %% marginbreak =margin, but with a line break after the header. %% nonumberplain =plain, without number. %% nonumberbreak =break, without number. %% empty No number, no name. Only the optional argument is typeset. %% \theoremclass \theoremnumbering %% \theorempreskip \theorempostkip \theoremindent %% \theoremprework \theorempostwork \def\current@theoremstyle{plain} \def\current@theoremseparator{\thmnumberingseppt} \theoremstyle{\current@theoremstyle} \theoremseparator{\current@theoremseparator} \theoremsymbol{} \newtheorem{X}{X}[chapter] % for most theorems \newtheorem{Xe}{Xe}[chapter] % for equations \newtheorem*{Xdisplaynone}{Xdisplaynone}[chapter] % a dummy counter, that will never be displayed. \newtheorem{Xsp}{Xsp}[chapter] % for special theorems \generatenestedthmnumbering{arabic}{chapter}{X} \generatenestedthmnumbering{arabic}{chapter}{Xe} \generatenestedthmnumbering{Roman}{chapter}{Xsp} \let\theXsp\theshortXsp \theoremheaderfont{\upshape\bfseries} \theorembodyfont{\slshape} \ranewthm{thm}{Theorem}{\enndeOnNeutralSign}[X] \ranewthm{satz}{Satz}{\enndeOnNeutralSign}[X] \ranewthm{claim}{Behauptung}{\enndeOnNeutralSign}[X] \ranewthm{lemm}{Lemma}{\enndeOnNeutralSign}[X] \ranewthm{cor}{Korollar}{\enndeOnNeutralSign}[X] \ranewthm{folg}{Folgerung}{\enndeOnNeutralSign}[X] \ranewthm{prop}{Proposition}{\enndeOnNeutralSign}[X] \theorembodyfont{\upshape} \ranewthm{defn}{Definition}{\enndeOnNeutralSign}[X] \ranewthm{conv}{Konvention}{\enndeOnNeutralSign}[X] \ranewthm{obs}{Beobachtung}{\enndeOnNeutralSign}[X] \ranewthm{e.g.}{Beipsiel}{\enndeOnNeutralSign}[X] \ranewthm{fact}{Fakt}{\enndeOnNeutralSign}[X] \ranewthm{rem}{Bemerkung}{\enndeOnNeutralSign}[X] \ranewthm{qstn}{Frage}{\enndeOnNeutralSign}[X] \ranewthm{exer}{Aufgabe}{\enndeOnNeutralSign}[X] \ranewthm{soln}{Lösung}{\enndeOnNeutralSign}[X] \theoremheaderfont{\itshape\bfseries} \theorembodyfont{\upshape} \ranewthm{proof@tmp}{Beweis}{\enndeOfProofSign}[Xdisplaynone] \rathmtransfer{proof@tmp*}{proof} \def\behauptungbeleg@claim{% \iflanguage{british}{Claim}{% \iflanguage{english}{Claim}{% \iflanguage{ngerman}{Behauptung}{% \iflanguage{russian}{Утверждение}{% Claim% }}}}% } \def\behauptungbeleg@pf@kurz{% \iflanguage{british}{Pf}{% \iflanguage{english}{Pf}{% \iflanguage{ngerman}{Bew}{% \iflanguage{russian}{Доказательство}{% Pf% }}}}% } \def\behauptungbeleg{\@ifnextchar\bgroup{\behauptungbeleg@c}{\behauptungbeleg@bes}} \def\behauptungbeleg@c#1{\item[{\bfseries \behauptungbeleg@claim\erlaubeplatz #1.}]} \def\behauptungbeleg@bes{\item[{\bfseries \behauptungbeleg@claim.}]} \def\belegbehauptung{\item[{\bfseries\itshape\behauptungbeleg@pf@kurz.}]} %% **************************************************************** %% ALTE UMGEBUNGEN: %% **************************************************************** \newcolumntype{\RECHTS}[1]{>{\raggedleft}p{#1}} \newcolumntype{\LINKS}[1]{>{\raggedright}p{#1}} \newcolumntype{m}{>{$}l<{$}} \newcolumntype{C}{>{$}c<{$}} \newcolumntype{L}{>{$}l<{$}} \newcolumntype{R}{>{$}r<{$}} \newcolumntype{0}{@{\hspace{0pt}}} \newcolumntype{\LINKSRAND}{@{\hspace{\@totalleftmargin}}} \newcolumntype{h}{@{\extracolsep{\fill}}} \newcolumntype{i}{>{\itshape}} \newcolumntype{t}{@{\hspace{\tabcolsep}}} \newcolumntype{q}{@{\hspace{1em}}} \newcolumntype{n}{@{\hspace{-\tabcolsep}}} \newcolumntype{M}[2]{% >{\begin{minipage}{#2}\begin{math}}% {#1}% <{\end{math}\end{minipage}}% } \newcolumntype{T}[2]{% >{\begin{minipage}{#2}}% {#1}% <{\end{minipage}}% } \setlength{\LTpre}{\baselineskip} \setlength{\LTpost}{0pt} \def\center{\centering} \def\endcenter{} \def\punkteumgebung@genbefehl#1#2#3{ \punkteumgebung@genbefehl@{#1}{#2}{#3}{}{} \punkteumgebung@genbefehl@{multi#1}{#2}{#3}{ \setlength{\columnsep}{10pt}% \setlength{\columnseprule}{0pt}% \begin{multicols}{\thecolumnanzahl}% }{\end{multicols}\nvraum{1}} } \def\punkteumgebung@genbefehl@#1#2#3#4#5{ \expandafter\gdef\csname #1\endcsname{ \@ifnextchar\bgroup{\csname #1@c\endcsname}{\csname #1@bes\endcsname} }%] \expandafter\def\csname #1@c\endcsname##1{ \@ifnextchar[{\csname #1@c@\endcsname{##1}}{\csname #1@c@\endcsname{##1}[\z@]} }%] \expandafter\def\csname #1@c@\endcsname##1[##2]{ \@ifnextchar[{\csname #1@c@@\endcsname{##1}[##2]}{\csname #1@c@@\endcsname{##1}[##2][\z@]} }%] \expandafter\def\csname #1@c@@\endcsname##1[##2][##3]{ \let\alterlinkerRand\gesamtlinkerRand \let\alterrechterRand\gesamtrechterRand \addtolength{\gesamtlinkerRand}{##2} \addtolength{\gesamtrechterRand}{##3} \advance\linewidth -##2% \advance\linewidth -##3% \advance\@totalleftmargin ##2% \parshape\@ne \@totalleftmargin\linewidth% #4 \begin{#2}[\upshape ##1]% \setlength{\parskip}{0.5\baselineskip}\relax% \setlength{\topsep}{\z@}\relax% \setlength{\partopsep}{\z@}\relax% \setlength{\parsep}{\parskip}\relax% \setlength{\itemsep}{#3}\relax% \setlength{\listparindent}{\z@}\relax% \setlength{\itemindent}{\z@}\relax% } \expandafter\def\csname #1@bes\endcsname{ \@ifnextchar[{\csname #1@bes@\endcsname}{\csname #1@bes@\endcsname[\z@]} }%] \expandafter\def\csname #1@bes@\endcsname[##1]{ \@ifnextchar[{\csname #1@bes@@\endcsname[##1]}{\csname #1@bes@@\endcsname[##1][\z@]} }%] \expandafter\def\csname #1@bes@@\endcsname[##1][##2]{ \let\alterlinkerRand\gesamtlinkerRand \let\alterrechterRand\gesamtrechterRand \addtolength{\gesamtlinkerRand}{##1} \addtolength{\gesamtrechterRand}{##2} \advance\linewidth -##1% \advance\linewidth -##2% \advance\@totalleftmargin ##1% \parshape\@ne \@totalleftmargin\linewidth% #4 \begin{#2}% \setlength{\parskip}{0.5\baselineskip}\relax% \setlength{\topsep}{\z@}\relax% \setlength{\partopsep}{\z@}\relax% \setlength{\parsep}{\parskip}\relax% \setlength{\itemsep}{#3}\relax% \setlength{\listparindent}{\z@}\relax% \setlength{\itemindent}{\z@}\relax% } \expandafter\gdef\csname end#1\endcsname{% \end{#2}#5 \setlength{\gesamtlinkerRand}{\alterlinkerRand} \setlength{\gesamtlinkerRand}{\alterrechterRand} } } \def\ritempunkt{{\Large\textbullet}} % \textbullet, $\sqbullet$, $\blacktriangleright$ \setdefaultitem{\ritempunkt}{\ritempunkt}{\ritempunkt}{\ritempunkt} \punkteumgebung@genbefehl{itemise}{compactitem}{\parskip}{}{} \punkteumgebung@genbefehl{kompaktitem}{compactitem}{\z@}{}{} \punkteumgebung@genbefehl{enumerate}{compactenum}{\parskip}{}{} \punkteumgebung@genbefehl{kompaktenum}{compactenum}{\z@}{}{} \let\ALTthebibliography\thebibliography \renewenvironment{thebibliography}[1]{% \begin{ALTthebibliography}{#1} \addcontentsline{toc}{part}{\bibname} }{% \end{ALTthebibliography} } %% **************************************************************** %% NEUE UMGEBUNGEN: %% **************************************************************** \def\matrix#1{\left(\begin{array}[mc]{#1}} \def\endmatrix{\end{array}\right)} \def\smatrix{\left(\begin{smallmatrix}} \def\endsmatrix{\end{smallmatrix}\right)} \def\vector{\begin{matrix}{c}} \def\endvector{\end{matrix}} \def\svector{\begin{smatrix}} \def\endsvector{\end{smatrix}} \def\multiargrekursiverbefehl#1#2#3#4#5#6#7#8{% \expandafter\gdef\csname#1\endcsname #2##1#4{\csname #1@anfang\endcsname##1#3\egroup} \expandafter\def\csname #1@anfang\endcsname##1#3{#5##1\@ifnextchar\egroup{\csname #1@ende\endcsname}{#7\csname #1@mitte\endcsname}} \expandafter\def\csname #1@mitte\endcsname##1#3{#6##1\@ifnextchar\egroup{\csname #1@ende\endcsname}{#7\csname #1@mitte\endcsname}} \expandafter\def\csname #1@ende\endcsname##1{#8} } \multiargrekursiverbefehl{svektor}{[}{;}{]}{\begin{smatrix}}{}{\\}{\\\end{smatrix}} \multiargrekursiverbefehl{vektor}{[}{;}{]}{\begin{matrix}{c}}{}{\\}{\\\end{matrix}} \multiargrekursiverbefehl{vektorzeile}{}{,}{;}{}{&}{}{} \multiargrekursiverbefehl{matlabmatrix}{[}{;}{]}{\begin{smatrix}\vektorzeile}{\vektorzeile}{;\\}{;\end{smatrix}} \def\underbracenodisplay#1{% \mathop{\vtop{\m@th\ialign{##\crcr $\hfil\displaystyle{#1}\hfil$\crcr \noalign{\kern3\p@\nointerlineskip}% \upbracefill\crcr\noalign{\kern3\p@}}}}\limits% } \def\mathe[#1]#2{% \ifthenelse{\equal{\boolinmdframed}{\boolwahr}}{}{\begin{escapeeinzug}} \noindent% \let\eqtagset\boolfalsch \let\eqtaglabel\boolleer \let\eqtagsymb\boolleer \let\alteqtag\eqtag \def\eqtag{\@ifnextchar[{\eqtag@loc@}{\eqtag@loc@[*]}}% \def\eqtag@loc@[##1]{\@ifnextchar\bgroup{\eqtag@loc@@[##1]}{\eqtag@loc@@[##1]{}}}% \def\eqtag@loc@@[##1]##2{% \gdef\eqtagset{\boolwahr} \gdef\eqtaglabel{##1} \gdef\eqtagsymb{##2} }% \def\verticalalign{}% \IfBeginWith{#1}{t}{\def\verticalalign{t}}{}% \IfBeginWith{#1}{m}{\def\verticalalign{c}}{}% \IfBeginWith{#1}{b}{\def\verticalalign{b}}{}% \def\horizontalalign{\null\hfill\null}% \IfEndWith{#1}{l}{}{\null\hfill\null}% \IfEndWith{#1}{r}{\def\horizontalalign{}}{}% \begin{math} \begin{array}[\verticalalign]{0#2}% } \def\endmathe{% \end{array} \end{math}\horizontalalign% \let\eqtag\alteqtag \ifthenelse{\equal{\eqtagset}{\boolwahr}}{\eqtag[\eqtaglabel]{\eqtagsymb}}{} \ifthenelse{\equal{\boolinmdframed}{\boolwahr}}{}{\end{escapeeinzug}}% } \def\longmathe[#1]#2{\relax \let\altarraystretch\arraystretch \renewcommand\arraystretch{1.2}\relax \begin{longtable}[#1]{\LINKSRAND #2} } \def\endlongmathe{ \end{longtable} \renewcommand\arraystretch{\altarraystretch} } \def\einzug{\@ifnextchar[{\indents@}{\indents@[\z@]}}%] \def\indents@[#1]{\@ifnextchar[{\indents@@[#1]}{\indents@@[#1][\z@]}}%] \def\indents@@[#1][#2]{% \begin{list}{}{\relax \setlength{\topsep}{\z@}\relax \setlength{\partopsep}{\z@}\relax \setlength{\parsep}{\parskip}\relax \setlength{\listparindent}{\z@}\relax \setlength{\itemindent}{\z@}\relax \setlength{\leftmargin}{#1}\relax \setlength{\rightmargin}{#2}\relax \let\alterlinkerRand\gesamtlinkerRand \let\alterrechterRand\gesamtrechterRand \addtolength{\gesamtlinkerRand}{#1} \addtolength{\gesamtrechterRand}{#2} }\relax \item[]\relax } \def\endeinzug{% \setlength{\gesamtlinkerRand}{\alterlinkerRand} \setlength{\gesamtlinkerRand}{\alterrechterRand} \end{list}% } \def\escapeeinzug{\begin{einzug}[-\gesamtlinkerRand][-\gesamtrechterRand]} \def\endescapeeinzug{\end{einzug}} \def\programmiercode{ \modulolinenumbers[1] \begin{einzug}[\rtab][\rtab]% \begin{linenumbers}% \fontfamily{cmtt}\fontseries{m}\fontshape{u}\selectfont% \setlength{\parskip}{1\baselineskip}% \setlength{\parindent}{0pt}% } \def\endprogrammiercode{ \end{linenumbers} \end{einzug} } \def\schattiertebox@genbefehl#1#2#3{ \expandafter\gdef\csname #1\endcsname{% \@ifnextchar[{\csname #1@args\endcsname}{\csname #1@args\endcsname[#3]}%]% } \expandafter\def\csname #1@args\endcsname[##1]{% \@ifnextchar[{\csname #1@args@l\endcsname[##1]}{\csname #1@args@n\endcsname[##1]}%]% } \expandafter\def\csname #1@args@l\endcsname[##1][##2]{% \@ifnextchar[{\csname #1@args@l@r\endcsname[##1][##2]}{\csname #1@args@l@n\endcsname[##1][##2]}%]% } \expandafter\def\csname #1@args@n\endcsname[##1]{% \let\boolinmdframed\boolwahr \begin{mdframed}[#2leftmargin=0,rightmargin=0,outermargin=0,innermargin=0,##1] } \expandafter\def\csname #1@args@l@n\endcsname[##1][##2]{% \let\boolinmdframed\boolwahr \begin{mdframed}[#2leftmargin=##2/2,rightmargin=##2/2,outermargin=##2/2,innermargin=##2/2,##1] } \expandafter\def\csname #1@args@l@r\endcsname[##1][##2][##3]{% \let\boolinmdframed\boolwahr \begin{mdframed}[#2leftmargin=##2,rightmargin=##3,outermargin=##2,innermargin=##3,##1] } \expandafter\gdef\csname end#1\endcsname{% \end{mdframed} \let\boolinmdframed\boolfalsch } } \schattiertebox@genbefehl{schattiertebox}{ splittopskip=0,% splitbottomskip=0,% frametitleaboveskip=0,% frametitlebelowskip=0,% skipabove=1\baselineskip,% skipbelow=1\baselineskip,% linewidth=2pt,% linecolor=black,% roundcorner=4pt,% }{ backgroundcolor=leer,% nobreak=true,% } \schattiertebox@genbefehl{schattierteboxdunn}{ splittopskip=0,% splitbottomskip=0,% frametitleaboveskip=0,% frametitlebelowskip=0,% skipabove=1\baselineskip,% skipbelow=1\baselineskip,% linewidth=1pt,% linecolor=black,% roundcorner=2pt,% }{ backgroundcolor=leer,% nobreak=true,% } \def\algorithm{\schattiertebox[backgroundcolor=hellgrau,nobreak=false]} \def\endalgorithm{\endschattiertebox} \def\tikzsetzenode#1{% \tikz[remember picture,baseline,overlay]{\node #1;}% } \def\tikzsetzepfeil#1{% \begin{tikzpicture}[remember picture,overlay,>=latex]% \draw #1;% \end{tikzpicture}% } \def\tikzsetzeoverlay#1{% \begin{tikzpicture}[remember picture,overlay,>=latex]% #1% \end{tikzpicture}% } \def\tikzsetzekreise[#1]#2#3{% \tikzsetzepfeil{% [rounded corners,#1]% ([shift={(-\tabcolsep,0.75\baselineskip)}]#2)% rectangle% ([shift={(\tabcolsep,-0.5\baselineskip)}]#3) }% } \tikzset{ >=stealth, auto, thick, main node/.style={ circle,draw,font=\sffamily\Large\bfseries,minimum size=0pt }, } %% ******************************************************************************** %% FILE: src/setup-layout.tex %% ******************************************************************************** \pagestyle{fancyplain} \@ifundefined{setcitestyle}{% %% do nothing }{% \setcitestyle{numeric-comp,open={[},close={]}} } \def\crefpairconjunction{ und } \def\crefmiddleconjunction{, } \def\creflastconjunction{, und } \raggedbottom %% <- pushes footers up \sloppy \def\headrulewidth{0pt} \def\footrulewidth{0pt} \setlength{\columnsep}{20pt} \setlength{\columnseprule}{1pt} \setlength{\headheight}{11pt} \setlength{\partopsep}{0pt} \setlength{\topsep}{\baselineskip} \setlength{\topskip}{0.5\baselineskip} \setlength{\footskip}{-1\baselineskip} \setlength{\maxdepth}{0pt} \renewcommand{\baselinestretch}{1} \renewcommand{\arraystretch}{1} \setcounter{LTchunksize}{\infty} \setlength{\abovedisplayskip}{0pt} \setlength{\parskip}{1\baselineskip} \def\firstparagraph{\noindent} \def\continueparagraph{\noindent} \hypersetup{ hidelinks=true, } \@addtoreset{chapter}{part} %% nötig für Hyperref. \def\partfont{\documentfont\fontseries{bx}\Huge\selectfont} \def\chapterfont{\documentfont\fontseries{bx}\huge\selectfont} \def\sectionfont{\documentfont\fontseries{bx}\Large\selectfont} \def\subsectionfont{\documentfont\fontseries{bx}\large\selectfont} \def\thepart{\Roman{part}} \generatenestedsecnumbering{arabic}{part}{chapter} \generatenestedsecnumbering{arabic}{chapter}{section} \generatenestedsecnumbering{arabic}{section}{subsection} \generatenestedsecnumbering{arabic}{subsection}{subsubsection} \def\theunitnamepart{\thepart} \def\theunitnamechapter{\theshortchapter} \def\theunitnamesection{\thelongsection} \def\theunitnamesubsection{\thelongsubsection} \def\theunitnamesubsubsection{\thelongsubsubsection} \def\partname{Teil\erlaubeplatz} \def\chaptername{Kapitel\erlaubeplatz} \def\sectionname{\S\erlaubeplatz} \def\subsectionname{} \def\subsubsectionname{} \let\appendix@orig\appendix \def\appendix{% \appendix@orig% \let\boolinappendix\boolwahr \addcontentsline{toc}{part}{\appendixname}% \addtocontents{toc}{\protect\setcounter{tocdepth}{0}} \def\sectionname{Appendix}% \def\theunitnamesection{\Alph{section}}% } \def\notappendix{% \let\boolinappendix\boolfalse \addtocontents{toc}{\protect\setcounter{tocdepth}{1 }} \def\sectionname{}% \def\theunitnamesection{\arabic{section}}% } %% \titlespacing{} %% {linker einzug}{platz oberhalb}{platz unterhalb}[rechter einzug] \titlespacing{\section}{0pt}{\baselineskip}{\baselineskip} \titlespacing{\subsection}{0pt}{\baselineskip}{\baselineskip} \titlespacing{\subsubsection}{0pt}{\baselineskip}{\baselineskip} \titlespacing{\paragraph}{0pt}{0pt}{1em} \titleformat{\part}[display] {\normalfont\headingfont\bfseries\Huge\centering} {% \ifthenelse{\equal{\partname}{}}{% \theunitnamepart% }{% \MakeUppercase{\partname}~\theunitnamepart% }% }{0pt}{% }[\thispagestyle{empty}] \titleformat{\chapter}[frame] {\normalfont\headingfont\bfseries\Large} {% \bedingtesspaceexpand{chaptername}{~}{\theunitnamechapter}% }{0.5em}{% }[\thispagestyle{empty}]%\titlerule%[2pt]% \titleformat{\section}[hang] {\normalfont\headingfont\bfseries\flushleft\large} {% \bedingtesspaceexpand{sectionname}{~}{\theunitnamesection}% }{0.5em} {% } [% \nvraum{0.25}% ] \titleformat{\subsection}[hang] {\normalfont\headingfont\bfseries\flushleft\large} {% \bedingtesspaceexpand{subsectionname}{~}{\theunitnamesubsection}% }{0.5em} {% } [% \nvraum{0.25}% ] \titleformat{\subsubsection}[hang] {\normalfont\headingfont\bfseries\flushleft\large} {% \bedingtesspaceexpand{subsubsectionname}{~}{\theunitnamesubsubsection}% }{0.5em} {% } [% \nvraum{0.25}% ] \def\rafootnotectr{20} \def\incrftnotectr#1{% \addtocounter{#1}{1}% \ifnum\value{#1}>\rafootnotectr\relax \setcounter{#1}{0}% \fi% } \def\footnoteref[#1]{\protected@xdef\@thefnmark{\ref{#1}}\@footnotemark} \let\altfootnotetext\footnotetext \def\footnotetext[#1]#2{\incrftnotectr{footnote}\altfootnotetext[\value{footnote}]{\label{#1}#2}} \let\altfootnotemark\footnotemark %% Undesirable solution, as the text is not hyperlinked. \def\footnotemark[#1]{\text{\textsuperscript{\getrefnumber{#1}}}} \DefineFNsymbols*{custom}{abcdefghijklmnopqrstuvwxyz} \setfnsymbol{custom} \def\footnotelayout{\documentfont\scriptsize} \def\thefootnote{\fnsymbol{footnote}} \def\kopfzeileleer{ \lhead[]{} \chead[]{} \rhead[]{} \lfoot[]{} \cfoot[]{} \rfoot[]{} } \def\kopfzeiledefault{ \lhead[]{} \lhead[]{} \chead[]{} \rhead[]{} \lfoot[]{} \cfoot{\footnotesize\thepage} \rfoot[]{} } \DeclareRobustCommand\crfamily{\fontfamily{pcr}\selectfont} \def\headingfont{\fontfamily{cmss}\selectfont} \def\documentfancyfont{% \gdef\headingfont{\crfamily}% \fontfamily{ccr}\fontseries{m}\selectfont% } \def\documentfont{% \gdef\headingfont{\fontfamily{cmss}\selectfont}% \fontfamily{cmss}\fontseries{m}\selectfont% \renewcommand{\sfdefault}{phv}% \renewcommand{\ttdefault}{pcr}% \renewcommand{\rmdefault}{cmr}% <— funktionieren nicht mit {ptm} \renewcommand{\bfdefault}{bx}% \renewcommand{\itdefault}{it}% \renewcommand{\sldefault}{sl}% \renewcommand{\scdefault}{sc}% \renewcommand{\updefault}{n}% } \allowdisplaybreaks \let\altcleardoublepage\cleardoublepage \let\cleardoublepage\clearpage \def\startdocumentlayoutoptions{ \selectlanguage{ngerman} \setlength{\parskip}{0.5\baselineskip} \setlength{\parindent}{0pt} \kopfzeiledefault \documentfont \normalsize } \def\highlightTerm#1{\emph{#1}} %% ******************************************************************************** %% FILE: srclocal/setup-localmacros.tex %% ******************************************************************************** %% **************************************************************** %% MATHE: %% **************************************************************** \def\cal#1{\mathcal{#1}} \def\reell{\mathbb{R}} \def\kmplx{\mathbb{C}} \def\Torus{\mathbb{T}} \def\rtnl{\mathbb{Q}} \def\intgr{\mathbb{Z}} \def\ntrl{\mathbb{N}} \def\ntrlpos{\mathbb{N}} \def\ntrlzero{\mathbb{N}_{0}} \def\reellNonNeg{\reell_{+}} \def\imageinh{\imath} \def\ReTeil{\mathop{\mathfrak{R}\text{\upshape e}}} \def\ImTeil{\mathop{\mathfrak{I}\text{\upshape m}}} \def\leer{\emptyset} \def\restr#1{\vert_{#1}} \def\ohne{\mathbin{\setminus}} \def\Pot{\mathop{\mathcal{P}}} \def\einser{\mathbf{1}} \def\supp{\mathop{\mathrm{supp}}} \def\brkt#1{\langle{}#1{}\rangle} \def\lsim{\mathop{\sim}} \def\lneg{\mathop{\neg}} \def\land{\mathop{\wedge}} \def\lor{\mathop{\vee}} \def\eps{\varepsilon} \let\altphi\phi \let\altvarphi\varphi \def\phi{\altvarphi} \def\varphi{\altphi} \def\vectorspacespan{\mathop{\text{\upshape Lin}}} \def\dim{\mathop{\text{\upshape dim}}} \def\rank{\mathop{\text{\upshape Rang}}} \def\onematrix{\text{\upshape\bfseries I}} \def\zeromatrix{\text{\upshape\bfseries 0}} \def\zerovector{\text{\upshape\bfseries 0}} \def\graph{\mathop{\text{\upshape Gph}}} \def\domain{\mathop{\text{\upshape dom}}} \def\range{\mathop{\text{\upshape Bild}}} \def\ker{\mathop{\text{\upshape Kern}}} \def\functionspace{\mathop{\text{\upshape Abb}}} \def\id{\text{\upshape id}} \def\modfn{\mathop{\text{\upshape mod}}} \def\divides{\mathbin{\mid}} \def\ndivides{\mathbin{\nmid}} \def\ggT{\mathop{\text{\upshape ggT}}} \def\choose#1#2{\begin{smatrix}#1\\#2\\\end{smatrix}} \makeatother \begin{document} \startdocumentlayoutoptions %% FRONTMATTER: \thispagestyle{plain} %% ******************************************************************************** %% FILE: front/index.tex %% ******************************************************************************** %% ******************************************************************************** %% FILE: front/title.tex %% ******************************************************************************** \begin{titlepage} \null \vraum \noindent\rule{\linewidth}{2pt} {\hraum\LARGE Lineare Algebra I\hraum}\\ {\hraum\LARGE $\oast$\,\rule[0.175\baselineskip]{0.65\linewidth}{1pt}\,$\oast$ \hraum}\\ {\hraum\Large Lösungen zu diversen Aufgaben im Kurs\hraum} \noindent\rule{\linewidth}{2pt} \vraum \noindent \hraum{\footnotesize Raj Dahya}\hraum\\ \hraum{\small \itshape Fakultät für Mathematik und Informatik}\hraum\\ \hraum{\small \itshape Universität Leipzig.}\hraum\\ \hraum{\small Wintersemester 2020/2021 }\hraum \end{titlepage} %% ******************************************************************************** %% FILE: front/foreword.tex %% ******************************************************************************** \chapter*{Vorwort} Dieses Dokument enthält Lösungsansätze zu den Übungsserien, Selbstkontrollenaufgaben, und Quizzes. (Diese werden natürlich \emph{nach} Abgabefristen hochgeladen.) Der Zweck dieser Lösungen besteht darin, Ansätze zu präsentieren, mit denen man seine eigenen Versuche vergleichen kann. %% ******************************************************************************** %% FILE: front/contents.tex %% ******************************************************************************** \kopfzeiledefault \footnotesize \setcounter{tocdepth}{1} \def\contentsname{Inhaltsverzeichnis} \tableofcontents %% HAUPTTEXT: %% ******************************************************************************** %% FILE: body/index.tex %% ******************************************************************************** \setcounternach{part}{1} \part{Übungsserien} \def\chaptername{Übungsserie} %% ******************************************************************************** %% FILE: body/uebung/ueb1.tex %% ******************************************************************************** \setcounternach{chapter}{1} \chapter[Woche 1]{Woche 1} \label{ueb:1} \textbf{ACHTUNG.} Diese Lösungen dienen \emph{nicht} als Musterlösungen sondern eher als Referenz. Hier wird eingehender gearbeitet, als generell verlangt wird. Das Hauptziel hier ist, eine Variant anzubieten, gegen die man seine Versuche vergleichen kann. %% AUFGABE 1-1 \let\altsectionname\sectionname \def\sectionname{Aufgabe} \section[Aufgabe 1]{} \label{ueb:1:ex:1} \let\sectionname\altsectionname Zu bestimmen ist die Lösungsmenge \begin{mathe}[mc]{rcl} L_{\alpha,\beta} &:= &\{ \mathbf{x}\in\reell^{n} \mid A_{\alpha}\mathbf{x}=\mathbf{b}_{\beta} \}\\ \end{mathe} für $\alpha,\beta\in\reell$, wobei $m=3$ und $n=4$, und $A_{\alpha}\in\reell^{m\times n}$ und $\mathbf{b}_{\beta}\in\reell^{m}$ durch \begin{mathe}[mc]{rclqrcl} A_{\alpha} &:= &\begin{matrix}{cccc} 1 &7 &2 &-1\\ 1 &8 &6 &-3\\ 2 &14 &\alpha &-2\\ \end{matrix} &\mathbf{b}_{\beta} &:= &\begin{vector} 4\\ 0\\ \beta\\\end{vector} \end{mathe} gegeben sind. Um die Lösungsmenge zu bestimmen führen wir das Gaußverfahren aus: \begin{algorithm}[\rtab][\rtab] Ursprüngliches LGS $(A_{\alpha}|b_{\beta})$: \begin{mathe}[mc]{c} \begin{matrix}{cccc|c} 1 &7 &2 &-1 &4\\ 1 &8 &6 &-3 &0\\ 2 &14 &\alpha &-2 &\beta\\ \end{matrix}\\ \end{mathe} Wende die Zeilentransformationen {\footnotesize \begin{mathe}[mc]{rcl} Z_{2} &\leftsquigarrow &Z_{2}-Z_{1}\\ Z_{3} &\leftsquigarrow &Z_{3}-2\cdot Z_{1}\\ \end{mathe}} an: \begin{mathe}[mc]{c} \begin{matrix}{cccc|c} \boxed{1} &7 &2 &-1 &4\\ 0 &\boxed{1} &4 &-2 &-4\\ 0 &0 &\boxed{\alpha - 4} &0 &\beta - 8\\ \end{matrix}\\ \end{mathe} \end{algorithm} Die eingezeichneten Einträge markieren die ersten Einträge der Stufen. Es gibt also $2$ oder $3$ Stufen, je nachdem, ob ${\alpha - 4=0}$. Dies führt zu einem Fallunterschied: \begin{enumerate}{\bfseries {Fall} 1.} %% FALL 1 \item $\alpha-4=0$. Das heißt, $\alpha=4$. In diesem Falle hat das augmentierte System genau $2$ Stufen und sieht wie folgt aus: \begin{mathe}[mc]{c} \begin{matrix}{cccc|c} \boxed{1} &7 &2 &-1 &4\\ 0 &\boxed{1} &4 &-2 &-4\\ 0 &0 &0 &0 &\beta - 8\\ \end{matrix}\\ \end{mathe} Dies führt zu zwei weiteren Fällen, denn die $3$. Gleichung ist jetzt genau dann lösbar, wenn $\beta-8=0$. \begin{enumerate}{\bfseries {Fall 1}a.} %% FALL 1a \item $\beta-8\neq 0$. Das heißt, $\beta\neq 8$. Dann ist die $3$. Gleichung und damit das LGS nicht lösbar. Darum erhalten wir $\boxed{L_{\alpha,\beta}=\leer}$. %% FALL 1b \item $\beta-8=0$. Das heißt, $\beta=8$. Dann ist die $3$. Gleichung trivialerweise erfüllt. Das augmentierte System sieht wird zum \begin{mathe}[mc]{c} \begin{matrix}{cccc|c} \boxed{1} &7 &2 &-1 &4\\ 0 &\boxed{1} &4 &-2 &-4\\ 0 &0 &0 &0 &0\\ \end{matrix}\\ \end{mathe} und kann jetzt aufgelöst werden. Wir arbeiten von unten nach oben: \begin{algorithm}[2\rtab][\rtab] Aus der ganzen Zeilenstufenform erschließt sich \begin{mathe}[mc]{c} x_{3},\, x_{4}\,\text{sind frei}\\ \end{mathe} Aus der Stufenform von Gleichungen $2$ und $1$ erschließt sich \begin{mathe}[mc]{rcl} x_{2} &= &-4 - 4x_{3} + 2x_{4}\\ x_{1} &= &4 - 7x_{2} - 2x_{3} + x_{4}\\ &= &4 - 7(-4 - 4x_{3} + 2x_{4}) - 2x_{3} + x_{4}\\ &= &32 + 26x_{3} + -13x_{4}\\ \end{mathe} Zusammengefasst erhalten wir die allgemeine Form der Lösung: \begin{mathe}[mc]{rcl} \mathbf{x} &= &\begin{svector} x_{1}\\ x_{2}\\ x_{3}\\ x_{4}\\\end{svector}\\ &= &\begin{svector} 32 + 26x_{3} + -13x_{4}\\ -4 - 4x_{3} + 2x_{4}\\ x_{3}\\ x_{4}\\\end{svector}\\ &= &\begin{svector} 32 + 26x_{3} + -13x_{4}\\ -4 - 4x_{3} + 2x_{4}\\ 0 + 1x_{3} + 0x_{4}\\ 0 + 0x_{3} + 1x_{4}\\\end{svector}\\ &= &\begin{svector} 32\\ -4\\ 0\\ 0\\\end{svector} + \begin{svector} 26x_{3}\\ -4x_{3}\\ 1x_{3}\\ 0x_{3}\\\end{svector} + \begin{svector} -13x_{4}\\ 2x_{4}\\ 1x_{4}\\ 1x_{4}\\\end{svector}\\ &= &\begin{svector} 32\\ -4\\ 0\\ 0\\\end{svector} + x_{3}\cdot\begin{svector} 26\\ -4\\ 1\\ 0\\\end{svector} + x_{4}\cdot\begin{svector} -13\\ 2\\ 1\\ 1\\\end{svector}\\ \end{mathe} mit $x_{3}$, $x_{4}$ frei wählbar. \end{algorithm} Also erhalten wird in diesem Falle $\boxed{ L_{\alpha,\beta}=\left\{ \begin{svector} 32\\ -4\\ 0\\ 0\\\end{svector} + t_{1}\cdot\begin{svector} 26\\ -4\\ 1\\ 0\\\end{svector} + t_{2}\cdot\begin{svector} -13\\ 2\\ 1\\ 1\\\end{svector} \mid t_{1}, t_{2}\in\reell \right\} }$, oder etwas kompakter formuliert, ${L_{\alpha,\beta}=\begin{svector} 32\\ -4\\ 0\\ 0\\\end{svector} + \vectorspacespan\left\{\begin{svector} 26\\ -4\\ 1\\ 0\\\end{svector}, \begin{svector} -13\\ 2\\ 1\\ 1\\\end{svector}\right\}}$. \end{enumerate} %% FALL 2 \item $\alpha-4\neq 0$. Das heißt, $\alpha\neq 4$. In diesem Falle hat das augmentierte System genau $3$ Stufen und diesmal ist nur $x_{4}$ frei. Man beachte, dass dies im Grunde genau wie Fall 1b ist, nur dass wir zusätzlich Gleichung 3 beachten und $x_{3}$ bestimmen müssen. \begin{algorithm}[2\rtab][\rtab] Aus der Stufenform von Gleichungen $3$ ergibt sich \begin{mathe}[mc]{rcl} x_{3} &= &\frac{\beta-8}{\alpha-4}\\ \end{mathe} Der Rest der Lösung des Gleichungssystems verhält sich genau wie im Fall 3b, das heißt \begin{mathe}[mc]{rcl} \mathbf{x} &= &\begin{svector} 32\\ -4\\ 0\\ 0\\\end{svector} + x_{3}\cdot\begin{svector} 26\\ -4\\ 1\\ 0\\\end{svector} + x_{4}\cdot\begin{svector} -13\\ 2\\ 1\\ 1\\\end{svector}\\ &= &\begin{svector} 32\\ -4\\ 0\\ 0\\\end{svector} + \frac{\beta-8}{\alpha-4}\cdot\begin{svector} 26\\ -4\\ 1\\ 0\\\end{svector} + x_{4}\cdot\begin{svector} -13\\ 2\\ 1\\ 1\\\end{svector},\\ \end{mathe} wobei $x_{4}$ frei wählbar ist. \end{algorithm} Also erhalten wird in diesem Falle $\boxed{ L_{\alpha,\beta}=\left\{ \begin{svector} 32\\ -4\\ 0\\ 0\\\end{svector} + \frac{\beta-8}{\alpha-4}\cdot\begin{svector} 26\\ -4\\ 1\\ 0\\\end{svector} + t\cdot\begin{svector} -13\\ 2\\ 1\\ 1\\\end{svector} \mid t\in\reell \right\} }$, oder etwas kompakter formuliert, ${L_{\alpha,\beta}=\begin{svector} 32\\ -4\\ 0\\ 0\\\end{svector} + \frac{\beta-8}{\alpha-4}\cdot\begin{svector} 26\\ -4\\ 1\\ 0\\\end{svector} + \vectorspacespan\left\{\begin{svector} -13\\ 2\\ 1\\ 1\\\end{svector}\right\}}$. \end{enumerate} Wir fassen die Lösung für alle Fälle zusammen: \begin{mathe}[mc]{rcl} L_{\alpha,\beta} &= &\begin{cases}[m]{lcl} \leer &: &\alpha=4,\,\beta\neq 8\\ \mathbf{u} + \vectorspacespan\{\mathbf{v},\mathbf{w}\} &: &\alpha=4,\,\beta=8\\ \mathbf{u} + \frac{\alpha-4}{\beta-8}\mathbf{v} + \vectorspacespan\{\mathbf{w}\} &: &\alpha\neq 4\\ \end{cases} \end{mathe} für alle $\alpha,\beta\in\reell$, wobei $\mathbf{u} = \begin{svector} 32\\ -4\\ 0\\ 0\\\end{svector}$, $\mathbf{v} = \begin{svector} 26\\ -4\\ 1\\ 0\\\end{svector}$, $\mathbf{w} = \begin{svector} -13\\ 2\\ 1\\ 1\\\end{svector}$. %% AUFGABE 1-2 \let\altsectionname\sectionname \def\sectionname{Aufgabe} \section[Aufgabe 2]{} \label{ueb:1:ex:2} \let\sectionname\altsectionname \begin{schattierteboxdunn} \begin{satz} \makelabel{satz:main:ueb:1:ex:2} Angewandt auf die erweiterte Koeffizientenmatrix eines linearen Gleichungssystems verändern die elementaren Zeilenumformungen vom Typ (I), (II) und (III) die Menge der Lösungen nicht. \end{satz} \end{schattierteboxdunn} Wir beweisen \Cref{satz:main:ueb:1:ex:2} mithilfe der folgenden Teilergebnisse. \begin{lemm} \makelabel{lemm:1:ueb:1:ex:2} Seien $m,n\in\ntrlpos$ und $A\in\reell^{m\times n}$ und $\mathbf{b}\in\reell^{m}$. Für $i,j\in\{1,2,\ldots,m\}$ mit $i\neq j$ bezeichne mit \begin{mathe}[mc]{rcl} (A|\mathbf{b}) &\overset{I;i,j}{\rightsquigarrow} &(A'|\mathbf{b}')\\ \end{mathe} die Anwendung von Zeilentransformation (I) auf $(A|\mathbf{b})$, wobei Zeile${}_{i}$ und Zeile${}_{j}$ umgetauscht werden, was in $(A'|\mathbf{b}')$ resultiert. Dann für alle ${\mathbf{x}\in\reell^{n}}$, falls $\mathbf{x}$ eine Lösung für $(A|\mathbf{b})$ ist, dann ist $\mathbf{x}$ eine Lösung für $(A'|\mathbf{b}')$. \end{lemm} \begin{einzug}[\rtab][\rtab] \begin{proof} Betrachte den Fall $ij$ lässt sich analog zeigen. Falls $i=j$ bleibt das System unverändert, sodass die Behauptung trivialerweise gilt. \end{proof} \end{einzug} \begin{lemm} \makelabel{lemm:2:ueb:1:ex:2} Seien $m,n\in\ntrlpos$ und $A\in\reell^{m\times n}$ und $\mathbf{b}\in\reell^{m}$. Für ${i\in\{1,2,\ldots,m\}}$ und ${\alpha\in\reell\ohne\{0\}}$ bezeichne mit \begin{mathe}[mc]{rcl} (A|\mathbf{b}) &\overset{II;i,\alpha}{\rightsquigarrow} &(A'|\mathbf{b}')\\ \end{mathe} die Anwendung von Zeilentransformation (II) auf $(A|\mathbf{b})$, wobei Zeile${}_{i}$ durch $\alpha\cdot$Zeile${}_{i}$ ersetzt wird, was in $(A'|\mathbf{b}')$ resultiert. Dann für alle ${\mathbf{x}\in\reell^{n}}$, falls $\mathbf{x}$ eine Lösung für $(A|\mathbf{b})$ ist, dann ist $\mathbf{x}$ eine Lösung für $(A'|\mathbf{b}')$. \end{lemm} \begin{einzug}[\rtab][\rtab] \begin{proof} Es gilt \begin{longtable}[mc]{RL} &\text{$\mathbf{x}$ eine Lösung für $(A|\mathbf{b})$}\\ \Longrightarrow &{\scriptsize \left\{ \begin{array}[m]{crccccclcl} &(a_{1,1}x_{1} &+ &a_{1,2}x_{2} &+ &\cdots &+ &a_{1,n}x_{n} &= &b_{1})\\ \text{und} &(a_{2,1}x_{1} &+ &a_{2,2}x_{2} &+ &\cdots &+ &a_{2,n}x_{n} &= &b_{2})\\ \cdots\\ \text{und} &(a_{i,1}x_{1} &+ &a_{i,2}x_{2} &+ &\cdots &+ &a_{i,n}x_{n} &= &b_{i})\\ \cdots\\ \text{und} &(a_{m,1}x_{1} &+ &a_{m,2}x_{2} &+ &\cdots &+ &a_{m,n}x_{n} &= &b_{m}) \end{array} \right.}\\ \\ \Longrightarrow &{\scriptsize \left\{ \begin{array}[m]{crccccclcl} &(a_{1,1}x_{1} &+ &a_{1,2}x_{2} &+ &\cdots &+ &a_{1,n}x_{n} &= &b_{1})\\ \text{und} &(a_{2,1}x_{1} &+ &a_{2,2}x_{2} &+ &\cdots &+ &a_{2,n}x_{n} &= &b_{2})\\ \cdots\\ \text{und} &(\alpha\cdot (a_{i,1}x_{1} &+ &a_{i,2}x_{2} &+ &\cdots &+ &a_{i,n}x_{n}) &= &\alpha\cdot b_{i})\\ \cdots\\ \text{und} &(a_{m,1}x_{1} &+ &a_{m,2}x_{2} &+ &\cdots &+ &a_{m,n}x_{n} &= &b_{m}) \end{array} \right.}\\ \\ \Longrightarrow &{\scriptsize \left\{ \begin{array}[m]{crccccclcl} &(a_{1,1}x_{1} &+ &a_{1,2}x_{2} &+ &\cdots &+ &a_{1,n}x_{n} &= &b_{1})\\ \text{und} &(a_{2,1}x_{1} &+ &a_{2,2}x_{2} &+ &\cdots &+ &a_{2,n}x_{n} &= &b_{2})\\ \cdots\\ \text{und} &(\alpha\cdot a_{i,1}x_{1} &+ &\alpha\cdot a_{i,2}x_{2} &+ &\cdots &+ &\alpha\cdot a_{i,n}x_{n} &= &\alpha\cdot b_{i})\\ \cdots\\ \text{und} &(a_{m,1}x_{1} &+ &a_{m,2}x_{2} &+ &\cdots &+ &a_{m,n}x_{n} &= &b_{m}) \end{array} \right.}\\ \\ &\text{$\mathbf{x}$ eine Lösung für $(A'|\mathbf{b})'$, da $(A|\mathbf{b})\overset{II;i,\alpha}{\rightsquigarrow}(A'|\mathbf{b}')$.} \end{longtable} Also gilt die Behauptung. \end{proof} \end{einzug} \begin{lemm} \makelabel{lemm:3:ueb:1:ex:2} Seien $m,n\in\ntrlpos$ und $A\in\reell^{m\times n}$ und $\mathbf{b}\in\reell^{m}$. Für ${i,j\in\{1,2,\ldots,m\}}$ mit $i\neq j$ und $\alpha\in\reell$ bezeichne mit \begin{mathe}[mc]{rcl} (A|\mathbf{b}) &\overset{III;i,j,\alpha}{\rightsquigarrow} &(A'|\mathbf{b}')\\ \end{mathe} die Anwendung von Zeilentransformation (III) auf $(A|\mathbf{b})$, wobei Zeile${}_{i}$ durch die Addition von Zeile${}_{i}$ mit $\alpha\cdot$Zeile${}_{j}$ ersetzt wird, was in $(A'|\mathbf{b}')$ resultiert. Dann für alle ${\mathbf{x}\in\reell^{n}}$, falls $\mathbf{x}$ eine Lösung für $(A|\mathbf{b})$ ist, dann ist $\mathbf{x}$ eine Lösung für $(A'|\mathbf{b}')$. \end{lemm} \begin{einzug}[\rtab][\rtab] \begin{proof} Es gilt \begin{longtable}[mc]{RL} &\text{$\mathbf{x}$ eine Lösung für $(A|\mathbf{b})$}\\ \Longrightarrow &{\scriptsize \left\{ \begin{array}[m]{crccccclcl} &(a_{1,1}x_{1} &+ &a_{1,2}x_{2} &+ &\cdots &+ &a_{1,n}x_{n} &= &b_{1})\\ \text{und} &(a_{2,1}x_{1} &+ &a_{2,2}x_{2} &+ &\cdots &+ &a_{2,n}x_{n} &= &b_{2})\\ \cdots\\ \text{und} &(a_{i,1}x_{1} &+ &a_{i,2}x_{2} &+ &\cdots &+ &a_{i,n}x_{n} &= &b_{i})\\ \cdots\\ \text{und} &(a_{m,1}x_{1} &+ &a_{m,2}x_{2} &+ &\cdots &+ &a_{m,n}x_{n} &= &b_{m}) \end{array} \right.}\\ \\ \Longrightarrow &{\scriptsize \left\{ \begin{array}[m]{crccccclcl} &(a_{1,1}x_{1} &+ &a_{1,2}x_{2} &+ &\cdots &+ &a_{1,n}x_{n} &= &b_{1})\\ \text{und} &(a_{2,1}x_{1} &+ &a_{2,2}x_{2} &+ &\cdots &+ &a_{2,n}x_{n} &= &b_{2})\\ \cdots\\ \text{und} &(a_{i,1}x_{1} &+ &a_{i,2}x_{2} &+ &\cdots &+ &a_{i,n}x_{n} + \alpha\cdot b_{j} &= &b_{i} + \alpha\cdot b_{j})\\ \cdots\\ \text{und} &(a_{m,1}x_{1} &+ &a_{m,2}x_{2} &+ &\cdots &+ &a_{m,n}x_{n} &= &b_{m}) \end{array} \right.}\\ \\ \Longrightarrow &{\scriptsize \left\{ \begin{array}[m]{crccccclcl} &(a_{1,1}x_{1} &+ &a_{1,2}x_{2} &+ &\cdots &+ &a_{1,n}x_{n} &= &b_{1})\\ \text{und} &(a_{2,1}x_{1} &+ &a_{2,2}x_{2} &+ &\cdots &+ &a_{2,n}x_{n} &= &b_{2})\\ \cdots\\ \text{und} &(a_{i,1}x_{1} &+ &a_{i,2}x_{2} &+ &\cdots &+ &a_{i,n}x_{n}\\ &+\alpha\cdot a_{j,1}x_{1} &+ &\alpha\cdot a_{j,2}x_{2} &+ &\cdots &+ &\alpha\cdot a_{j,n}x_{n} &= &b_{i} + \alpha\cdot b_{j})\\ \cdots\\ \text{und} &(a_{m,1}x_{1} &+ &a_{m,2}x_{2} &+ &\cdots &+ &a_{m,n}x_{n} &= &b_{m}) \end{array} \right.}\\ \\ &\text{da laut der $j$-ten Gleichung gilt ${b_{j}=\sum_{k=1}^{m}a_{j,k}x_{k}}$}\\ \\ \Longrightarrow &{\scriptsize \left\{ \begin{array}[m]{crccccclcl} &(a_{1,1}x_{1} &+ &a_{1,2}x_{2} &+ &\cdots &+ &a_{1,n}x_{n} &= &b_{1})\\ \text{und} &(a_{2,1}x_{1} &+ &a_{2,2}x_{2} &+ &\cdots &+ &a_{2,n}x_{n} &= &b_{2})\\ \cdots\\ \text{und} &(a'_{i,1}x_{1} &+ &a'_{i,2}x_{2} &+ &\cdots &+ &a'_{i,n}x_{n} &= &b'_{i})\\ \cdots\\ \text{und} &(a_{m,1}x_{1} &+ &a_{m,2}x_{2} &+ &\cdots &+ &a_{m,n}x_{n} &= &b_{m}), \end{array} \right.}\\ \\ &\text{wobei $a'_{i,k}=a_{i,k}+\alpha\cdot a_{j,k}$ für alle $k$ und $b'_{i}=b_{i}+\alpha\cdot b_{j}$}\\ \\ \Longrightarrow &\text{$\mathbf{x}$ eine Lösung für $(A'|\mathbf{b})'$, da $(A|\mathbf{b})\overset{III;i,j,\alpha}{\rightsquigarrow}(A'|\mathbf{b}')$.} \end{longtable} Also gilt die Behauptung. \end{proof} \end{einzug} Endlich können wir \Cref{satz:main:ueb:1:ex:2} beweisen: \begin{proof}[von \Cref{satz:main:ueb:1:ex:2}] Seien $m,n\in\ntrlpos$ und $A\in\reell^{m\times n}$ und $\mathbf{b}\in\reell^{m}$. Seien $A'\in\reell^{m\times n}$ und $\mathbf{b}'\in\reell^{m}$, so dass $(A|\mathbf{b})$ durch eine Transformation der Art (I), (II) oder (III) aus $(A|\mathbf{b})$ entsteht. Das heißt, entweder \begin{mathe}[mc]{lrcl} \eqtag[eq:0:\beweislabel] &(A|\mathbf{b}) &\overset{I;i,j}{\rightsquigarrow} &(A'|\mathbf{b}')\\ \text{oder} &(A|\mathbf{b}) &\overset{I;i,\alpha}{\rightsquigarrow} &(A'|\mathbf{b}')\\ \text{oder} &(A|\mathbf{b}) &\overset{III;i,j,\alpha}{\rightsquigarrow} &(A'|\mathbf{b}')\\ \end{mathe} gilt, für ein $i,j\in\{1,2,\ldots,m\}$ mit $i\neq j$ und $\alpha\in\reell\ohne\{0\}$.\\ \textbf{Zu zeigen:} \begin{mathe}[mc]{rcl} \eqtag[eq:1:\beweislabel] \{\mathbf{x}\in\reell^{n}\mid\mathbf{x}\text{ eine Lösung für }(A|\mathbf{b})\} &= &\{\mathbf{x}\in\reell^{n}\mid\mathbf{x}\text{ eine Lösung für }(A|\mathbf{b})\}.\\ \end{mathe} Wir zeigen dies in zwei Teile: \uline{\bfseries ($\subseteq$.)}\\ Sei $\mathbf{x}\in\reell^{n}$ ein beliebiges Element aus der linken Menge, d.\,h. $\mathbf{x}$ ist eine Lösung zu $(A|\mathbf{b})$. Laut \Cref{lemm:1:ueb:1:ex:2} + \Cref{lemm:2:ueb:1:ex:2} + \Cref{lemm:3:ueb:1:ex:2} und wegen \eqcref{eq:0:\beweislabel} erhalten wir, dass $\mathbf{x}$ eine Lösung zu $(A'|\mathbf{b}')$ ist, d.\,h. $\mathbf{x}$ liegt in der rechten Menge. Also ist die linke Menge in der rechten enthalten. \uline{\bfseries ($\supseteq$.)}\\ Man beachte zuerst, dass sich die Transformation in \eqcref{eq:0:\beweislabel} umkehren lässt---\text{und zwar durch Elementartransformationen}. Es ist einfach zu sehen, dass entweder \begin{mathe}[mc]{lrcl} &(A'|\mathbf{b}') &\overset{I;i,j}{\rightsquigarrow} &(A|\mathbf{b})\\ \text{oder} &(A'|\mathbf{b}') &\overset{I;i,\alpha^{-1}}{\rightsquigarrow} &(A|\mathbf{b})\\ \text{oder} &(A'|\mathbf{b}') &\overset{III;i,j,-\alpha}{\rightsquigarrow} &(A|\mathbf{b}).\\ \end{mathe} Die Situation ist also analog zum $\subseteq$-Teil. Darum gilt die $\supseteq$-Inklusion in \eqcref{eq:1:\beweislabel}. \end{proof} \clearpage %% AUFGABE 1-3 \let\altsectionname\sectionname \def\sectionname{Aufgabe} \section[Aufgabe 3]{} \label{ueb:1:ex:3} \let\sectionname\altsectionname Für diese Aufgabe wird das Konzept der \emph{linearen Unabhängigkeit} aus Kapitel 5 angewandt. \begin{defn} Seien $m,n\in\ntrlpos$ mit $m>n$ und seien $A\in\reell^{m\times n}$, $\mathbf{b}\in\reell^{m}$, und $I\subseteq\{1,2,\ldots,m\}$. Bezeichne mit $(A|\mathbf{b})_{I}$ die erweiterte Koeffizientenmatrix $(A|\mathbf{b})$, die auf die Zeilen mit Indexes aus $I$ (in bspw. aufsteigender Reihenfolge) reduziert ist. \end{defn} \begin{e.g.} Für $(A|\mathbf{b})$ gleich {\scriptsize \begin{mathe}[mc]{c} \begin{matrix}{ccc|c} -5 &0 &0 &-7\\ 4 &-6 &-10 &6\\ -2 &-6 &-6 &9\\ -7 &4 &-1 &-5\\ 4 &-5 &2 &-9\\ -5 &8 &-7 &-5\\ \end{matrix} \end{mathe}} und $I=\{2,5,6\}$ ist $(A|\mathbf{b})_{I}$ gleich {\scriptsize \begin{mathe}[bc]{c} \begin{matrix}{ccc|c} 4 &-6 &-10 &6\\ 4 &-5 &2 &-9\\ -5 &8 &-7 &-5\\ \end{matrix}. \end{mathe}} \nvraum{1} \end{e.g.} Mit diesem Mittel können wir nun die Hauptaussage in der Aufgabe formulieren: \begin{schattierteboxdunn} \begin{satz} \makelabel{satz:main:ueb:1:ex:3} Seien $m,n\in\ntrlpos$ mit $m>n$ und seien $A\in\reell^{m\times n}$ und $\mathbf{b}\in\reell^{m}$. Falls $(A|\mathbf{b})$ unlösbar ist, dann existiert $I\subseteq\{1,2,\ldots,m\}$ mit $|I|=n+1$, so dass $(A|\mathbf{b})_{I}$ unlösbar ist. \end{satz} \end{schattierteboxdunn} \begin{einzug}[\rtab][\rtab] \begin{proof}[*][\Cref{\beweislabel}] Es stehen nun die \emph{Zeilen} der Matrix $A$ im Fokus. Wir verwandeln diese in Vektoren, d.\,h. setze \begin{mathe}[mc]{c} \mathbf{z}^{(i)}\in\reell^{n}\,\text{die $i$-te Zeile von $A$ als Vektor geschrieben} \end{mathe} für $i\in\{1,2,\ldots,m\}$. Da ${\mathbf{z}^{(1)},\mathbf{z}^{(2)},\ldots,\mathbf{z}^{(m)}\in\reell^{n}}$, können wir eine \emph{maximale Menge} ${I_{0}\subseteq\{1,2,\ldots,m\}}$ finden, so dass $(\mathbf{z}^{(i)})_{i\in I_{0}}$ aus linear unabhängigen Vektoren besteht. Wegen der Dimension von $\reell^{n}$ gilt ${|I|\leq\min\{m,n\}=n}$. Sei ${k\in\{1,2,\ldots,m\}\ohne I_{0}}$ beliebig. Wegen Maximalität muss $(\mathbf{z}^{(i)})_{i\in I_{0}\cup\{k\}}$ \emph{linear abhängig} sein. Und wegen der linearen Unabhängigkeit von $(\mathbf{z}^{(i)})_{i\in I_{0}}$ existieren (eindeutige) Koeffizienten $c_{k,i}\in\reell$ für $i\in I_{0}$ so dass \begin{mathe}[mc]{rcl} \eqtag[eq:1:\beweislabel] \mathbf{z}^{(k)} &= &\sum_{i\in I_{0}:~}c_{k,i}\mathbf{z}^{(i)}\\ \end{mathe} gilt. Um nun die Hauptaussage zu zeigen, nehmen wir an, dass $(A|\mathbf{b})$ unlösbar ist. \textbf{Zu zeigen:} Es gibt eine Teilmenge ${I\subseteq\{1,2,\ldots,m\}}$ mit ${|I|=n+1}$, so dass $(A|\mathbf{b})_{I}$ unlösbar ist. \fbox{Angenommen, dies sei nicht der Fall.} Aus dieser Annahme leiten wir folgende Behauptungen ab: \begin{kompaktitem}[\rtab][\rtab] \behauptungbeleg{1} Die Verhältnisse zwischen den Zeilenvektoren in \eqcref{eq:1:\beweislabel} gelten auch für die Einträge aus $\mathbf{b}$. Das heißt \begin{mathe}[mc]{rcl} \eqtag[eq:2:\beweislabel] b_{k} &= &\sum_{i\in I_{0}:~}c_{k,i}b_{i}\\ \end{mathe} für alle ${k\in\{1,2,\ldots,m+1\}\ohne I_{0}}$.\\ \voritemise \belegbehauptung Sei $k\in\{1,2,\ldots,m+1\}\ohne I_{0}$ beliebig. Da $|I_{0}|\leq nn$ sieht nun die Zeilenstufenform, also $(A^{(N)}|\mathbf{b}^{(N)})$, folgendermaßen aus: {\scriptsize \begin{mathe}[mc]{rcl} \begin{matrix}{cccccccc|c} \underbrace{0\,0\,\ldots\,0}_{\ell_{1}} &\gamma_{1} &\cdots\cdots &\ast &\cdots\cdots &\cdots\cdots &\ast &\cdots\cdots &b^{(N)}_{1}\\ 0\,0\,\ldots\,0 &0 &\underbrace{0\,0\,\ldots\,0}_{\ell_{2}} &\gamma_{2} &\cdots\cdots &\cdots\cdots &\ast &\cdots\cdots &b^{(N)}_{2}\\ \vdots & & & & & & &\vdots\\ 0\,0\,\ldots\,0 &0 &0\,0\,\ldots\,0 &0 &\cdots\cdots &\underbrace{0\,0\,\ldots\,0}_{\ell_{r}} &\gamma_{r} &\cdots\cdots &b^{(N)}_{r}\\ 0\,0\,\ldots\,0 &0 &0\,0\,\ldots\,0 &0 &\cdots\cdots &0\,0\,\ldots\,0 &0 &\cdots\cdots &b^{(N)}_{r+1}\\ \vdots & & & & & & &\vdots\\ 0\,0\,\ldots\,0 &0 &0\,0\,\ldots\,0 &0 &\cdots\cdots &0\,0\,\ldots\,0 &0 &\cdots\cdots &b^{(N)}_{m}\\ \end{matrix} \end{mathe}} wobei $r\in\ntrlzero$ die Anzahl der Stufen ist, ${\ell_{1},\ell_{2},\ldots,\ell_{r}\in\ntrlzero}$, und $\gamma_{1},\gamma_{2},\ldots,\gamma_{r}\in\reell\ohne\{0\}$ die Hauptkoeffizienten der Stufen sind. Es muss nun $0\leq r\leq \min\{m,n\}=n$ gelten. Jetzt kann man leicht dafür argumentiere, dass (1) die Zeilenstufenform, $(A^{(N)}|\mathbf{b}^{(N)})$, die Implikation erfüllt. Dann aufgrund der Umkehrbarkeit der Elementartransformationen, reicht es aus zu zeigen, dass (2): wenn ${(A',\mathbf{b}')\rightsquigarrow(A'',\mathbf{b}'')}$ und wenn $(A',\mathbf{b}')$ die Implikation erfüllt, dann erfüllt $(A'',\mathbf{b}'')$ die Implikation. Dies ist nur etwas mühseliger und die Argumentation von (2) führt letzten Endes zu ähnlichen Ideen, die im Beweis oben vorkommen. \end{rem} %% ******************************************************************************** %% FILE: body/uebung/ueb2.tex %% ******************************************************************************** \setcounternach{chapter}{2} \chapter[Woche 2]{Woche 2} \label{ueb:2} \textbf{ACHTUNG.} Diese Lösungen dienen \emph{nicht} als Musterlösungen sondern eher als Referenz. Hier wird eingehender gearbeitet, als generell verlangt wird. Das Hauptziel hier ist, eine Variant anzubieten, gegen die man seine Versuche vergleichen kann. %% AUFGABE 2-1 \let\altsectionname\sectionname \def\sectionname{Aufgabe} \section[Aufgabe 1]{} \label{ueb:2:ex:1} \let\sectionname\altsectionname \begin{schattierteboxdunn} \begin{satz}[vgl. {\cite[Korollar 1.3.3]{sinn2020}}] \makelabel{satz:main:ueb:2:ex:1} Sei $V$ ein Vektorraum über $\reell$ wie $\reell^{n}$ für ein $n\in\ntrlpos$. Seien $\mathbf{v},\mathbf{w}\in V$ mit $\mathbf{v}\neq \mathbf{w}$ und $\mathbf{w}\neq\zerovector$ und sei \begin{mathe}[mc]{rcl} L &:= &\{s\mathbf{v} + (1-s)\mathbf{w}\mid s\in\reell\}\\ \end{mathe} die Verbindungsgerade zw. $\mathbf{v}$ und $\mathbf{w}$. Dann gilt $\zerovector\in L$ $\Leftrightarrow$ $\exists{c\in\reell:~}\mathbf{v}=c\mathbf{w}$. \end{satz} \end{schattierteboxdunn} \begin{proof} Der Beweis wird in zwei Teilen gezeigt. \hinRichtung Angenommen, $\zerovector\in L$. \textbf{Zu zeigen:} $\exists{c\in\reell:~}\mathbf{v}=c\mathbf{w}$.\\ Per Definition von $L$ existiert ein $s\in\reell$, so dass sich $\zerovector$ als $\zerovector=s\mathbf{v} + (1-s)\mathbf{w}$ darstellen lässt. Daraus lässt sich ableiten: \begin{mathe}[mc]{rcl} \zerovector=s\mathbf{v} + (1-s)\mathbf{w} &\Longleftrightarrow &s\mathbf{v} = (s-1)\mathbf{w}\\ &\Longleftrightarrow &\underbrace{% (s=0\,\text{und}\,\mathbf{w}=s(\mathbf{w}-\mathbf{v})=\zerovector) }_{% \text{unmöglich, da $\mathbf{w}\neq\zerovector$ per Voraussetzung} } \,\text{oder}\,(s\neq 0\,\text{und}\,\mathbf{v} = ((s-1)/s)\mathbf{w})\\ &\Longleftrightarrow &s\neq 0\,\text{und}\,\mathbf{v} = ((s-1)/s)\mathbf{w}\\ &\Longrightarrow &\exists{c\in\reell:~}\mathbf{v} = c\mathbf{w}.\\ \end{mathe} \herRichtung Angenommen, $\mathbf{v} = c\mathbf{w}$ für ein $c\in\reell$. \textbf{Zu zeigen:} $\zerovector\in L$.\\ Per Voraussetzung gilt nun $\mathbf{v}\neq\mathbf{w}$, sodass $c=1$ direkt ausgeschlossen ist.\\ Setze nun \fbox{$s:=\frac{1}{1-c}\in\reell$}, was wohldefiniert ist, da $c\neq 1$.\\ Man berechnet nun \begin{mathe}[mc]{rcccccccl} \overbrace{s\mathbf{v}+(1-s)\mathbf{w}}^{\in L,\,\text{per Definition}} &= &\frac{1}{1-c}c\mathbf{w}+(1-\frac{1}{1-c})\mathbf{w} &= &(\underbrace{\frac{c}{1-c}+1-\frac{1}{1-c}}_{=\frac{c-1}{1-c}+1=0})\mathbf{w} &= &0\mathbf{w} &= &\zerovector.\\ \end{mathe} Darum gilt $\zerovector\in L$. \end{proof} %% AUFGABE 2-2 \clearpage \let\altsectionname\sectionname \def\sectionname{Aufgabe} \section[Aufgabe 2]{} \label{ueb:2:ex:2} \let\sectionname\altsectionname \begin{enumerate}{\bfseries (a)} %% AUFGABE 2-2a \item \begin{schattierteboxdunn} \begin{satz} \makelabel{satz:main:ueb:2:ex:2a} Seien $\mathbf{v},\mathbf{v}^{\prime},\mathbf{w},\mathbf{w}^{\prime}\in\reell^{2}$ mit $\mathbf{w},\mathbf{w}^{\prime}\neq\zerovector$. Seien $L:=\{\mathbf{v}+t\mathbf{w}\mid t\in\reell\}$ und $L^{\prime}:=\{\mathbf{v}^{\prime}+s\mathbf{w}^{\prime}\mid s\in\reell\}$. Angenommen, $L\neq L^{\prime}$. Dann sind folgende Aussagen äquivalent: \begin{kompaktenum}{(i)} \item\punktlabel{1} $L\cap L^{\prime}=\leer$; \item\punktlabel{2} $\mathbf{w},\mathbf{w}^{\prime}$ sind kolinear, d.\,h. $\exists{c\in\reell:~}\mathbf{w}=c\mathbf{w}^{\prime}$. \end{kompaktenum} \nvraum{1} \end{satz} \end{schattierteboxdunn} \begin{proof} Der Beweis wird in zwei Teilen gezeigt. \hinRichtung{1}{2} Angenommen, $L\cap L^{\prime}=\leer$. \textbf{Zu zeigen:} $\exists{c\in\reell:~}\mathbf{w}=c\mathbf{w}^{\prime}$.\\ \fbox{Angenommen, dies sei nicht der Fall.}\\ Da $\mathbf{w},\mathbf{w}^{\prime}\neq\zerovector$ bedeutet dies, dass $\mathbf{w},\mathbf{w}^{\prime}$ \emph{linear unabhängig} sind. ($\to$ Warum??)\\ Also gilt für den Untervektorraum $U:=\vectorspacespan\{\mathbf{w},\mathbf{w}^{\prime}\}$, dass $\dim(U)=2$.\\ Da $U\subseteq\reell^{2}$ Vektorräume sind und $\dim(U)=2=\dim(\reell^{2})$, folgt hieraus, dass $U=\reell^{2}$. ($\to$ Warum??)\\ Betrachte bspw. den Vektor \begin{mathe}[mc]{rcl} \eqtag[eq:1-2:1:\beweislabel] \mathbf{\xi} &:= &\mathbf{v}^{\prime}-\mathbf{v}\in\reell^{2}.\\ \end{mathe} Dann $\mathbf{\xi}\in U=\vectorspacespan\{\mathbf{w},\mathbf{w}^{\prime}\}$. Folglich existieren Skalare $\alpha,\beta\in\reell$, so dass $\alpha\mathbf{w}+\beta\mathbf{w}^{\prime}=\mathbf{\xi}$ gilt.\\ Setze nun \fbox{$t:=\alpha$} und \fbox{$s:=-\beta$}. Dann gilt \begin{mathe}[mc]{rclcl} \overbrace{% \mathbf{v}+t\mathbf{w} }^{\in L} &= &(\mathbf{v}+t\mathbf{w})-(\mathbf{v}^{\prime}+s\mathbf{w}^{\prime}) +\mathbf{v}^{\prime}+s\mathbf{w}^{\prime}\\ &= &(\mathbf{v}-\mathbf{v}^{\prime})+(t\mathbf{w}-s\mathbf{w}^{\prime}) +\mathbf{v}^{\prime}+s\mathbf{w}^{\prime}\\ &= &(\mathbf{v}-\mathbf{v}^{\prime})+(\alpha\mathbf{w}+\beta\mathbf{w}^{\prime}) +\mathbf{v}^{\prime}+s\mathbf{w}^{\prime}\\ &\eqcrefoverset{eq:1-2:1:\beweislabel}{=} &-\mathbf{\xi}+\mathbf{\xi} +\mathbf{v}^{\prime}+s\mathbf{w}^{\prime} &= &\underbrace{% \mathbf{v}^{\prime}+s\mathbf{w}^{\prime}% }_{\in L^{\prime}}.\\ \end{mathe} Darum gilt $L\cap L^{\prime}\neq\leer$, was ein Widerspruch ist.\\ Darum stimmt die o.\,s. Annahme nicht. Also sind $\mathbf{w},\mathbf{w}^{\prime}$ kolinear. \hinRichtung{2}{1} Angenommen, $\mathbf{w}=c\mathbf{w}^{\prime}$ für ein $c\in\reell$. \textbf{Zu zeigen:} $L\cap L^{\prime}=\leer$.\\ \fbox{Angenommen, dies sei nicht der Fall.} Dann existiert ein Vektor, $\mathbf{u}\in L\cap L^{\prime}$.\\ Per Konstruktion existieren dann $s_{0},t_{0}\in\reell$, so dass \begin{mathe}[mc]{rcccl} \mathbf{v}+t_{0}\mathbf{w} &= &\mathbf{u} &= &\mathbf{v}^{\prime}+s_{0}\mathbf{w}^{\prime}.\\ \end{mathe} Aus der Voraussetzung für diese Richtung folgt \begin{mathe}[mc]{rcl} \eqtag[eq:2-1:1:\beweislabel] \mathbf{v}^{\prime} &= &\mathbf{v}+(t_{0}-s_{0}c)\mathbf{w}\\ \end{mathe} Beachte, dass \fbox{$c\neq 0$}, denn sonst würde $\mathbf{w}=c\mathbf{w}^{\prime}=\zerovector$ gelten, was ein Widerspruch ist. Wir berechnen \begin{mathe}[mc]{rcl} \eqtag[eq:2-1:2:\beweislabel] L^{\prime} &= &\{\mathbf{v}^{\prime}+s\mathbf{w}^{\prime}\mid s\in\reell\}\\ &\eqcrefoverset{eq:2-1:1:\beweislabel}{=} &\{\mathbf{v}+(t_{0}-s_{0}c)\mathbf{w}+sc\mathbf{w}\mid s\in\reell\}\\ &= &\{\mathbf{v}+(t_{0}+(s-s_{0})c)\mathbf{w}\mid s\in\reell\}\\ &= &\{\mathbf{v}+t\mathbf{w}\mid t\in R\},\\ \end{mathe} wobei $R=\{t_{0}+(s-s_{0})c\mid s\in\reell\}=f(\reell)$. Also $R=f(\reell)$, wobei ${f:\reell\to\reell}$ eine durch ${f(s)=t_{0}+(s-s_{0})c}$ definierte Funktion ist. Da $c\neq 0$, ist es einfach zu sehen, dass $f$ surjektiv ist (in der Tat bijektiv). Darum gilt $R=f(\reell)=\reell$.\\ Aus \eqcref{eq:2-1:2:\beweislabel} folgt also ${L^{\prime}=\{\mathbf{v}+t\mathbf{w}\mid t\in\reell\}=L}$, was ein Widerspruch ist.\\ Darum stimmt die o.\,s. Annahme nicht. Also gilt $L\cap L^{\prime}=\leer$. \end{proof} %% AUFGABE 2-2b \item Wir zeigen nun ein minimales Beispiel dafür, dass \Cref{satz:main:ueb:2:ex:2a} im allgemeinen für andere Vektorräume nicht gilt. Betrachte den Vektorraum $\reell^{3}$. Betrachte die folgenden Vektoren in $\reell^{3}$: \begin{mathe}[mc]{rclqrclqrclqrcl} \mathbf{v} &= &\begin{svector} 0\\ 0\\ 0\\\end{svector}, &\mathbf{v}^{\prime} &= &\begin{svector} 1\\ 0\\ 0\\\end{svector}, &\mathbf{w} &= &\begin{svector} 0\\ 1\\ 0\\\end{svector}, &\mathbf{w}^{\prime} &= &\begin{svector} 0\\ 1\\ 1\\\end{svector}.\\ \end{mathe} Bis auf 2-Dimensionalität erfüllen diese die Voraussetzungen in \Cref{satz:main:ueb:2:ex:2a}. Einerseits wurden $\mathbf{w}$, $\mathbf{w}^{\prime}$ so gewählt, dass sie \emph{nicht} kolinear sind. Dennoch schneiden sich die beiden Geraden, $L$, $L^{\prime}$, nicht, da ${L\subseteq \{\mathbf{x}\in\reell^{3}\mid x_{1}=0\}=:E}$ und ${L^{\prime}\subseteq \{\mathbf{x}\in\reell^{3}\mid x_{1}=1\}=:E^{\prime}}$ und offensichtlich $E\cap E'=\leer$. \end{enumerate} %% AUFGABE 2-3 \clearpage \let\altsectionname\sectionname \def\sectionname{Aufgabe} \section[Aufgabe 3]{} \label{ueb:2:ex:3} \let\sectionname\altsectionname \begin{enumerate}{\bfseries (a)} %% AUFGABE 2-3a \item Für jedes $\gamma\in\reell$ sei die Gerade $L_{\gamma}\subseteq\reell^{2}$ gegeben durch \begin{mathe}[mc]{rcl} L_{\gamma} &= &\{(x,y)\in\reell^{2}\mid 2x+y=\gamma\cdot(x-3y-7)\}.\\ \end{mathe} \begin{schattierteboxdunn} \begin{satz} \makelabel{satz:main:ueb:2:ex:3a} Es gibt exakt einen Punkt in dem Schnitt aus den Geraden, $L_{\gamma}$, $\gamma\in\reell$. Es gilt nämlich ${\displaystyle\bigcap_{\gamma\in\reell}L_{\gamma}=\{\mathbf{\xi}\}}$, wobei $\mathbf{\xi}=(1,-2)$. \end{satz} \end{schattierteboxdunn} \begin{proof} Wir teilen diesen Beweis in zwei Teilen auf: \BeweisRichtung[$\supseteq$] Es reicht aus, für alle $\gamma\in\reell$ \textbf{zu zeigen}, dass $\mathbf{\xi}\in L_{\gamma}$.\\ Fixiere also ein beliebiges $\gamma\in\reell$. Dann \begin{mathe}[mc]{rclclcll} 2\xi_{1}+\xi_{2} &= &2\cdot 1+(-2) &= &0, &&&\text{und}\\ \gamma\cdot(\xi_{1}-3\xi_{2}-7) &= &\gamma\cdot(1-3(-2)-7) &= &\gamma\cdot 0 &= &0.\\ \end{mathe} Also ${2\xi_{1}+\xi_{2}=\gamma\cdot(\xi_{1}-3\xi_{2}-7)}$. Folglich gilt $\mathbf{\xi}\in L_{\gamma}$ per Konstruktion. \BeweisRichtung[$\subseteq$] Sei ${\mathbf{\eta}:=(x,y)\in\bigcap_{\gamma\in\reell}L_{\gamma}}$ beliebig. \textbf{Zu zeigen:} $\mathbf{\eta}=\mathbf{\xi}$.\\ Zu diesem Zwecke seien $\gamma_{1},\gamma_{2}\in\reell$ irgendwelche Werte mit $\gamma_{1}\neq\gamma_{2}$. Per Wahl gilt $\mathbf{\eta}\in L_{\gamma_{1}}\cap L_{\gamma_{2}}$. Also \begin{mathe}[mc]{rcl} 2x+y &= &\gamma_{1}\cdot(x-3x-7),\,\text{und}\\ 2x+y &= &\gamma_{2}\cdot(x-3x-7).\\ \end{mathe} Wir können ganz naiv arbeiten und die Gleichungen subtrahieren. Dies liefert $(\gamma_{1}-\gamma_{2})\cdot(x-3x-7)=0$, woraus sich ergibt, dass $x-3y-7=0$ gelten muss, da $\gamma_{1}\neq\gamma_{2}$. Eingesetzt in die erste Gleichung oben liefert $2x+y=\gamma\cdot 0=0$. Darum muss $\begin{svector} x\\ y\\\end{svector}$ das LGS $(A|\mathbf{b})$ lösen, wobei \begin{mathe}[mc]{rclqrcl} A &= &\begin{smatrix} 1 &-3\\ 2 &1\\ \end{smatrix}, &\mathbf{b} &= &\begin{svector} 7\\ 0\\\end{svector} \end{mathe} \begin{algorithm}[\rtab][\rtab] Gaußverfahren angewandt auf $(A|\mathbf{b})$: \begin{mathe}[mc]{c} \begin{matrix}{cc|c} 1 &-3 &7\\ 2 &1 &0\\ \end{matrix}\\ \end{mathe} Wende die Zeilentransformation ${Z_{2}\leftsquigarrow Z_{2}-2\cdot Z_{1}}$ an: \begin{mathe}[mc]{c} \begin{matrix}{cc|c} 1 &-3 &7\\ 0 &7 &-14\\ \end{matrix}\\ \end{mathe} Aus der Stufenform erschließt sich \begin{mathe}[bc]{rclcl} y &= &\frac{-14}{7} &= &-2\\ x &= &7 + 3\cdot y &= &1.\\ \end{mathe} \end{algorithm} Also ${\mathbf{\eta}=(x, y)=(1, -2)=\mathbf{\xi}}$ für alle $\mathbf{\eta}\in\bigcap_{\gamma\in\reell}L_{\gamma}$. Das heißt $\bigcap_{\gamma\in\reell}L_{\gamma}\subseteq\{\mathbf{\xi}\}$. \end{proof} \clearpage %% AUFGABE 2-3b \item \begin{enumerate}{\bfseries (i)} %% AUFGABE 2-3b-i \item Sei $\gamma\in\reell$. Dann gilt \begin{mathe}[mc]{rcl} (-3,2)\in L_{\gamma} &\Longleftrightarrow &2(-3)+(2)=\gamma\cdot((-3)-3(2)-7)\\ &\Longleftrightarrow &\gamma=\frac{-4}{-16}=\frac{1}{4}.\\ \end{mathe} Also ist \fbox{$\gamma=\frac{1}{4}$} der eindeutige Parameter, für den $(-3,2)\in L_{\gamma}$ gilt. %% AUFGABE 2-3b-ii \item Sei $\gamma\in\reell$. Man beobachte, dass \begin{longmathe}[mc]{RCL} L_{\gamma} &= &\{(x,y)\in\reell^{2}\mid (2-\gamma)x+(1+3\gamma)y=-7\gamma\}\\ &= &\begin{cases}[m]{lcl} \{(x,y)\in\reell^{2}\mid 0x + (1+3\cdot 2)y=-7\cdot 2\} &: &\gamma=2\\ \{(x,y)\in\reell^{2}\mid (2-\frac{-1}{3})x + 0y=-7\cdot\frac{-1}{3}\} &: &\gamma=-\frac{1}{3}\\ \{(x,y)\in\reell^{2}\mid (2-\gamma)x+(1+3\gamma)y=-7\gamma\} &: &\text{sonst} \end{cases}\\ &= &\begin{cases}[m]{lcl} \{(x,y)\in\reell^{2}\mid y=-2\} &: &\gamma=2\\ \{(x,y)\in\reell^{2}\mid x=1\} &: &\gamma=-\frac{1}{3}\\ \{(x,y)\in\reell^{2}\mid y=\frac{\gamma-2}{1+3\gamma}x - \frac{7\gamma}{1+3\gamma}\} &: &\text{sonst} \end{cases}.\\ \end{longmathe} Daraus folgt, dass $L_{\gamma}$ \begin{kompaktitem} \item parallel zur $x$-Achse für $\gamma=2$ ist, \item parallel zur $y$-Achse für $\gamma=-\frac{1}{3}$ ist, \item und ansonsten weder zur $x$- noch $y$-Achse parallel ist, da in diesem Falle $L_{\gamma}$ die Gerade »${y=ax+b}$« ist, wobei $a\neq 0$. \end{kompaktitem} Also ist der gesuchte Parameterwert eindeutig \fbox{$\gamma=-\frac{1}{3}$}. %% AUFGABE 2-3b-iii \item Die Gerade »$x-2y=-1$« lässt sich äquivalent als »$y=\frac{1}{2}x+1$ darstellen. Darum wird ein Wert $\gamma\in\reell$ gesucht, so dass die Gerade $L_{\gamma}$ weder zur $x$- noch $y$-Achse parallel ist, und die die $y$-$x$-Steigung $\frac{1}{2}$ hat. Nach der o.\,s. Berechnung in (ii) kommt dies nur für den 3. Fall in Frage. Darum gilt \begin{mathe}[mc]{rcl} L_{\gamma}\,\text{parallel zur Gerade »$x-2y=-1$«} &\Longleftrightarrow &\gamma\notin\{2,-\frac{1}{3}\} \,\text{und}\, \frac{\gamma-2}{1+3\gamma}=\frac{1}{2}\\ &\Longleftrightarrow &\gamma\notin\{2,-\frac{1}{3}\} \,\text{und}\, (\gamma-2)=\frac{1}{2}(1+3\gamma)\\ &\Longleftrightarrow &\gamma\notin\{2,-\frac{1}{3}\} \,\text{und}\, \gamma=-5\\ &\Longleftrightarrow &\gamma=-5.\\ \end{mathe} Also ist der gesuchte Parameterwert eindeutig \fbox{$\gamma=-5$}. \end{enumerate} \end{enumerate} %% ******************************************************************************** %% FILE: body/uebung/ueb3.tex %% ******************************************************************************** \setcounternach{chapter}{3} \chapter[Woche 3]{Woche 3} \label{ueb:2} \textbf{ACHTUNG.} Diese Lösungen dienen \emph{nicht} als Musterlösungen sondern eher als Referenz. Hier wird eingehender gearbeitet, als generell verlangt wird. Das Hauptziel hier ist, eine Variant anzubieten, gegen die man seine Versuche vergleichen kann. %% AUFGABE 3-1 \let\altsectionname\sectionname \def\sectionname{Aufgabe} \section[Aufgabe 1]{} \label{ueb:3:ex:1} \let\sectionname\altsectionname Wir arbeiten im Vektorraum $\reell^{3}$ und betrachten die Vektoren \begin{mathe}[mc]{rclqrclqrclqrcl} \mathbf{v}_{1} &= &\begin{svector} 1\\ 3\\ 1\\\end{svector} &\mathbf{v}_{2} &= &\begin{svector} -2\\ 5\\ -2\\\end{svector} &\mathbf{w}_{1} &= &\begin{svector} 4\\ -3\\ -3\\\end{svector} &\mathbf{w}_{2} &= &\begin{svector} 0\\ 1\\ 1\\\end{svector}\\ \end{mathe} \textbf{Zu berechnen:} $U:=\vectorspacespan\{\mathbf{v}_{1},\mathbf{v}_{2}\} \cap\vectorspacespan\{\mathbf{w}_{1},\mathbf{w}_{2}\}$ als Untervektorraum von $\reell^{3}$.\\ Zu diesem Zwecke betrachte einen beliebigen Vektor, $\mathbf{\xi}\in\reell^{3}$. Es gilt \begin{mathe}[mc]{rcl} \eqtag[eq:0:ueb:3:ex:1] \mathbf{\xi}\in U &\Longleftrightarrow &\exists{t_{1},t_{2},t_{3},t_{4}\in\reell:~} \mathbf{\xi}=t_{1}\mathbf{v}_{1}+t_{2}\mathbf{v}_{2} \,\text{und}\, \mathbf{\xi}=t_{3}\mathbf{w}_{1}+t_{4}\mathbf{w}_{2}\\ &\Longleftrightarrow &\exists{\mathbf{t}\in\reell^{4}:~} \mathbf{\xi}=t_{1}\mathbf{v}_{1}+t_{2}\mathbf{v}_{2} \,\text{und}\, t_{1}\mathbf{v}_{1}+t_{2}\mathbf{v}_{2} =t_{3}\mathbf{w}_{1}+t_{4}\mathbf{w}_{2}\\ &\Longleftrightarrow &\exists{\mathbf{t}\in\reell^{4}:~} \mathbf{\xi}=t_{1}\mathbf{v}_{1}+t_{2}\mathbf{v}_{2} \,\text{und}\, t_{1}\mathbf{v}_{1}+t_{2}\mathbf{v}_{2} -t_{3}\mathbf{w}_{1}-t_{4}\mathbf{w}_{2} =\zerovector\\ &\Longleftrightarrow &\exists{\mathbf{t}\in\reell^{4}:~} \mathbf{\xi}=t_{1}\mathbf{v}_{1}+t_{2}\mathbf{v}_{2} \,\text{und}\, t_{1}\mathbf{v}_{1}+t_{2}\mathbf{v}_{2} +t_{3}\mathbf{w}_{1}+t_{4}\mathbf{w}_{2} =\zerovector\\ &\Longleftrightarrow &\exists{\mathbf{t}\in\reell^{4}:~} \mathbf{\xi}=t_{1}\mathbf{v}_{1}+t_{2}\mathbf{v}_{2} \,\text{und}\, A\mathbf{t}=\zerovector,\\ \end{mathe} wobei \begin{mathe}[mc]{rcccl} A &:= &\left( \mathbf{v}_{1}~ \mathbf{v}_{2}~ \mathbf{w}_{1}~ \mathbf{w}_{2} \right) &= &\begin{smatrix} 1 &-2 &4 &0\\ 3 &5 &-3 &1\\ 1 &-2 &-3 &1\\ \end{smatrix}\\ \end{mathe} Darum ist es notwendig und hinreichend, die \emph{homogenen Lösungen} für $A$ zu finden, und daraus die Parameter abzulesen. \begin{algorithm}[\rtab][\rtab] Homogenes Problem für $A$:\\ Zeilentransformationen ${Z_{2}\leftsquigarrow Z_{2}-3\cdot Z_{1}}$, ${Z_{3}\leftsquigarrow Z_{3}-Z_{1}}$ anwenden: \begin{mathe}[mc]{c} \begin{smatrix} 1 &-2 &4 &0\\ 0 &11 &-15 &1\\ 0 &0 &-7 &1\\ \end{smatrix}\\ \end{mathe} Wende die Zeilentransformation ${Z_{2}\leftsquigarrow Z_{2}-Z_{3}}$ an: \begin{mathe}[mc]{c} \begin{smatrix} 1 &-2 &4 &0\\ 0 &11 &-8 &0\\ 0 &0 &-7 &1\\ \end{smatrix}\\ \end{mathe} Aus der Zeilenstufenform erschließt sich, dass $t_{4}$ frei ist. Also $t_{4}=\alpha$ für ein frei wählbares $\alpha\in\reell$. Aus der Stufenform von Gleichungen $3,2,1$ erschließt sich \begin{mathe}[mc]{rcl} t_{3} &= &\frac{1}{7}t_{4} = \frac{1}{7}\alpha\\ t_{2} &= &\frac{8}{11}t_{3} = \frac{8}{77}\alpha\\ t_{1} &= &2t_{2} - 4t_{3} = \frac{16}{77}\alpha - \frac{4}{7}\alpha = -\frac{28}{77}\alpha\\ \end{mathe} Man kann o.\,E. $\alpha$ durch $\beta:=-77\alpha$ ersetzen. Also ist die homogene Lösung gegeben durch \begin{mathe}[mc]{rcl} \mathbf{t} &= &\beta\begin{svector} 28\\ -8\\ -11\\ -77\\\end{svector}, \quad\text{mit $\beta\in\reell$ frei wählbar}. \end{mathe} \end{algorithm} Wir können nun \eqcref{eq:0:ueb:3:ex:1} fortsetzen und erhalten \begin{mathe}[mc]{rcl} \eqtag[eq:1:ueb:3:ex:1] \mathbf{\xi}\in U &\Longleftrightarrow &\exists{\mathbf{t}\in\reell^{4}:~} \mathbf{\xi}=t_{1}\mathbf{v}_{1}+t_{2}\mathbf{v}_{2} \,\text{und}\, A\mathbf{t}=\zerovector\\ &\Longleftrightarrow &\exists{\mathbf{t}\in\reell^{4}:~} \mathbf{\xi}=t_{1}\mathbf{v}_{1}+t_{2}\mathbf{v}_{2} \,\text{und}\, \exists{\beta\in\reell:~} \mathbf{t}=\beta\begin{svector} 28\\ -8\\ -11\\ -77\\\end{svector}\\ &\Longleftrightarrow &\exists{\beta\in\reell:~} \mathbf{\xi}=\beta\cdot( \underbrace{ 28\mathbf{v}_{1}+-8\mathbf{v}_{2} }_{=:\mathbf{u}} )\\ &\Longleftrightarrow &\mathbf{\xi}\in\vectorspacespan\{\mathbf{u}\}\\ \end{mathe} für alle $\mathbf{\xi}\in\reell^{3}$.\\ Es gilt \begin{mathe}[mc]{rcccccl} \mathbf{u} &= &28\begin{svector} 1\\ 3\\ 1\\\end{svector} -8\begin{svector} -2\\ 5\\ -2\\\end{svector} &= &\begin{svector} 44\\ 44\\ 44\\\end{svector} &= &44\begin{svector} 1\\ 1\\ 1\\\end{svector}.\\ \end{mathe} Aus \eqcref{eq:1:ueb:3:ex:1} ergibt sich der zu berechnende Untervektorraum als \begin{mathe}[mc]{rcccccccl} \vectorspacespan\{\mathbf{v}_{1},\mathbf{v}_{2}\} \cap\vectorspacespan\{\mathbf{w}_{1},\mathbf{w}_{2}\} &= &U &= &\vectorspacespan\{\mathbf{u}\} &= &\vectorspacespan\{44\begin{svector} 1\\ 1\\ 1\\\end{svector}\} &= &\vectorspacespan\{\begin{svector} 1\\ 1\\ 1\\\end{svector}\}.\\ \end{mathe} %% AUFGABE 3-2 \clearpage \let\altsectionname\sectionname \def\sectionname{Aufgabe} \section[Aufgabe 2]{} \label{ueb:3:ex:2} \let\sectionname\altsectionname Seien $X$, $Y$ nicht leere Mengen und ${f:X\to Y}$ eine Funktion. \begin{enumerate}{\bfseries (a)} %% AUFGABE 3-2a \item \begin{claim*} Die Aussage $\forall{A,B\subseteq X:~}f(A\cap B)=f(A)\cap f(B)$ ist \fbox{\uline{nicht} allgemein gültig}. \end{claim*} \begin{proof} Betrachte das Beispiel $X=\{0,1\}$, $Y=\{2\}$, und ${f:X\to Y}$ mit $f(x)=2$ für alle $x\in X$. Für $A=\{0\}$ und $B=\{1\}$ gilt $f(A\cap B)=f(\leer)=\leer$, während $f(A)\cap f(B)=\{2\}\cap\{2\}=\{2\}$. Also $f(A\cap B)\neq f(A)\cap f(B)$. Darum ist dies ein Gegenbeispiel zur Aussage. \end{proof} \text{Bemerkung.} Die Aussage ist eigentlich genau dann wahr, wenn $f$ injektiv ist. %% AUFGABE 3-2b \item \begin{claim*} Die Aussage $\forall{A,B\subseteq X:~}f(A\cup B)=f(A)\cup f(B)$ ist \fbox{allgemein gültig}. \end{claim*} Für manche (doppelte) Implikationen hier, nämlich für den Umgang mit Existenzquantoren, braucht man Grundkenntnisse in Prädikatenlogik 1. Stufe. Hierfüg gibt es zahlreiche Einführungswerke in die mathematische Logik, bspw. \cite{ebbinghaus2018}. \begin{proof} Seien $A,B\subseteq X$ beliebige Teilmengen. Es reicht aus \textbf{zu zeigen}, dass $y\in f(A\cup B)\Leftrightarrow y\in f(A)\cup f(B)$ für alle $y\in Y$ gilt.\\ Sei also $y\in Y$ beliebig. Es gilt \begin{longmathe}[mc]{RCL} y\in f(A\cup B) &\Longleftrightarrow &\exists{x\in A\cup B:~}y=f(x)\\ &\Longleftrightarrow &\exists{x\in X:~}x\in A\cup B\,\text{und}\,y=f(x)\\ &\Longleftrightarrow &\exists{x\in X:~} (x\in A\,\text{oder}\,x\in B) \,\text{und}\,y=f(x)\\ &\Longleftrightarrow &\exists{x\in X:~} \big( (x\in A\,\text{und}\,y=f(x)) \,\text{oder}\, (x\in B\,\text{und}\,y=f(x)) \big)\\ &\Longleftrightarrow &\exists{x\in X:~}(x\in A\,\text{und}\,y=f(x)) \,\text{oder}\, \exists{x\in X:~}(x\in B\,\text{und}\,y=f(x))\\ &\Longleftrightarrow &\exists{x\in A:~}y=f(x) \,\text{oder}\, \exists{x\in B:~}y=f(x)\\ &\Longleftrightarrow &y\in f(A)\,\text{oder}\,y\in f(B)\\ &\Longleftrightarrow &y\in f(A)\cup f(B).\\ \end{longmathe} Darum gilt $f(A\cup B)=f(A)\cup f(B)$ für alle $A,B\subseteq X$. \end{proof} %% AUFGABE 3-2c \item \begin{claim*} Die Aussage $\forall{A\subseteq X:~}f(X\ohne A)=Y\ohne f(A)$ ist \fbox{\uline{nicht} allgemein gültig}. \end{claim*} \begin{proof} Betrachte das Beispiel $X=\{0,1\}$, $Y=\{2\}$, und ${f:X\to Y}$ mit $f(x)=2$ für alle $x\in X$. Für $A=\{0\}$ gilt $f(X\ohne A)=f(\{1\})=\{2\}$, während $Y\cap f(A)=\{2\}\ohne\{2\}=\leer$. Also $f(X\ohne A)\neq Y\cap f(A)$. Darum ist dies ein Gegenbeispiel zur Aussage. \end{proof} \text{Bemerkung.} Die Aussage ist eigentlich genau dann wahr, wenn $f$ bijektiv ist. Und eine leicht modifizierte Aussage, $\forall{A\subseteq X:~}f(X\ohne A)=f(X)\cap f(A)$, ist genau dann wahr, wenn $f$ injektiv ist. %% AUFGABE 3-2d \item \begin{claim*} Die Aussage $\forall{A,B\subseteq Y:~}f^{-1}(A\cap B)=f^{-1}(A)\cap f^{-1}(B)$ ist \fbox{allgemein gültig}. \end{claim*} \begin{proof} Seien $A,B\subseteq Y$ beliebige Teilmengen. Es reicht aus \textbf{zu zeigen}, dass $x\in f^{-1}(A\cap B)\Leftrightarrow x\in f^{-1}(A)\cap f^{-1}(B)$ für alle $x\in X$ gilt.\\ Sei also $x\in X$ beliebig. Es gilt \begin{longmathe}[mc]{RCL} x\in f^{-1}(A\cap B) &\Longleftrightarrow &f(x)\in A\cap B\\ &\Longleftrightarrow &f(x)\in A\,\text{und}\,f(x)\in B\\ &\Longleftrightarrow &x\in f^{-1}(A)\,\text{und}\,x\in f^{-1}(B)\\ &\Longleftrightarrow &x\in f^{-1}(A)\cap f^{-1}(B).\\ \end{longmathe} Darum gilt $f^{-1}(A\cap B)=f^{-1}(A)\cap f^{-1}(B)$ für alle $A,B\subseteq Y$. \end{proof} %% AUFGABE 3-2e \item \begin{claim*} Die Aussage $\forall{A,B\subseteq Y:~}f^{-1}(A\cup B)=f^{-1}(A)\cup f^{-1}(B)$ ist \fbox{allgemein gültig}. \end{claim*} \begin{proof} Seien $A,B\subseteq Y$ beliebige Teilmengen. Es reicht aus \textbf{zu zeigen}, dass $x\in f^{-1}(A\cup B)\Leftrightarrow x\in f^{-1}(A)\cup f^{-1}(B)$ für alle $x\in X$ gilt.\\ Sei also $x\in X$ beliebig. Es gilt \begin{longmathe}[mc]{RCL} x\in f^{-1}(A\cup B) &\Longleftrightarrow &f(x)\in A\cup B\\ &\Longleftrightarrow &f(x)\in A\,\text{oder}\,f(x)\in B\\ &\Longleftrightarrow &x\in f^{-1}(A)\,\text{oder}\,x\in f^{-1}(B)\\ &\Longleftrightarrow &x\in f^{-1}(A)\cup f^{-1}(B).\\ \end{longmathe} Darum gilt $f^{-1}(A\cup B)=f^{-1}(A)\cup f^{-1}(B)$ für alle $A,B\subseteq Y$. \end{proof} \end{enumerate} %% AUFGABE 3-3 \clearpage \let\altsectionname\sectionname \def\sectionname{Aufgabe} \section[Aufgabe 3]{} \label{ueb:3:ex:3} \let\sectionname\altsectionname \begin{enumerate}{\bfseries (a)} %% AUFGABE 3-3a \item Seien $n\in\ntrlpos$ und $v\in\reell^{n}$. Sei ${f:\reell^{n}\to\reell^{n}}$ durch $f(x)=x+v$ definiert. \begin{claim*} $f$ ist \fbox{bijektiv}. \end{claim*} \begin{proof} Sei ${g:\reell^{n}\to\reell^{n}}$ durch $g(x)=x-v$ definiert. Es ist einfach zu sehen, dass $f\circ g=\id_{\reell^{n}}$ und $g\circ f=\id_{\reell^{n}}$. Per Definition ist als $f$ eine Bijektion mit Inversem $g$. \end{proof} %% AUFGABE 3-3b \item Seien $n\in\ntrlpos$ und $X=\reell^{n}\times(\reell^{n}\ohne\{\zerovector\})$. Sei $Y$ die Menge aller Geraden im $\reell^{n}$. Sei ${f:X\to Y}$ durch $f(v,w)=\{v+t\cdot w\mid t\in\reell\}$ definiert. \begin{claim*} $f$ ist \fbox{surjektiv} aber \fbox{nicht injektiv}. \end{claim*} \begin{proof} \uwave{{\bfseries Surjektivität}}\\ \textbf{Idee:} Folgt aus der Definition von Geraden durch Parameter.\\ Sei $L\subseteq\reell^{n}$ eine beliebige Gerade. \textbf{Zu zeigen:} $L\in f(X)$.\\ Nun, \emph{per Definition} einer Geraden existieren $u,v\in\reell^{n}$ mit $w\neq\zerovector$ und so dass $L=\{u+t\cdot w\mid t\in\reell\}$. Offensichtlicht gilt $(v,w)\in X$. Darum gilt $L=f((v,w))\in f(X)$. \uwave{{\bfseries Nichtinjektivität}}\\ \textbf{Idee:} Wir wissen, dass verschiedene aber parallele Vektoren dieselbe Gerade definieren.\\ Fixiere beliebiges $v,w\in\reell^{n}$ und wähle ein $c\in\reell\ohne\{0,1\}$.\\ Dann sind $w,cw\neq\zerovector$ verschiedene aber parallele Vektoren.\\ Darum gilt $f((v,w))=\{v+t\cdot w\mid t\in\reell\}=\{v+tc\cdot w\mid t\in\reell\}=f((v,cw))$.\\ Da $(v,w)\neq(v,cw)$, ist $f$ somit nicht injektiv. \end{proof} %% AUFGABE 3-3c \item Es sei $X$ die Menge aller Bücher in einem fixierten Kontext. Sei $Y$ die Menge alle Autor(inn)en von Büchern. Sei ${f:X\to\Pot(Y)}$ definiert durch $f(x)=\{y\mid \text{$y$ ein(e) Autor(in) vom Buch $x$}\}$ für alle $x\in X$. \begin{claim*} $f$ ist \fbox{nicht im Allgemeinen injektiv} und \fbox{niemals surjektiv}. \end{claim*} \begin{proof} \uwave{{\bfseries Nichtsurjektivität}}\\ \textbf{Zu zeigen:} Es gibt konstellationen von Autor(inn)en, die kein gemeinsames Buch verfasst haben.\\ Es gibt \emph{immer} eine(n) Autor(in) eines Buchs, sodass $\leer\notin f(X)$ in allen Kontexten. Darum ist $f$ niemals surjektiv. \uwave{{\bfseries Nichtinjektivität}}\\ \textbf{Zu zeigen:} Es gibt zwei verschiedene Bücher, die von der gleichen Konstellation an Autor(inn)en verfasst wurden. In unserem Kontext hat bspw. $a=\text{{\itshape JK~Rowling}}$ alleine die Bücher ${b_{1}:=\text{{\itshape »HP~and~the~Philosopher's~Stone«}}}$ und ${b_{2}:=\text{{\itshape »HP~and~the~Goblet~of~Fire«}}}$ geschrieben. Darum $b_{1}\neq b_{2}$ und $f(b_{1})=\{a\}=f(b_{2})$. Also ist $f$ in unserem Kontext nicht injektiv. \end{proof} \textbf{Anmerkung.} Falls wir $\leer$ von der Bildmenge $\Pot(Y)$ exludieren, dann können wir mindestens dafür argumentieren, dass $f$ \fbox{nicht im Allgemeinen surjektiv} ist: In unserem konkreten Kontext haben bspw. {\itshape JK~Rowling} und {\itshape Oscar~Wilde} nie am selben Buch gearbeitet, also gilt $\{\text{JK Rowling},\,\text{Oscar Wilde}\}\notin f(X)$. In der Tat ist ein Kontext kaum vorstellbar, in dem sich \emph{alle} Autor(inn)en an einem gemeinsamen Buch beteiligt haben, d.\,h. $Y\in f(X)$ sowie alle „große“ Teilmengen sind fast immer ausgeschlossen. %% AUFGABE 3-3d \item Seien $X$ die Menge aller in Deutschland zugelassener Kfz und $Y$ die Menge aller amtlicher Kennzeichen. Sei ${f:X\to Y}$ die Abbildung, die jedem Kfz sein Kennzeichen zuordnet. \begin{claim*} $f$ ist \fbox{injektiv} aber \fbox{nicht im Allgemeinen surjektiv}. \end{claim*} \begin{proof} \uwave{{\bfseries Injektivität:}} Jedes Kennzeichen darf per Gesetz nur einem Kfz zugehören. \uwave{{\bfseries Nichtsurjektivität:}} Es besteht zwar die Chance, dass irgendwann alle Kennzeichen aufgebraucht werden, aber in der Praxis ist die Menge $Y$ sehr groß, dass dies aktuell und für eine lange Zeit nicht vorkommt. \end{proof} \end{enumerate} %% ******************************************************************************** %% FILE: body/uebung/ueb4.tex %% ******************************************************************************** \setcounternach{chapter}{4} \chapter[Woche 4]{Woche 4} \label{ueb:4} %% AUFGABE 4-1 \let\altsectionname\sectionname \def\sectionname{Aufgabe} \section[Aufgabe 1]{} \label{ueb:4:ex:1} \let\sectionname\altsectionname \begin{enumerate}{\bfseries (a)} %% AUFGABE 4-1a \item Betrachte die Menge $X:=\intgr\times\ntrlpos$ und die binäre Relation, $\sim\subseteq X\times X$, die durch \begin{mathe}[mc]{rcl} (a,b)\sim (a',b') &\Longleftrightarrow &ab'=a'b \end{mathe} für $(a,b),(a',b')\in X$ definiert wird. \begin{claim*} $(X,\sim)$ ist eine Äquivalenzrelation. \end{claim*} \begin{proof} Wir gehen die Axiome durch: \begin{kompaktenum}[\rtab][\rtab] \item[\uwave{{\itshape Reflexivität:}}] Sei $(a,b)\in X$ beliebig.\\ \textbf{Zu zeigen:} $(a,b)\sim(a,b)$.\\ Offensichtlich gilt $ab=ab$.\\ Per Konstruktion gilt also $(a,b)\sim(a,b)$. \item[\uwave{{\itshape Symmetrie:}}] Seien $(a,b),(a',b')\in X$ beliebig.\\ \textbf{Zu zeigen:} ${(a,b)\sim(a',b')\Rightarrow(a',b')\sim(a,b)}$.\\ Es gilt \begin{mathe}[mc]{rclql} (a,b)\sim (a',b') &\Longleftrightarrow &ab'=a'b &\text{(per Konstruktion)}\\ &\Longrightarrow &a'b=ab'\\ &\Longleftrightarrow &(a',b')\sim(a,b) &\text{(per Konstruktion).}\\ \end{mathe} \item[\uwave{{\itshape Transitivität:}}] Seien $(a,b),(a',b'),(a'',b'')\in X$ beliebig.\\ \textbf{Zu zeigen:} $(a,b)\sim(a',b')$ und $(a',b')\sim(a'',b'')$ $\Rightarrow$ $(a,b)\sim(a'',b'')$.\\ Es gilt \begin{mathe}[mc]{rcl} \begin{array}[b]{0l0} (a,b)\sim (a',b')\\ \,\text{und}\,(a',b')\sim(a'',b'')\\ \end{array} &\Longleftrightarrow &ab'=a'b\,\text{und}\,a'b''=a''b'\\ &&\quad\text{(per Konstruktion)}\\ &\Longrightarrow &(ab'')b'=(ab')b''=(a'b)b''=(a'b'')b=(a''b')b=(a''b)b'\\ &\Longrightarrow &ab''=a''b,\\ &&\quad\text{da $b'\neq 0$}\\ &\Longleftrightarrow &(a,b)\sim(a'',b'')\\ &&\quad\text{(per Konstruktion).}\\ \end{mathe} \end{kompaktenum} Darum erfüllt $(X,\sim)$ die Axiome einer Äquivalenzrelation. \end{proof} \textbf{Bemerkung.} Man kann zeigen, dass ${f:X/\lsim\to\rtnl}$ definiert durch $f([(a,b)])=a/b$ wohldefiniert und bijektiv ist. In der Tat realisieren manche Werke die rationalen Zahlen, $\rtnl$, als genau diesen Quotientenraum, d.\,h. man kann die Äquivalenzklassen hier als rationale Zahlen deuten. %% AUFGABE 4-1b \item Betrachte die Menge $X:=\intgr\times\intgr$ und die binäre Relation, $\leq\subseteq X\times X$, die durch \begin{mathe}[mc]{rcl} (a,b)\leq(a',b') &\Longleftrightarrow &a\leq a'\,\text{und}\,b\leq b'\\ \end{mathe} für $(a,b),(a',b')\in X$ definiert wird. \begin{claim*} $(X,\leq)$ ist eine partielle Ordnung aber \fbox{nicht total}. \end{claim*} \begin{proof} Wir gehen die Axiome durch: \begin{kompaktenum}[\rtab][\rtab] \item[\uwave{{\itshape Reflexivität:}}] Sei $(a,b)\in X$ beliebig.\\ \textbf{Zu zeigen:} $(a,b)\leq(a,b)$.\\ Offensichtlich gilt $a\leq a$ und $b\leq b$.\\ Per Konstruktion gilt also $(a,b)\leq(a,b)$. \item[\uwave{{\itshape Antisymmetrie:}}] Seien $(a,b),(a',b')\in X$ beliebig.\\ \textbf{Zu zeigen:} $(a,b)\leq(a',b')$ und $(a',b')\leq(a,b)$ $\Rightarrow$ $(a,b)=(a',b')$.\\ Es gilt \begin{mathe}[mc]{rcl} \begin{array}[b]{0l0} (a,b)\leq (a',b')\\ \,\text{und}\,(a',b')\leq(a,b)\\ \end{array} &\Longleftrightarrow &a\leq a'\,\text{und}\,b\leq b' \text{und}\, a'\leq a\,\text{und}\,b'\leq b\\ &&\text{(per Konstruktion)}\\ &\Longrightarrow &a=a\,\text{und}\,b=b',\\ &&\text{da $(\intgr,\leq)$ antisymmetrisch ist}\\ &\Longleftrightarrow &(a,b)=(a',b').\\ \end{mathe} \item[\uwave{{\itshape Transitivität:}}] Seien $(a,b),(a',b'),(a'',b'')\in X$ beliebig.\\ \textbf{Zu zeigen:} $(a,b)\leq(a',b')$ und $(a',b')\leq(a'',b'')$ $\Rightarrow$ $(a,b)\leq(a'',b'')$.\\ Es gilt \begin{mathe}[mc]{rcl} \begin{array}[b]{0l0} (a,b)\leq (a',b')\\ \,\text{und}\,(a',b')\leq(a'',b'')\\ \end{array} &\Longleftrightarrow &a\leq a'\,\text{und}\,b\leq b' \text{und}\, a'\leq a''\,\text{und}\,b'\leq b''\\ &&\text{(per Konstruktion)}\\ &\Longrightarrow &a\leq a''\,\text{und}\,b\leq b'',\\ &&\text{da $(\intgr,\leq)$ transitiv ist}\\ &\Longleftrightarrow &(a,b)\leq(a'',b'')\\ &&\text{(per Konstruktion).}\\ \end{mathe} \end{kompaktenum} Darum erfüllt $(X,\leq)$ die einer partiellen Ordnung.\\ Zum Schluss, beachte, dass $(0,1)$ und $(1,0)$ bzgl. $\leq$ unvergleichbar sind. Darum ist $(X,\leq)$ nicht total. \end{proof} \end{enumerate} %% AUFGABE 4-2 \clearpage \let\altsectionname\sectionname \def\sectionname{Aufgabe} \section[Aufgabe 2]{} \label{ueb:4:ex:2} \let\sectionname\altsectionname Fixiere $n\in\ntrlpos$. Wir definieren die binäre Relation ${\sim\subseteq\intgr\times\intgr}$ mittels \begin{mathe}[mc]{rcl} a \sim b &:\Longleftrightarrow &\modfn(a,n)=\modfn(b,n)\\ \end{mathe} für $a,b\in\intgr$. \begin{enumerate}{\bfseries (a)} %% AUFGABE 4-2a \item \begin{claim*} $(\intgr,\sim)$ ist eine Äquivalenzrelation. \end{claim*} \begin{proof} Wir gehen die Axiome durch: \begin{kompaktenum}[\rtab][\rtab] \item[\uwave{{\itshape Reflexivität:}}] Sei $a\in \intgr$ beliebig. \textbf{Zu zeigen:} $a\sim a$.\\ Offensichtlich gilt $\modfn(a,n)=\modfn(a,n)$.\\ Per Konstruktion gilt also $a\sim a$. \item[\uwave{{\itshape Symmetrie:}}] Seien $a, a'\in \intgr$ beliebig. \textbf{Zu zeigen:} ${a\sim a'\Rightarrow a'\sim a}$.\\ Es gilt \begin{mathe}[mc]{rclql} a\sim a' &\Longleftrightarrow &\modfn(a,n)=\modfn(a',n) &\text{(per Konstruktion)}\\ &\Longrightarrow &\modfn(a',n)=\modfn(a,n)\\ &\Longleftrightarrow & a'\sim a &\text{(per Konstruktion).}\\ \end{mathe} \item[\uwave{{\itshape Transitivität:}}] Seien $ a, a', a''\in \intgr$ beliebig. \textbf{Zu zeigen:} $a\sim a'$ und $a'\sim a''$ $\Rightarrow$ $a\sim a''$.\\ Es gilt \begin{mathe}[mc]{rclql} a\sim a'\,\text{und}\, a'\sim a'' &\Longleftrightarrow &\modfn(a,n)=\modfn(a',n) \,\text{und}\, \modfn(a',n)=\modfn(a'',n)\\ &&\text{(per Konstruktion)}\\ &\Longrightarrow &\modfn(a,n)=\modfn(a'',n)\\ &\Longleftrightarrow &a\sim a'' \quad\text{(per Konstruktion).}\\ \end{mathe} \end{kompaktenum} Darum erfüllt $(\intgr,\sim)$ die Axiome einer Äquivalenzrelation. \end{proof} \textbf{Bemerkung.} Es gibt einen einfacheren Ansatz. Zunächst beweist man das allgemeine Lemma: Für jede Äquivalenzrelation $(Y,\approx)$ und jede Relation $(X,R)$, falls eine Funktion ${f:X\to Y}$ existiert, so dass ${\forall{x,x'\in X:~}(x,x')\in R\Leftrightarrow f(x)\approx f(x')}$, so gilt dass $(X,R)$ eine Äquivalenzrelation ist. Und jetzt wendet man dies auf unseren Kontext an: Wir die Äquivalenzrelation $(\{0,1,2\ldots,n-1\},=)$ und die Relation $(\intgr,\sim)$ und eine Abbildung ${f:a\in\intgr\mapsto\modfn(a,n)}$, für die ${\forall{a,a'\in\intgr:~}(a,a')\in\sim\Leftrightarrow f(a)\approx f(a')}$ \emph{per Konstruktion} gilt. Darum ist $(\intgr,\sim)$ eine Äquivalenzrelation. %% AUFGABE 4-2b \item \begin{claim*} Es gibt $n$ Äquivalenzklassen. \end{claim*} \begin{proof} Betrachte die Abbildung \begin{mathe}[mc]{rcccl} \rho &: &\intgr/\lsim &\to &\{0,1,\ldots,n-1\}\\ &: &[a] &\mapsto &\modfn(a,n)\\ \end{mathe} Es reicht aus \textbf{zu zeigen}, dass $\rho$ eine wohldefinierte Bijektion ist. \begin{kompaktenum}[\rtab][\rtab] \item[\uwave{{\itshape Wohldefiniertheit:}}] Sei $C\in\intgr/\sim$ beliebig. Seien $a,a'\in\intgr$ mit $[a]=C$ und $[a']=C$.\\ \textbf{Zu zeigen:} $\modfn(a,n)=\modfn(a',n)$.\\ Aus $[a]=C=[a']$ folgt $a\sim a'$ und damit per Konstruktion $\modfn(a,n)=\modfn(a',n)$. Darum ordnet $\rho$ einen eindeutig Wert $[a]$ zu. \item[\uwave{{\itshape Injektivität:}}] Seien $C,C'\in\intgr/\lsim$ beliebig. \textbf{Zu zeigen:} ${\rho(C)=\rho(C')\Rightarrow C=C'}$.\\ Wähle zunächst $a,a'\in\intgr$, so dass $C=[a]$ und $C=[a']$. Dann gilt \begin{mathe}[mc]{rcl} \rho(C)=\rho(C') &\Longrightarrow &\modfn(a,n)=\modfn(a',n)\\ &\Longrightarrow &a\sim a'\\ &\Longrightarrow &C=[a]=[a']=C'.\\ \end{mathe} \item[\uwave{{\itshape Surjektivität:}}] Sei $k\in\{0,1,\ldots,n-1\}$ beliebig. \textbf{Zu zeigen:} $k\in\rho(\intgr/\lsim)$.\\ Setze $C=[k]\in\intgr/\lsim$. Dann $\rho(C)=\modfn(k,n)=k$.\footnote{ Seien $q\in\intgr$ und $r\in\{0,1,\ldots,n-1\}$ mit $qn+r=k$. Da $k,r\in\{0,1,\ldots,n-1\}$, gilt $qn=k-r\in\intgr\cap(-n,n)$. Das heißt, $k-r$ ist eine durch $n$ teilbare ganze Zahl in $(-n,n)$. Die einzige solche Zahl ist $0$. Also $k-r=0$. Also $\modfn(k,n)=r=k$. } Also gilt $k\in\rho(\intgr/\lsim)$. \end{kompaktenum} Darum ist $\rho$ eine Bijektion. Also gilt $|\intgr/\lsim|=|\{0,1,\ldots,n-1\}|=n$. \end{proof} %% AUFGABE 4-2c \item Laut der Berechnung in Aufgabe 2(b) gilt ${\intgr/\lsim=\{[0],[1],\ldots,[n-1]\}}$. Für jedes ${k\in\{0,1,\ldots,n-1\}}$ lässt sich die Äquivalenzklasse $[k]$ wie folgt als Teilmenge beschreiben \begin{mathe}[bc]{rcl} [k] &= &\{a\in\intgr \mid a\sim k\} \,\text{per Definition}\\ &= &\{a\in\intgr \mid \modfn(a,n)=\modfn(k,n)\}\\ &= &\{a\in\intgr \mid \modfn(a,n)=k\}\\ &= &\{a\in\intgr \mid \exists{q\in\intgr:~}a=qn+k\}\\ &= &\{qn+k \mid q\in\intgr\}\\ &= &\intgr\cdot n + k.\\ \end{mathe} Also lassen sich die Äquivalenzklassen durch die Teilmengen ${\{\intgr\cdot n+k\mid k\in\{0,1,\ldots,n-1\}\}}$ darstellen. \end{enumerate} %% AUFGABE 4-3 \let\altsectionname\sectionname \def\sectionname{Aufgabe} \section[Aufgabe 3]{} \label{ueb:4:ex:3} \let\sectionname\altsectionname \begin{claim*} \makelabel{claim:main:ueb:4:ex:3} Für $n\in\ntrlpos$ bezeichne mit $\Phi(n)$ die Aussage, dass für alle Mengen $X$, $Y$ mit $|X|=|Y|=n$ \begin{mathe}[mc]{rcl} \eqtag[eq:1:\beweislabel] |\{f\mid f\,\text{eine Bijektion zw. $X$ und $Y$}\}| &= &n!.\\ \end{mathe} Dann gilt $\forall{n\in\ntrlpos:~}\Phi(n)$. \end{claim*} \begin{proof}[Ansatz I][Ansatz I] Sei $n\in\ntrlpos$ und seien $X$, $Y$ $n$-elementige Mengen. Sei $(x_{1},x_{2},\ldots,x_{n})$ eine Auflistung der Elemente in $X$. Um eine Injektion zw. $X$ und $Y$ zu definieren, wählt man zuerst ein Element $y_{1}\in Y$ für $x_{1}$ (dafür gibt es $n$ Möglichkeiten), dann ein Element $y_{2}\in Y$ for $x_{2}$ (dafür bleiben $n-1$ Möglichkeiten übrig), usw. Darum gibt es insgesamt $n\cdot (n-1)\cdot\cdots 1=n!$ Injektionen zwischen $X$ und $Y$. Da $X$ und $Y$ endlich und gleichmächtig sind, ist jede Injektion zwischen diesen Mengen automatisch surjektiv und damit bijektiv. Darum gibt es $n!$ Bijektionen zwischen $X$ und $Y$. \end{proof} \begin{proof}[Ansatz II][Ansatz II] Wir beweisen die Behauptung per Induktion über $n$. \begin{kompaktenum}[\rtab][\rtab] \item[\uwave{{\bfseries Induktionsanfang:}}] Sei $n=1$. Für $1$-elementigen Mengen $X$, $Y$, gibt es offensichtlich exakt eine Funktion zwischen $X$ und $Y$, und dies ist eine Bijektion. Darum gilt \eqcref{eq:1:\beweislabel}. \item[\uwave{{\bfseries Induktionsvoraussetzung:}}] Sei $n>1$. Angenommen, $\Phi(n-1)$ gilt. \item[\uwave{{\bfseries Induktionsschritt:}}] Seien $X$, $Y$ beliebige $n$-elementige Mengen. \textbf{Zu zeigen:} \eqcref{eq:1:\beweislabel} gilt.\\ Fixiere $x_{0}\in X$. Beobachte, dass für alle $y_{0}\in Y$ die Mengen $X':=X\ohne\{x_{0}\}$ und $Y':=Y\ohne\{y_{0}\}$ beide $n-1$-elementig sind. Betrachte nun die Abbildung \begin{mathe}[mc]{rcccl} F &: &\{g\mid g\,\text{Bij. zw. $X\ohne\{x_{0}\}$ und $Y\ohne\{x_{0}\}$}\} &\to &\{f\mid f\,\text{Bij. zw. $X$ und $Y$},\,f(x_{0})=y_{0}\}\\ &: &g &\mapsto &g\cup\{(x_{0},y_{0})\}.\\ \end{mathe} Das heißt, jede Bijektion ${g:X\ohne\{x_{0}\}\to Y\ohne\{y_{0}\}}$ wird durch $F$ zu einer Funktion von $X$ nach $Y$ fortgesetzt, indem das zusätzliche Element, $x_{0}$, auf $y_{0}$ abgebildet wird. Es ist einfach zu sehen, dass $F$ wohldefiniert ist, d.\,h. für jede Bijektion ${g:X\ohne\{x_{0}\}\to Y\ohne\{y_{0}\}}$, es gilt, dass $F(g)$ eine wohldefinierte Funktion zwischen $X$ und $Y$ ist und weiterhin ist dies eine Bijektion. Außerdem ist es klar, dass die Abbildung \begin{mathe}[mc]{rcccl} G &: &\{f\mid f\,\text{Bij. zw. $X$ und $Y$},\,f(x_{0})=y_{0}\} &\to &\{g\mid g\,\text{Bij. zw. $X\ohne\{x_{0}\}$ und $Y\ohne\{x_{0}\}$}\}\\ &: &f &\mapsto &f\restr{X\ohne\{x_{0}\}\times Y\ohne\{x_{0}\}}\\ \end{mathe} die Abbildung $F$ nach rechts und links invertiert. Also ist $G$ eine Bijektion. Daraus folgt per Definition von Kardinalität \begin{mathe}[mc]{rcl} \eqtag[eq:2:\beweislabel] |\{f\mid f\,\text{Bij. zw. $X$ und $Y$},\,f(x_{0})=y_{0}\}| &= &|\{g\mid g\,\text{Bij. zw. $X\ohne\{x_{0}\}$ und $Y\ohne\{x_{0}\}$}\}|\\ &= &(n-1)!\,\text{laut IV}.\\ \end{mathe} Anderseits ist ${(\{f\mid f\,\text{Bij. zw. $X$ und $Y$},\,f(x_{0})=y_{0}\})_{y_{0}\in Y}}$ eine Partition von ${\{f\mid f\,\text{Bij. zw. $X$ und $Y$}\}}$. Darum gilt \begin{mathe}[mc]{rcl} |\{f\mid f\,\text{Bij. zw. $X$ und $Y$}\}| &= &|\bigcup_{y_{0}\in Y}\{f\mid f\,\text{Bij. zw. $X$ und $Y$},\,f(x_{0})=y_{0}\}|\\ &= &\sum_{y_{0}\in Y}|\{f\mid f\,\text{Bij. zw. $X$ und $Y$},\,f(x_{0})=y_{0}\}|\\ &&\text{wegen paarweise Disjunktheit}\\ &\eqcrefoverset{eq:2:\beweislabel}{=} &\sum_{y_{0}\in Y}(n-1)! = |Y|\cdot (n-1)! = n\cdot (n-1)! = n!.\\ \end{mathe} Also gilt \eqcref{eq:1:\beweislabel}. \end{kompaktenum} Darum gilt $\Phi(n)$ per Induktion für alle $n\in\ntrl$. \end{proof} %% ******************************************************************************** %% FILE: body/uebung/ueb5.tex %% ******************************************************************************** \setcounternach{chapter}{5} \chapter[Woche 5]{Woche 5} \label{ueb:5} \textbf{ACHTUNG.} Diese Lösungen dienen \emph{nicht} als Musterlösungen sondern eher als Referenz. Hier wird eingehender gearbeitet, als generell verlangt wird. Das Hauptziel hier ist, eine Variant anzubieten, gegen die man seine Versuche vergleichen kann. %% AUFGABE 5-1 \let\altsectionname\sectionname \def\sectionname{Aufgabe} \section[Aufgabe 1]{} \label{ueb:5:ex:1} \let\sectionname\altsectionname Fixiere eine natürliche Zahl $n\in\ntrlzero$. Sei $a_{i}\in\{0,1,\ldots,10-1\}$ die eindeutige Zahlen, so dass \begin{mathe}[mc]{rcl} n &= &\sum_{i\in\ntrlzero}a_{i}10^{i}\\ \end{mathe} gilt. \begin{enumerate}{\bfseries (a)} %% AUFGABE 5-1a \item \begin{claim*} $3\divides n$ $\Leftrightarrow$ $3\mid\sum_{i\in\ntrlzero}a_{i}$. \end{claim*} \begin{proof} Beachte zunächst, dass $10\equiv 1\mod 3$. Also gilt modulo $3$ \begin{mathe}[mc]{rcccl} n &\equiv &\sum_{i\in\ntrlzero}a_{i}1^{i} &\equiv &\sum_{i\in\ntrlzero}a_{i}.\\ \end{mathe} Folglich gilt \begin{mathe}[mc]{rcccccl} 3\mid n &\Longleftrightarrow &n\equiv 0\mod 3 &\Longleftrightarrow &\sum_{i\in\ntrlzero}a_{i}\equiv 0\mod 3 &\Longleftrightarrow &3\mid\sum_{i\in\ntrlzero}a_{i} \end{mathe} wie behauptet. \end{proof} %% AUFGABE 5-1b \item \begin{claim*} $11\divides n$ $\Leftrightarrow$ $1\mid\sum_{i\in\ntrlzero}(-1)^{i}a_{i}$. \end{claim*} \begin{proof} Beachte zunächst, dass $10=-1\mod 11$. Also gilt modulo $11$ \begin{mathe}[mc]{rcccl} n &\equiv &\sum_{i\in\ntrlzero}a_{i}(-1)^{i}.\\ \end{mathe} Folglich gilt \begin{mathe}[mc]{rcccccl} 11\mid n &\Longleftrightarrow &n\equiv 0\mod 11 &\Longleftrightarrow &\sum_{i\in\ntrlzero}a_{i}\equiv 0\mod 11 &\Longleftrightarrow &11\mid\sum_{i\in\ntrlzero}(-1)^{i}a_{i} \end{mathe} wie behauptet. \end{proof} \end{enumerate} %% AUFGABE 5-2 \let\altsectionname\sectionname \def\sectionname{Aufgabe} \section[Aufgabe 2]{} \label{ueb:5:ex:2} \let\sectionname\altsectionname \begin{enumerate}{\bfseries (a)} %% AUFGABE 5-2a \item Seien $a=142$ und $b=84$. Wir berechnen $\ggT(a,b)$ mittels des Euklidischen Algorithmus (siehe \cite[Satz 3.4.7]{sinn2020}). \begin{longtable}[mc]{|c|c|} \hline \hline Restberechnung (symbolisch) &Restberechnung (Werte)\\ \hline \endhead $a = b\cdot q_{1} + r_{1}$ &$142 = 84\cdot 1 + 58$\\ $b = r_{1}\cdot q_{2} + r_{2}$ &$84 = 58\cdot 1 + 26$\\ $r_{1} = r_{2}\cdot q_{3} + r_{3}$ &$58 = 26\cdot 2 + 6$\\ $r_{2} = r_{3}\cdot q_{4} + r_{4}$ &$26 = 6\cdot 4 + \boxed{\mathbf{2}}$\\ $r_{3} = r_{4}\cdot q_{5} + r_{5}$ &$6 = 2\cdot 3 + 0$\\ \hline \hline \end{longtable} Darum gilt $\ggT(a,b)=r_{2}=2$. %% AUFGABE 5-2b \item \begin{claim} \makelabel{claim:main:ueb:5:ex:2b} Seien $a,b,c\in\intgr$ mit $a,b\neq 0$. Die folgenden Aussagen sind äquivalent: \begin{kompaktenum}{\bfseries (i)}[\rtab][\rtab] \item\punktlabel{1} $\exists{x,y\in\intgr:~}ax+by=c$ \item\punktlabel{2} $\ggT(a,b)\divides c$ \end{kompaktenum} \end{claim} \begin{proof} Fixiere zunächst $d:=\ggT(a,b)$. Da $a,b\in\intgr\ohne\{0\}$, ist $d\in\ntrl$ eine wohldefinierte positive Zahl. \hinRichtung{1}{2} Angenommen, $ax+by=c$ für ein $x,y\in\intgr$.\\ Da $x,y\in\intgr$, erhalten wir $c=ax+by\equiv 0x+0z\equiv 0$ modulo $d$.\\ Also $\ggT(a,b)=d\divides c$. \hinRichtung{2}{1} Angenommen, $\ggT(a,b)\divides c$.\\ Dann existiert ein $k\in\intgr$, so dass $c=k\cdot\ggT(a,b)$.\\ Laut des Lemmas von B\'ezout (siehe \cite[Lemma 3.4.8]{sinn2020}) existiere nun $u,v\in\intgr$, so dass $\ggT(a,b)=au+bv$.\\ Daraus folgt ${c=k\cdot\ggT(a,b)=aku+bkv}$.\\ Da $ku,kv\in\intgr$, haben wir \punktcref{1} bewiesen. \end{proof} \end{enumerate} %% AUFGABE 5-3 \let\altsectionname\sectionname \def\sectionname{Aufgabe} \section[Aufgabe 3]{} \label{ueb:5:ex:3} \let\sectionname\altsectionname \begin{enumerate}{\bfseries (a)} %% AUFGABE 5-3a \item Sei $H:=\intgr/2\intgr$ die (abelsche) Gruppe von Restklassen modulo $2$ unter Addition.\\ Sei $G:=H\times H$ mit Neutralelement $e=([0],[0])$ und versehen mit der Produktstruktur.\\ Als Produkt von (abelschen) Gruppen ist $G$ automatisch eine (abelsche) Gruppe. Und offensichtlich hat $G$ genau $|G|=|H\times H|=|H|\cdot|H|=2\cdot 2=4$ Elemente.\\ Es bleibt \textbf{zu zeigen}, dass $\forall{a\in G:~}a\ast a=e$.\\ Sei also $a=([k],[j])\in H\times H=G$ ein beliebiges Element. Es gilt \begin{mathe}[mc]{rcl} a\ast a &= &([k],[j])\ast([k],[j])\\ &= &([k]+[k],[j]+[j])\\ &= &([k+k],[j+j])\\ &= &([2k],[2j]) =([0],[0]) =e, \end{mathe} da $2\equiv 0\mod 2$.\\ Also ist unsere Konstruktion von $G$ ein passendes Beispiel. %% AUFGABE 5-3b \item \begin{claim*} Sei $(G,\ast,e)$ eine Gruppe. Angenommen, $\forall{a\in G:~}a\ast a=e$. Dann ist $G$ kommutativ. \end{claim*} \begin{proof} Beachte zunächst, dass wegen Eindeutigkeit des Inverses die Annahme zu \begin{mathe}[mc]{c} \eqtag[eq:1:ueb:5:ex:3] \forall{a\in G:~}a^{-1}=a \end{mathe} äquivalent ist.\\ \textbf{Zu zeigen:} Für alle $a,b\in G$ gilt $a\ast b=b\ast a$.\\ Seien also $a,b\in G$ beliebige Elemente. Es gilt \begin{mathe}[mc]{rcccccl} a\ast b &\eqcrefoverset{eq:1:ueb:5:ex:3}{=} &a^{-1}\ast b^{-1} &= &(b\ast a)^{-1} &\eqcrefoverset{eq:1:ueb:5:ex:3}{=} &b\ast a.\\ \end{mathe} Also ist $G$ eine kommutative Gruppe. \end{proof} \end{enumerate} %% ******************************************************************************** %% FILE: body/uebung/ueb6.tex %% ******************************************************************************** \setcounternach{chapter}{6} \chapter[Woche 6]{Woche 6} \label{ueb:6} \textbf{ACHTUNG.} Diese Lösungen dienen \emph{nicht} als Musterlösungen sondern eher als Referenz. Hier wird eingehender gearbeitet, als generell verlangt wird. Das Hauptziel hier ist, eine Variant anzubieten, gegen die man seine Versuche vergleichen kann. %% AUFGABE 6-1 \let\altsectionname\sectionname \def\sectionname{Aufgabe} \section[Aufgabe 1]{} \label{ueb:6:ex:1} \let\sectionname\altsectionname Es sei $X$ eine Menge und $R=\Pot(X)$. Auf $R$ definiere man die folgenden Verknüpfungen: \begin{mathe}[mc]{rcl} A+B &= &A\cup B\ohne(A\cap B)\\ A\cdot B &= &A\cap B\\ \end{mathe} für alle $A,B\in R$. \begin{enumerate}{\bfseries (a)} %% AUFGABE 6-1a \item Die Additions und Multiplikationstabellen für eine $3$-elementige Menge, $X=\{a,b,c\}$, sehen wie folgt aus: \hraum \begin{tabular}[mc]{|C|CCCCCCCC|} \hline + &\leer &\{c\} &\{b\} &\{b,c\} &\{a\} &\{a,c\} &\{a,b\} &\{a,b,c\}\\ \hline \leer &\leer &\{c\} &\{b\} &\{b,c\} &\{a\} &\{a,c\} &\{a,b\} &\{a,b,c\}\\ \{c\} &\{c\} &\leer &\{b,c\} &\{b\} &\{a,c\} &\{a\} &\{a,b,c\} &\{a,b\}\\ \{b\} &\{b\} &\{b,c\} &\leer &\{c\} &\{a,b\} &\{a,b,c\} &\{a\} &\{a,c\}\\ \{b,c\} &\{b,c\} &\{b\} &\{c\} &\leer &\{a,b,c\} &\{a,b\} &\{a,c\} &\{a\}\\ \{a\} &\{a\} &\{a,c\} &\{a,b\} &\{a,b,c\} &\leer &\{c\} &\{b\} &\{b,c\}\\ \{a,c\} &\{a,c\} &\{a\} &\{a,b,c\} &\{a,b\} &\{c\} &\leer &\{b,c\} &\{b\}\\ \{a,b\} &\{a,b\} &\{a,b,c\} &\{a\} &\{a,c\} &\{b\} &\{b,c\} &\leer &\{c\}\\ \{a,b,c\} &\{a,b,c\} &\{a,b\} &\{a,c\} &\{a\} &\{b,c\} &\{b\} &\{c\} &\leer\\ \hline \end{tabular} \hraum \hraum \begin{tabular}[mc]{|C|CCCCCCCC|} \hline \cdot &\leer &\{c\} &\{b\} &\{b,c\} &\{a\} &\{a,c\} &\{a,b\} &\{a,b,c\}\\ \hline \leer &\leer &\leer &\leer &\leer &\leer &\leer &\leer &\leer\\ \{c\} &\leer &\{c\} &\leer &\{c\} &\leer &\{c\} &\leer &\{c\}\\ \{b\} &\leer &\leer &\{b\} &\{b\} &\leer &\leer &\{b\} &\{b\}\\ \{b,c\} &\leer &\{c\} &\{b\} &\{b,c\} &\leer &\{c\} &\{b\} &\{b,c\}\\ \{a\} &\leer &\leer &\leer &\leer &\{a\} &\{a\} &\{a\} &\{a\}\\ \{a,c\} &\leer &\{c\} &\leer &\{c\} &\{a\} &\{a,c\} &\{a\} &\{a,c\}\\ \{a,b\} &\leer &\leer &\{b\} &\{b\} &\{a\} &\{a\} &\{a,b\} &\{a,b\}\\ \{a,b,c\} &\leer &\{c\} &\{b\} &\{b,c\} &\{a\} &\{a,c\} &\{a,b\} &\{a,b,c\}\\ \hline \end{tabular} \hraum Der Additionstabelle ist zu entnehmen, dass \fbox{$\leer$ das Nullelement} (d.\,h. additives Neutralelement) ist. Der Multiplikationstabelle ist zu entnehmen, dass \fbox{$\{a,b,c\}$ das Einselement} (d.\,h. multiplikatives Neutralelement) ist.\\ %% AUFGABE 6-1b \clearpage \item Sei nun $X$ eine allgemeine Menge. \begin{claim} \makelabel{claim:main:ueb:6:ex:1b} $(R,+,\cdot,\leer,X)$ bildet einen kommutativen Ring, wobei $R=\Pot(X)$. \end{claim} Es gibt hier zwei Ansätze. \begin{proof}[von \Cref{claim:main:ueb:6:ex:1b}, Ansatz I] Wir gehen einfach alle Axiome durch. Zunächst aber beobachten wir für alle $A,B\in R$ und $x\in X$, dass \begin{mathe}[mc]{rcl} \eqtag[eq:0:simplification:\beweislabel] x\in A+B &\textoverset{Defn}{\Longleftrightarrow} &x\in (A\cup B)\ohne(A\cap B)\\ &\Longleftrightarrow &\text{$x$ in $A$ oder $B$, aber nicht beides}\\ &\Longleftrightarrow &\text{$x$ in exakt einem von $A$, $B$}.\\ \end{mathe} Darauf werden wir uns in einigen Berechnungen berufen. \begin{kompaktitem}[\rtab][\rtab] \item[\uwave{{\bfseries Addition/Assoziativität:}}] Seien $A,B,C\in R$ beliebig. \textbf{Zu zeigen:} $(A+B)+C=A+(B+C)$.\\ Da es sich auf beiden Seiten der Gleichung um Mengen handelt, reicht es aus, für alle $x\in X$ \textbf{zu zeigen}, dass $x\in (A+B)+C$ gdw. $x\in A+(B+C)$.\\ Sei also $x\in X$ beliebig. Es gilt \begin{mathe}[mc]{rcl} x\in A+(B+C) &\eqcrefoverset{eq:0:simplification:\beweislabel}{\Longleftrightarrow} &\text{exakt eines von $x\in A$ oder $x\in B+C$ gilt}\\ &\eqcrefoverset{eq:0:simplification:\beweislabel}{\Longleftrightarrow} &\text{exakt eines von $x\in A$ oder (exakt eines von $x\in B$ oder $x\in C$) gilt}\\ &\Longleftrightarrow &\text{exakt eines von $x\in A$ oder $x\in B$ oder $x\in C$ gilt}\\ &\Longleftrightarrow &\text{$x$ in exakt einem von $A$, $B$, oder $C$}\\ \end{mathe} und \begin{mathe}[mc]{rcl} x\in (A+B)+C &\eqcrefoverset{eq:0:simplification:\beweislabel}{\Longleftrightarrow} &\text{exakt eines von $x\in A+B$ oder $x\in C$ gilt}\\ &\eqcrefoverset{eq:0:simplification:\beweislabel}{\Longleftrightarrow} &\text{exakt eines von (exakt eines von $x\in A$ oder $x\in B$) oder $x\in C$ gilt}\\ &\Longleftrightarrow &\text{exakt eines von $x\in A$ oder $x\in B$ oder $x\in C$ gilt}\\ &\Longleftrightarrow &\text{$x$ in exakt einem von $A$, $B$, oder $C$}\\ \end{mathe} Darum gilt $x\in A+(B+C)\Leftrightarrow x\in (A+B)+C$ für alle $x\in X$. Also $A+(B+C)=(A+B)+C$ für alle $A,B,C\in R$. Also ist $(R,+)$ assoziativ. \item[\uwave{{\bfseries Addition/Kommutativität:}}] Seien $A,B\in R$ beliebig. \textbf{Zu zeigen:} $A+B=B+A$.\\ Es gilt \begin{mathe}[mc]{rcccccl} A+B &\textoverset{Defn}{=} &(A\cup B)\ohne(A\cap B) &\overset{(\ast)}{=} &(B\cup A)\ohne(B\cap A) &\textoverset{Defn}{=} &B+A,\\ \end{mathe} wobei die Gleichung bei $(\ast)$ gilt, weil die Mengenoperationen, $\cap$ und $\cup$, bekanntermaßen kommutativ sind. Also ist $(R,+)$ kommutativ. \item[\uwave{{\bfseries Addition/Nullelement:}}] Wir behaupten, dass $0:=\leer$ das additive Neutralelement ist. Sei also $A\in R$ beliebig. \textbf{Zu zeigen:} $A+0=0+A=A$.\\ Wegen Kommutativität reicht es aus, $A+0=A$ zu zeigen. Es gilt \begin{mathe}[mc]{rcccccl} A+0 &\textoverset{Defn}{=} &(A\cup\leer)\ohne(A\cap\leer) &= &A\ohne\leer &= &A\\ \end{mathe} Also ist $\leer$ ein Neutralelement für $(R,+)$. \item[\uwave{{\bfseries Addition/Inverses:}}] Sei $A\in R$ beliebig. \textbf{Zu zeigen:} Es gibt ein Element $A'\in R$, so dass $A'+A=A+A'=0$.\\ Wir betrachten als Möglichkeit $A':=A$: \begin{mathe}[mc]{rcccccccl} A'+A &= &A+A &\textoverset{Defn}{=} &(A\cup A)\ohne(A\cap A) &= &A\ohne A &= &\leer.\\ \end{mathe} Da wie bereits gezeigt, $\leer$ ein Neutralelement in $(R,+)$ ist, haben wir somit bewiesen, dass $A$ sein eigenes additives Inverses ist. \item[\uwave{{\bfseries Multiplikation/Assoziativität:}}] Da die Mengenschnittoperation bekanntermaßen assoziativ ist, ist hier eigentlich nichts zu zeigen. \item[\uwave{{\bfseries Multiplikation/Kommutativität:}}] Da die Mengenschnittoperation bekanntermaßen kommutativ ist, ist hier eigentlich nichts zu zeigen. \item[\uwave{{\bfseries Multiplikation/Einselement:}}] Wir behaupten, dass $1:=X$ das multiplikative Neutralelement ist. Sei also $A\in R$ beliebig. \textbf{Zu zeigen:} $A\cdot 1=1\cdot A=A$.\\ Wegen Kommutativität reicht es aus, $A\cdot 1=A$ zu zeigen. Es gilt \begin{mathe}[mc]{rcccl} A\cdot 1 &= &A\cap X &= A,\\ \end{mathe} weil $A\in R=\Pot(X)$ und damit $A\subseteq X$ gilt. Also ist $X$ ein Neutralelement für $(R,\cdot)$. \item[\uwave{{\bfseries Linksdistributivität:}}] Seien $A,B,C\in R$ beliebig. \textbf{Zu zeigen:} $A\cdot(B+C)=(A\cdot B)+(A\cdot C)$.\\ Da es sich auf beiden Seiten der Gleichung um Mengen handelt, reicht es aus, für alle $x\in X$ \textbf{zu zeigen}, dass $x\in A\cdot(B+C)$ gdw. $x\in (A\cdot B)+(A\cdot C)$.\\ Sei also $x\in X$ beliebig. Es gilt \begin{mathe}[mc]{rcl} x\in A\cdot(B+C) &\textoverset{Defn}{\Longleftrightarrow} &x\in A\cap((B\cup C)\ohne(B\cap C))\\ &\eqcrefoverset{eq:0:simplification:\beweislabel}{\Longleftrightarrow} &\text{$x$ in $A$ und $x$ in exakt einer der Mengen $B$, $C$}\\ &\Longleftrightarrow &\text{$x$ in exakt einer der Mengen $A\cap B$, $A\cap C$}\\ &\eqcrefoverset{eq:0:simplification:\beweislabel}{\Longleftrightarrow} &x\in (A\cdot B)+(A\cdot C).\\ \end{mathe} Also gilt $A\cdot(B+C)=(A\cdot B)+(A\cdot C)$. Also weist $(R,+,\cdot)$ linksdistributivität auf. \item[\uwave{{\bfseries Rechtsdistributivität:}}] Da Multiplikation kommutativ ist, folgt Rechtsdistributivität automatisch aus Linksdistributivität. \end{kompaktitem} Darum erfüllt $(R,+,\cdot)$ die Axiome eines Rings und dieser Ring hat ein Einselement und heißt kommutativ, weil hier Multiplikation kommutativ ist. \end{proof} \clearpage \begin{proof}[von \Cref{claim:main:ueb:6:ex:1b}, Ansatz II] Ein scharfes Auge erkennt, dass wir Teilmengen von $X$ mit binären Tupeln identifizieren kann. Vielmehr wollen wir diese Menge von binären Tupeln mit einer bekannten algebraischen Struktur in Verbindung setzen, also betrachten wir konkret die Abbildungen \begin{mathe}[mc]{rclcl} \Phi &: &\prod_{x\in X}\intgr/2\intgr &\to &\Pot(X)\\ &: &\alpha &\mapsto &\supp(\alpha):=\{x\in X\mid \alpha_{x}=1\}\\ \\ \Psi &: &\Pot(X) &\to &\prod_{x\in X}\intgr/2\intgr\\ &: &A &\mapsto &(\einser_{A}(x))_{x\in X}\\ \end{mathe} um Elemente aus dem einen Raum auf Elemente aus dem anderen zu übertragen. Nun ist $\intgr/2\intgr$ bekanntermaßen ein kommutativer Ring (eigentlich ein Körper). Darum ist das Produkt $\prod_{x\in X}\intgr/2\intgr$, versehen mit punktweise Addition und punktweise Multiplikation, ebenfalls ein kommutativer Ring. Darum reicht es aus \textbf{zu zeigen}, dass $\Phi$ eine Bijektion ist, die die Operationen erhält (auch \emph{Isomorphismus} genannt). \begin{kompaktitem}[\rtab][\rtab] \item[\uwave{{\bfseries Bijektion:}}] Wir beobachten, dass \begin{mathe}[mc]{rclcl} \Phi(\Psi(A)) &= &\{x\in X\mid \Psi(A)_{x}=1\}\\ &= &\{x\in X\mid \einser_{A}(x)=1\}\\ &= &\{x\in X\mid x\in A\} &= &A\\ \end{mathe} für alle $A\in\Pot(X)$ und \begin{mathe}[mc]{rclcl} \Psi(\Phi(\alpha)) &= &(\einser_{\Phi(\alpha)}(x))_{x\in X}\\ &= &(\einser_{\{x'\in X\mid \alpha_{x'}=1\}}(x))_{x\in X}\\ &= &\left( \begin{cases}[mc]{lcl} 1 &: &x\in\{x'\in X\mid \alpha_{x'}=1\}\\ 0 &: &\text{sonst}\\ \end{cases} \right)_{x\in X}\\ &= &\left( \begin{cases}[mc]{lcl} 1 &: &\alpha_{x}=1\\ 0 &: &\alpha_{x}=0\\ \end{cases} \right)_{x\in X} &= &(\alpha_{x})_{x\in X} = \alpha\\ \end{mathe} für alle $\alpha\in\prod_{x\in X}\intgr/2\intgr$. Also $\Phi\circ\Psi=\id$ und $\Psi\circ\Phi=\id$. Darum sind $\Phi$ und $\Psi$ Bijektion (und invertieren einander). \item[\uwave{{\bfseries Erhaltung der Operationen:}}] Seien $\alpha,\beta\in\prod_{x\in X}\intgr/2\intgr$. \textbf{Zu zeigen:} $\Phi(\alpha+\beta)=\Phi(\alpha)+\Phi(\beta)$ und $\Phi(\alpha\cdot\beta)=\Phi(\alpha)\cdot\Phi(\beta)$. Es gilt \begin{mathe}[mc]{rcl} \Phi(\alpha+\beta) &= &\{x\in X\mid (\alpha+\beta)_{x}=1\}\\ &= &\{x\in X\mid \alpha_{x}+\beta_{x}=1\},\\ &&\quad\text{da Operationen auf $\prod_{x\in X}\intgr/2\intgr$ punktweise definiert sind}\\ &= &\{x\in X\mid \alpha_{x}=1\,\text{od.}\,\beta_{x}=1,\,\text{aber nicht beides}\}\\ &= &(\{x\in X\mid \alpha_{x}=1\}\cup\{x\in X\mid \beta_{x}=1\})\\ &&\,\ohne\,(\{x\in X\mid \alpha_{x}=1\}\cap\{x\in X\mid \beta_{x}=1\})\\ &= &(\Phi(\alpha)\cup\Phi(\beta)) \ohne(\Phi(\alpha)\cap\Phi(\beta)) \quad\text{per Konstruktion von $\Phi$}\\ &= &\Phi(\alpha)+\Phi(\beta) \quad\text{per Definition von Addition in $R$}\\ \end{mathe} und \begin{mathe}[mc]{rcl} \Phi(\alpha\cdot\beta) &= &\{x\in X\mid (\alpha\cdot\beta)_{x}=1\}\\ &= &\{x\in X\mid \alpha_{x}\cdot\beta_{x}=1\},\\ &&\quad\text{da Operationen auf $\prod_{x\in X}\intgr/2\intgr$ punktweise definiert sind}\\ &= &\{x\in X\mid \alpha_{x}=1\,\text{und.}\,\beta_{x}=1\}\\ &= &\{x\in X\mid \alpha_{x}=1\}\cap\{x\in X\mid \beta_{x}=1\}\\ &= &\Phi(\alpha)\cap\Phi(\beta) \quad\text{per Konstruktion von $\Phi$}\\ &= &\Phi(\alpha)\cdot\Phi(\beta) \quad\text{per Definition von Multiplikation in $R$}.\\ \end{mathe} Darum präserviert $\Phi$ die Operationen. \end{kompaktitem} Zusammegefasst haben wir gezeigt, dass $(\Pot(X),+,\cdot)$ zu dem kommutativen Ring, $(\prod_{x\in X}\intgr/2\intgr,+,\cdot)$ isomorph ist (und zwar mittels $\Phi$), und damit dass $(R,+,\cdot)$ selbst ein kommutativer Ring ist. \end{proof} \textbf{Bemerkung.} Aus diesem Beweis geht hervor, dass das Nullelement durch $\Phi((0)_{x\in X})=\leer$ und das Einselement durch $\Phi((1)_{x\in X})=X$ gegeben sind. \end{enumerate} %% AUFGABE 6-2 \clearpage \let\altsectionname\sectionname \def\sectionname{Aufgabe} \section[Aufgabe 2]{} \label{ueb:6:ex:2} \let\sectionname\altsectionname Wir identifizieren $\kmplx$ mit $\reell^{2}$ mittel der Abbildungen \begin{mathe}[mc]{rcl} z\in\kmplx &\mapsto &\begin{svector} \ReTeil(z)\\ \ImTeil(z)\\\end{svector}\in\reell^{2},\\ \mathbf{x}\in\reell^{2} &\mapsto &x_{1}+\imageinh x_{2}\in\kmplx.\\ \end{mathe} \begin{enumerate}{\bfseries (a)} %% AUFGABE 6-2a \item \begin{claim*} Für alle $z\in\kmplx\ohne\{0\}$ existieren eindeutige Werte $r\in(0,\infty)$ und $\alpha\in[0,2\pi)$, dann $z=r\cdot\begin{svector} \cos(\alpha)\\ \sin(\alpha)\\\end{svector}$ (unter der o.\,s. Identifizierung). \end{claim*} \begin{proof} Unter der Identifizierung können wir $z=\begin{svector} x\\ y\\\end{svector}$ schreiben, wobei $x,y\in\reell$. Da $z\neq 0=\begin{svector} 0\\ 0\\\end{svector}$, muss entweder $x\neq 0$ oder $y\neq 0$ gelten. Zur {\bfseries Existenz}: Sei $r:=\sqrt{x^{2}+y^{2}}$. Dann $r>0$ weil $(x,y)\neq (0,0)$.\\ Um $\alpha$ zu bestimmen, werden folgende Fälle aufgeführt: \begin{kompaktenum}{\bfseries {Fall} 1.}[\rtab][\rtab] \item $y=0$. Dann $x\neq 0$ und in diesem Falle gilt $r=|x|$. Man setze $\alpha := \begin{cases}[mc]{lcl} 0 &: &x>0\\ \pi &: &x<0\\ \end{cases}$. Dann $r\cos(\alpha) := \begin{cases}[mc]{lcl} r &: &x>0\\ -r &: &x<0\\ \end{cases} = x$ und $r\sin(\alpha)=0$. \item $y>0$. Man setze $\alpha\in(0,\pi)$ der eindeutige Winkel mit $\cos(\alpha)=\frac{x}{r}$. Dann $r\cos(\alpha)=x$ und $r\sin(\alpha)=r\sqrt{1-\cos^{2}(\alpha)} =\sqrt{r^{2}-(r\cos(\alpha))^{2}} =\sqrt{(x^{2}+y^{2})-x^{2}} =\sqrt{y^{2}} =|y|=y$. \item $y<0$. Man setze $\alpha\in(\pi,2\pi)$ der eindeutige Winkel mit $\cos(\alpha)=\frac{x}{r}$. Dann $r\cos(\alpha)=x$ und $r\sin(\alpha)=r\cdot -\sqrt{1-\cos^{2}(\alpha)} =-\sqrt{r^{2}-(r\cos(\alpha))^{2}} =-\sqrt{(x^{2}+y^{2})-x^{2}} =-\sqrt{y^{2}} =-|y|=y$. \end{kompaktenum} Darum gilt in allen Fällen $r\cdot\begin{svector} \cos(\alpha)\\ \sin(\alpha)\\\end{svector} =\begin{svector} r\cos(\alpha)\\ r\sin(\alpha)\\\end{svector} =\begin{svector} x\\ y\\\end{svector} =z$. Zur {\bfseries Eindeutigkeit}: Seien $r_{i}\in(0,\infty)$, $\alpha_{i}\in[0,2\pi)$ mit $r_{i}\cdot\begin{svector} \cos(\alpha_{i})\\ \sin(\alpha_{i})\\\end{svector}=z$ für $i\in\{1,2\}$. \textbf{Zu zeigen:} $r_{1}=r_{2}$ und $\alpha_{1}=\alpha_{2}$. Es gilt \begin{mathe}[mc]{rclcl} r_{1}^{2} &= &r_{1}^{2}(\cos^{2}(\alpha_{1}) + \sin^{2}(\alpha_{1}))\\ &= &(r_{1}\cos(\alpha_{1}))^{2} + (r_{1}\sin(\alpha_{1}))^{2}\\ &= &x^{2}+y^{2}\\ &= &(r_{2}\cos(\alpha_{2}))^{2} + (r_{2}\sin(\alpha))^{2}\\ &= &r_{2}^{2}(\cos^{2}(\alpha_{2}) + \sin^{2}(\alpha_{2})) &= &r_{2}^{2},\\ \end{mathe} woraus sich ergibt, dass $r_{1}=r_{2}$, weil $r_{1},r_{2}\geq 0$. Da $r_{1},r_{2}>0$, folgt \begin{mathe}[mc]{ccccccccccl} \cos(\alpha_{1}) &= &\dfrac{r_{1}\cos(\alpha_{1})}{r_{1}} &= &\dfrac{x}{r_{1}} &= &\dfrac{x}{r_{2}} &= &\dfrac{r_{2}\cos(\alpha_{2})}{r_{2}} &= &\cos(\alpha_{2})\\ \sin(\alpha_{1}) &= &\dfrac{r_{1}\sin(\alpha_{1})}{r_{1}} &= &\dfrac{y}{r_{1}} &= &\dfrac{y}{r_{2}} &= &\dfrac{r_{2}\sin(\alpha_{2})}{r_{2}} &= &\sin(\alpha_{2})\\ \end{mathe} Da $\alpha_{1},\alpha_{2}\in[0,2\pi)$ und wegen Injektivität von $\cos$ auf $[0,\pi)$ und $[\pi,2\pi)$ und der Symmetrie um $\pi$, erhalten wir aus $\cos(\alpha_{1})=\cos(\alpha_{2})$, dass (i)~$\alpha_{1}=\alpha_{2}$ oder (ii)~$\alpha_{1}=2\pi-\alpha_{2}$ gelten muss.\\ Falls (ii) gilt, so gilt $\sin(\alpha_{1})=\sin(2\pi-\alpha_{2})=-\sin(\alpha_{2})$. Da aber $\sin(\alpha_{1})=\sin(\alpha_{2})$, folgt daraus $\sin(\alpha_{2})=0$, und damit (iii)~$\alpha_{2}=0$ oder (iv)~$\pi$. Falls (iii) gilt, so gilt wegen (ii) $\alpha_{1}=2\pi-0=2\pi$, was ein Widerspruch ist, weil $\alpha_{1}\in[0,2\pi)$. Darum muss (iv) gelten. Wegen (ii) gilt also $\alpha_{1}=2\pi-\pi=\pi=\alpha_{2}$.\\ Zusammegefasst gilt entweder (i) $\alpha_{1}=\alpha_{2}$ oder (ii), aus dem sich (iv) ergibt, was wiederum $\alpha_{1}=\alpha_{2}$ zur Folge hat. D.\,h., in allen Fällen gilt $\alpha_{1}=\alpha_{2}$. Darum gelten $r_{1}=r_{2}$ und $\alpha_{1}=\alpha_{2}$. Also ist die Darstellung eindeutig. \end{proof} %% AUFGABE 6-2b \clearpage \item \begin{claim*} Seien $z_{1},z_{2}\in\kmplx\ohne\{0\}$ mit Darstellungen $z_{i}=r_{i}\cdot\begin{svector} \cos(\alpha_{i})\\ \sin(\alpha_{i})\\\end{svector}$ für $i\in\{1,2\}$. Dann gilt die Rechenregel $z_{1}z_{2}=r_{1}r_{2}\cdot\begin{svector} \cos(\alpha_{1}+\alpha_{2})\\ \sin(\alpha_{1}+\alpha_{2})\\\end{svector}$. \end{claim*} \begin{proof} Multiplikation in $\kmplx$ liefert \begin{longmathe}[mc]{RCL} z_{1}z_{2} &= &\begin{svector} \ReTeil(z_{1})\ReTeil(z_{2})-\ImTeil(z_{1})\ImTeil(z_{2})\\ \ReTeil(z_{1})\ImTeil(z_{2})+\ReTeil(z_{1})\ImTeil(z_{2})\\\end{svector}\\ &= &\begin{svector} r_{1}\cos(\alpha_{1})\cdot r_{2}\cos(\alpha_{2})-r_{1}\sin(\alpha_{1})\cdot r_{2}\sin(\alpha_{2})\\ r_{1}\cos(\alpha_{1})\cdot r_{2}\sin(\alpha_{2})+r_{1}\cos(\alpha_{1})\cdot r_{2}\sin(\alpha_{2})\\\end{svector}\\ &= &r_{1}r_{2}\begin{svector} \cos(\alpha_{1})\cos(\alpha_{2})-\sin(\alpha_{1})\sin(\alpha_{2})\\ \cos(\alpha_{1})\sin(\alpha_{2})+\cos(\alpha_{1})\sin(\alpha_{2})\\\end{svector}\\ &= &r_{1}r_{2}\begin{svector} \cos(\alpha_{1}+\alpha_{2})\\ \sin(\alpha_{1}+\alpha_{2})\\\end{svector}.\\ \end{longmathe} Die letzte Vereinfachung folgt aus der trigonometrischen Additionsregel. \end{proof} %% AUFGABE 6-2c \item \begin{claim*}[de Moivre] Sei $z\in\kmplx\ohne\{0\}$ mit Darstellungen $z=r\cdot\begin{svector} \cos(\alpha)\\ \sin(\alpha)\\\end{svector}$. Dann gilt die Potenzregel $z^{n}=r^{n}\cdot\begin{svector} \cos(n\alpha)\\ \sin(n\alpha)\\\end{svector}$ für alle $n\in\ntrlpos$. \end{claim*} \begin{proof} Wir beweisen dies per Induktion über $n$. \begin{kompaktenum}[\rtab][\rtab] \item[\uwave{{\bfseries Induktionsanfang:}}] Die Gleichung gilt offensichtlich für $n=1$. \item[\uwave{{\bfseries Induktionsvoraussetzung:}}] Sei $n>1$. Angenommen, $z^{n-1}=r^{n-1}\cdot\begin{svector} \cos((n-1)\alpha)\\ \sin((n-1)\alpha)\\\end{svector}$. \item[\uwave{{\bfseries Induktionsschritt:}}] \textbf{Zu zeigen:} $z^{n}=r^{n}\cdot\begin{svector} \cos(n\alpha)\\ \sin(n\alpha)\\\end{svector}$.\\ Per rekursive Definition vom Potenzieren gilt zunächst $z^{n}=z^{n-1}\cdot z$ (Multiplikation innerhalb der Algebra $\kmplx$). Aufgabe 6-2(b) zur Folge gilt somit \begin{mathe}[mc]{rcl} z^{n}=z^{n-1}\cdot z &\textoverset{IV}{=} &r^{n-1}\cdot\begin{svector} \cos((n-1)\alpha)\\ \sin((n-1)\alpha)\\\end{svector} \cdot r\cdot\begin{svector} \cos(\alpha)\\ \sin(\alpha)\\\end{svector}\\ &\textoverset{(2b)}{=} &r^{n-1}r\cdot\begin{svector} \cos((n-1)\alpha+\alpha)\\ \sin((n-1)\alpha+\alpha)\\\end{svector}\\ &= &r^{n}\cdot\begin{svector} \cos(n\alpha)\\ \sin(n\alpha)\\\end{svector}.\\ \end{mathe} Darum gilt die Gleichung für $n$. \end{kompaktenum} Also gilt die Gleichung für alle $n\in\ntrlzero$. \end{proof} \begin{rem*} Wir können eigentlich zeigen, dass dies für alle $n\in\intgr$ gilt. Für $n=0$, gilt $% z^{0} =1+\imageinh 0 =\begin{svector} 1\\ 0\\\end{svector} =r^{0}\cdot\begin{svector} \cos(0\alpha)\\ \sin(0\alpha)\\\end{svector}% $. Für $n=-1$ liefert uns die Rechenregel für Multiplikation innerhalb $\kmplx$, dass $r^{-1}\cdot\begin{svector} \cos(-\alpha)\\ \sin(-\alpha)\\\end{svector}$ eine hinreichende Konstruktion für ein Inverses von $z$ ist, und darum ist dies wegen Eindeutigkeit des Inverses gleich $z^{-1}$. Für $n<0$ allgemein wenden wir schließlich $% z^{n} =(z^{-1})^{|n|} =(r^{-1}\cdot\begin{svector} \cos(-\alpha)\\ \sin(-\alpha)\\\end{svector})^{|n|} =(r^{-1})^{|n|}\cdot\begin{svector} \cos(|n|\cdot-\alpha)\\ \sin(|n|\cdot-\alpha)\\\end{svector} =r^{n}\cdot\begin{svector} \cos(n\alpha)\\ \sin(n\alpha)\\\end{svector}% $ an. \end{rem*} \end{enumerate} %% AUFGABE 6-3 \clearpage \let\altsectionname\sectionname \def\sectionname{Aufgabe} \section[Aufgabe 3]{} \label{ueb:6:ex:3} \let\sectionname\altsectionname Es sei $K$ ein Körper und $F:=K\times K$ versehen mit den Operationen ${+,\cdot:F\times F\to F}$, definiert vermöge \begin{mathe}[mc]{rcl} (a,b)+(a',b') &= &(a+a',b+b')\\ (a,b)\cdot (a',b') &= &(aa'-bb',ab'+a'b)\\ \end{mathe} für alle $a,b,a',b'\in K$. Wir werden folgendes klassifizierendes Ergebnis verwenden, um die Aufgaben zu behandeln (und dieses Resultat dann anschließend beweisen). \begin{satz} \makelabel{satz:1:ueb:6:ex:3} $(F,+,\cdot)$ ist genau dann ein Körper, wenn $a^{2}+b^{2}\neq 0$ innerhalb $K$ für alle $(a,b)\in F\ohne\{(0,0)\}$. \end{satz} \begin{enumerate}{\bfseries (a)} %% AUFGABE 6-3a \item \begin{schattierteboxdunn} \begin{claim*} Sei $K=\mathbf{F}_{2}=\intgr/2\intgr$. Dann ist $(F,+,\cdot)$ kein Körper. \end{claim*} \end{schattierteboxdunn} \begin{proof} Da $(a,b):=(1,1)\in F\ohne\{(0,0)\}$ und $a^{2}+b^{2}=1+1=0$ innerhalb $K=\intgr/2\intgr$, ist \Cref{satz:1:ueb:6:ex:3} zufolge $F$ kein Körper. Es scheitert genau das Axiom der Existenz multiplikativer Inverser. (Nichtsdestotrotz bildet $F$ einen kommutativen Ring mit Einselement.) \end{proof} %% AUFGABE 6-3b \item \begin{schattierteboxdunn} \begin{claim*} Sei $K=\mathbf{F}_{3}=\intgr/3\intgr$. Dann ist $(F,+,\cdot)$ ein Körper. \end{claim*} \end{schattierteboxdunn} \begin{proof} Laut \Cref{satz:1:ueb:6:ex:3} reicht es aus für alle $(a,b)\in F\ohne\{(0,0)\}$ \textbf{zu zeigen}, dass $a^{2}+b^{2}\neq 0$ innerhalb $K=\intgr/3\intgr$. Sei also $(a,b)\in F\ohne\{(0,0)\}$ beliebig. Da $a\neq 0$ oder $b\neq 0$ gibt es folgende Fälle: \begin{kompaktenum}{\bfseries {Fall} 1.}[\rtab][\rtab] %% FALL 1 \item $a=0$, $b\neq 0$. Dann $b=\pm 1\mod 3$. Also $a^{2}+b^{2}=0+1=1\nequiv 0\mod 3$. %% FALL 2 \item $a\neq 0$, $b=0$. Dann $a=\pm 1\mod 3$. Also $a^{2}+b^{2}=1+0=1\nequiv 0\mod 3$. %% FALL 3 \item $a\neq 0$, $b\neq 0$. Dann $a,b=\pm 1\mod 3$. Also $a^{2}+b^{2}=1+1=2\nequiv 0\mod 3$. \end{kompaktenum} Also gilt in jedem Falle $a^{2}+b^{2}\neq 0$. Darum bildet $F$ einen Körper. \end{proof} \end{enumerate} Um \Cref{satz:1:ueb:6:ex:3} zu beweisen, brauchen wir zunächst folgendes Zwischenresultat. \begin{lemm} \makelabel{lemm:1:ueb:6:ex:3} $(F,+,\cdot)$ ist genau dann ein Körper, wenn in der Teilstruktur $(F,\cdot)$ multiplikative Inverse existieren für jedes Element. \end{lemm} \begin{einzug}[\rtab][\rtab] \begin{proof}[von \Cref{lemm:1:ueb:6:ex:3}][\Cref{lemm:1:ueb:6:ex:3}] Da die Teilstruktur, $(F,+)$, durch die Produktstruktur $(K,+)\times (K,+)$ gegeben ist, dessen Faktoren all kommutative Gruppen sind, ist $(F,+)$ eine kommutative Gruppe. Das heißt, die {\bfseries Additionsaxiome} unter den Körperaxiomen sind allesamt erfüllt. (Insbesondere ist das Nullelement durch $0_{F}=(0,0)$ gegeben.) Bei den {\bfseries Multiplikationsaxiomen} sehen wir dass Kommutativität offensichtlich gilt, weil die o.\,s. Definitions von Multiplikation in den Argumenten offensichtlich symmetrisch ist, und weil die Operationen in $K$ kommutativ sind. Es gilt auch $(a,b)\cdot(1,0)=(a\cdot 1-b\cdot 0,a\cdot 0+1\cdot b)=(a,b)$, sodass $1_{F}:=(1,0)$ das Einselement von $F$ ist. Assoziativität von Multiplikation ist auch erfüllt, weil \begin{mathe}[mc]{rcl} (a,b)\cdot((a',b')\cdot(a'',b'')) &= &(a,b)\cdot(a'a''-b'b'', a'b''+a''b')\\ &= &(a(a'a''-b'b'')-b(a'b''+a''b'), a(a'b''+a''b')+(a'a''-b'b'')b)\\ &= &(aa'a'' - ab'b'' - ba'b'' - ba''b', aa'b'' + aa''b' + a'a''b - b'b''b)\\ &= &\boxed{(aa'a'' - ab'b'' - a'bb'' - a''bb', aa'b'' + aa''b' + a'a''b - bb'b'')}\\ \end{mathe} und \begin{mathe}[mc]{rcl} ((a,b)\cdot(a',b'))\cdot(a'',b'') &= &(aa'-bb',ab'+a'b)\cdot(a'',b'')\\ &= &((aa'-bb')a'' - (ab'+a'b)b'', (aa'-bb')b'' + a''(ab'+a'b))\\ &= &(aa'a'' - bb'a'' - ab'b'' - a'bb'', aa'b'' - bb'b'' + a''ab' + a''a'b)\\ &= &\boxed{(aa'a'' - ab'b'' - a'bb'' - a''bb', aa'b'' + aa''b' + a'a''b - bb'b'')}\\ &= &(a,b)\cdot((a',b')\cdot(a'',b'')) \end{mathe} für alle $(a,b),(a',b'),(a'',b'')\in F$. Darum ist $(F,\cdot)$ assoziativ, kommutativ, und hat ein Neutralelement. Wegen Kommutativität von Multiplikation in $F$, ist {\bfseries Distributitivität} zu Linksdistributivität äquivalent, und da \begin{mathe}[mc]{rcl} (a,b)\cdot((a',b')+(a'',b'')) &= &(a,b)\cdot(a'+a'',b'+b'')\\ &= &(a(a'+a'')-b(b'+b''), a(b'+b'')+(a'+a'')b)\\ &= &((aa'-bb')+(aa''-bb''), (ab'+a'b)+(ab''+a''b))\\ &= &(aa'-bb',ab'+a'b)+(aa''-bb'', ab''+a''b)\\ &= &(a,b)\cdot(a',b')+(a,b)/cdot (a'',b'')\\ \end{mathe} für alle $(a,b),(a',b'),(a'',b'')\in F$, ist dies erfüllt. Darum erfüllt $(F,+,\cdot)$ jedes Axiom eines Körpers, evtl. bis auf das Axiom für multiplikative Inverse. Darum gilt: $(F,+,\cdot)$ bildet einen Körper $\Leftrightarrow$ jedes Element in $F\ohne\{0_{F}\}$ hat ein multiplikatives Inverses. \end{proof} \end{einzug} Jetzt können wir uns dem Beweis von \Cref{satz:1:ueb:6:ex:3} widmen \begin{proof}[von \Cref{satz:1:ueb:6:ex:3}][\Cref{satz:1:ueb:6:ex:3}] Laut \Cref{lemm:1:ueb:6:ex:3} gilt $(F,+,\cdot)$ ein Körper gdw. jedes Element in $F$ hat ein multiplikatives Inverses. Darum reicht es aus \textbf{zu zeigen}, dass \begin{mathe}[mc]{rcl} \forall{(a,b)\in F\ohne\{0_{F}\}:~}\exists{(a',b')\in F:~}(a,b)\cdot (a',b')=1_{F} &\Longleftrightarrow &\forall{(a,b)\in F\ohne\{0_{F}\}:~}a^{2}+b^{2}\neq 0 \end{mathe} gilt.\footnote{ Man beachte: Wegen multiplikativer Kommutativität folgt aus $(a,b)\cdot (a',b')=1_{F}$, dass auch $(a',b')\cdot (a,b)=1_{F}$ gilt. } \herRichtung Angenommen, $a^{2}+b^{2}\neq 0$ für alle $(a,b)\in F\ohne\{0_{F}\}$.\\ Sei $(a,b)\in F\ohne\{0_{F}\}$ beliebig. \textbf{Zu zeigen:} $(a,b)$ sei innerhalb $F$ invertierbar.\\ Per Annahme gilt nun $r:=a^{2}+b^{2}\neq 0$ und somit ist $r$ innerhalb $K$ invertierbar. Setze $(a',b'):=(r^{-1}a,-r^{-1}b)$. Dann \begin{mathe}[mc]{rcl} (a,b)\cdot(a',b') &= &(a\cdot r^{-1}a-b(-r^{-1}b),a(-r^{-1}b)+(r^{-1}a)b)\\ &= &(r^{-1}(a^{2}+b^{2}), 0)\\ &= &(r^{-1}r, 0)\\ &= &(1, 0)\\ &= &1_{F}.\\ \end{mathe} Also ist jedes $(a,b)\in F\ohne\{0_{F}\}$ innerhalb $F$ invertierbar. \hinRichtung Angenommen, jedes Element in $F\ohne\{0_{F}\}$ sei invertierbar.\\ Wie oben erklärt, ist $(F,+,\cdot)$ somit ein Körper.\\ Sei $(a,b)\in F\ohne\{0_{F}\}$ beliebig. \textbf{Zu zeigen:} $a^{2}+b^{2}\neq 0$.\\ Da $(a,b)\neq 0_{F}=(0,0)$, gilt $a\neq 0$ oder $b\neq 0$ und damit gilt auch $(a,-b)\in F\ohne\{0_{F}\}$.\\ Da $F$ ein Körper ist, sind $(a,b)$ und $(a,-b)$ und folglich auch das Produkt $(a,b)\cdot(a,-b)$ invertierbar.\\ Da $F$ ein Körper ist, bedeutet dies wiederum, dass $(a,b)\cdot(a,-b)\neq 0_{F}$ gilt. Nun, \begin{mathe}[mc]{rclcl} (a,b)\cdot(a,-b) &= &(aa-b(-b),a(-b)+ab) &= &(a^{2}+b^{2},0).\\ \end{mathe} Darum gilt $(a^{2}+b^{2},0)=(a,b)\cdot(a,-b)\neq 0_{F}=(0,0)$, woraus sich $a^{2}+b^{2}\neq 0$ ergibt. \end{proof} %% ******************************************************************************** %% FILE: body/uebung/ueb7.tex %% ******************************************************************************** \setcounternach{chapter}{7} \chapter[Woche 7]{Woche 7} \label{ueb:7} \textbf{ACHTUNG.} Diese Lösungen dienen \emph{nicht} als Musterlösungen sondern eher als Referenz. Hier wird eingehender gearbeitet, als generell verlangt wird. Das Hauptziel hier ist, eine Variant anzubieten, gegen die man seine Versuche vergleichen kann. %% AUFGABE 7-1 \let\altsectionname\sectionname \def\sectionname{Aufgabe} \section[Aufgabe 1]{} \label{ueb:7:ex:1} \let\sectionname\altsectionname Betrachte den Vektorraum $V:=\functionspace(\reell,\reell)$ über dem Körper $\reell$. Wir bezeichnen mit $0(\cdot)$ die konstante Funktion, die überall gleich $0$ ist. Dies ist zufälligerweise auch das Nullelement von $V$. \begin{enumerate}{\bfseries (a)} %% AUFGABE 7-1a \item \begin{claim*} Seien $a,b\in\reell$ beliebig. Dann \fbox{ist} $U_{1}:=\{f\in V\mid f(a)=f(b)\}$ ein Untervektorraum (»linearer Unterraum«). \end{claim*} \begin{proof} Wir gehen die Axiome aus der Definition durch: \begin{kompaktitem}[\rtab][\rtab] \item[{\bfseries (NL)}] Offensichtlich gilt $0(\cdot)\in U_{1}$, da diese Funktion überall und damit insbesondere auf $\{a,b\}$ gleich ist. Also, $U_{1}\neq\leer$. \item[{\bfseries (LK)}] Seien $\alpha,\beta\in\reell$ und $f,g\in U_{1}$. \textbf{Zu zeigen:} $\alpha\cdot f+\beta\cdot g\in U_{1}$\\ Es gilt \begin{mathe}[mc]{rcl} (\alpha\cdot f+\beta\cdot g)(a) &= &\alpha\cdot f(a)+\beta\cdot g(a)\\ &\overset{(\ast)}{=} &\alpha\cdot f(b)+\beta\cdot g(b)\\ &= &(\alpha\cdot f+\beta\cdot g)(b)\\ \end{mathe} Hier gilt $(\ast)$, weil $f,g\in U_{1}$.\\ Darum gilt per Konstruktion, dass $\alpha\cdot f+\beta\cdot g\in U_{1}$. \end{kompaktitem} Darum bildet $U_{1}$ einen Untervektorraum. \end{proof} %% AUFGABE 7-1b \item \begin{claim*} Seien $a,b\in\reell$ beliebig. Dann ist $U_{2}:=\{f\in V\mid f(a)=f(b)=1\}$ \fbox{kein Untervektorraum}. \end{claim*} \begin{proof} Offensichtlich ist der Nullvektor, $0(\cdot)$, nicht in $U_{2}$, da $0(a)=0\neq 1$. Darum kann $U_{2}$ kein Untervektorraum sein. (Siehe \cite[Lemma~5.1.3]{sinn2020}.) \end{proof} Alternativ für die $\Rightarrow$-Richtung kann man folgendermaßen argumentieren: die konstante Funktion $1(\cdot)$ liegt in $U_{2}$, aber $0\cdot 1(\cdot)\notin U_{2}$, sodass $U_{2}$ nicht unter Skalarmultiplikation stabil ist. %% AUFGABE 7-1c \item \begin{claim*} Die Teilmenge $U_{3}:=\{f\in V\mid f\,\text{injektiv}\}$ \fbox{ist kein Untervektorraum} von $V$. \end{claim*} \begin{proof} Offensichtlich ist der Nullvektor, $0(\cdot)$, nicht in $U_{3}$, weil diese konstante Funktion nicht injektiv ist. Darum kann $U_{3}$ kein Untervektorraum sein. (Siehe \cite[Lemma~5.1.3]{sinn2020}.) \end{proof} %% AUFGABE 7-1d \item \begin{claim*} Die Teilmenge $U_{4}:=\{f\in V\mid f\,\text{surjektiv}\}$ \fbox{ist kein Untervektorraum} von $V$. \end{claim*} \begin{proof} Offensichtlich ist der Nullvektor, $0(\cdot)$, nicht in $U_{4}$, weil die konstante Funktion nicht surjektiv ist. Darum kann $U_{4}$ kein Untervektorraum sein. (Siehe \cite[Lemma~5.1.3]{sinn2020}.) \end{proof} \end{enumerate} %% AUFGABE 7-2 \clearpage \let\altsectionname\sectionname \def\sectionname{Aufgabe} \section[Aufgabe 2]{} \label{ueb:7:ex:2} \let\sectionname\altsectionname \begin{enumerate}{\bfseries (a)} %% AUFGABE 7-2a \item \begin{claim*} Sei $K=\rtnl$. Dann sind ${\mathbf{v}_{1}=\begin{svector} 1\\ 2\\ 2\\\end{svector}}$, ${\mathbf{v}_{2}=\begin{svector} 3\\ 2\\ 1\\\end{svector}}$, und ${\mathbf{v}_{3}=\begin{svector} 2\\ 1\\ -1\\\end{svector}}$ über $K$ \fbox{linear unabhängig}. \end{claim*} \begin{proof} Es reicht aus, den (Zeilen)rang der Matrix \begin{mathe}[mc]{rcl} A &:= &\begin{matrix}{ccc} 1 &3 &2\\ 2 &2 &1\\ 2 &1 &-1\\ \end{matrix} \end{mathe} zu untersuchen. Wir berechnen \begin{algorithm}[\rtab][\rtab] Reduktion der Matrix, $A$, mittels des Gaußverfahrens:\\ Zeilentransformationen ${Z_{2} \leftsquigarrow 3\cdot Z_{1}-Z_{2}}$ und ${Z_{3} \leftsquigarrow 2\cdot Z_{1}-Z_{3}}$ anwenden: \begin{mathe}[mc]{c} \begin{matrix}{ccc} 1 &2 &2\\ 0 &4 &5\\ 0 &3 &5\\ \end{matrix}\\ \end{mathe} Zeilentransformation ${Z_{3} \leftsquigarrow 4\cdot Z_{3}-3\cdot Z_{2}}$ anwenden: \begin{mathe}[mc]{c} \begin{matrix}{ccc} \boxed{1} &2 &2\\ 0 &\boxed{4} &5\\ 0 &0 &\boxed{5}\\ \end{matrix}\\ \end{mathe} \end{algorithm} Der Zeilenstufenform entnimmt man, $\rank(A)=3$. Darum sind alle $3$ Vektoren, $\{\mathbf{v}_{1},\mathbf{v}_{2},\mathbf{v}_{3}\}$, linear unabhängig. \end{proof} %% AUFGABE 7-2b \item \begin{claim*} Sei $K=\mathbb{F}_{5}$. Dann sind ${\mathbf{v}_{1}=\begin{svector} 1\\ 2\\ 2\\\end{svector}}$, ${\mathbf{v}_{2}=\begin{svector} 3\\ 2\\ 1\\\end{svector}}$, und ${\mathbf{v}_{3}=\begin{svector} 2\\ 1\\ 4\\\end{svector}}$ über $K$ \fbox{nicht linear unabhängig}. \end{claim*} \begin{proof} Es reicht aus, den (Zeilen)rang der Matrix \begin{mathe}[mc]{rcl} A &:= &\begin{matrix}{ccc} 1 &3 &2\\ 2 &2 &1\\ 2 &1 &4\\ \end{matrix} \end{mathe} zu untersuchen. Um dies zu bestimmen, können wir das Gaußverfahren anwenden. Da wir in $\mathbf{F}_{5}$ arbeiten, genügt es, die Matrix über $\intgr$ zu behandeln, und lediglich in den Zeilenoperationen Vielfache von $5$ zu vermeiden. Da die Matrix dieselbe ist wie in \textbf{Aufgabe 7.2(a)} und in dem Gaußverfahren dort Vielfache von $5$ vermieden wurden, ist das Resultat \begin{mathe}[mc]{c} \begin{matrix}{ccl} \boxed{1} &2 &2\\ 0 &\boxed{4} &5(=0)\\ 0 &0 &5(=0)\\ \end{matrix}\\ \end{mathe} Der Zeilenstufenform entnimmt man, $\rank(A)=2$. Darum sind nur $2$ der Vektoren, und zwar $\{\mathbf{v}_{1},\mathbf{v}_{2}\}$, linear unabhängig. Der Vektor, $\mathbf{v}_{3}$, hingegen, hängt linear von diesen ab. \end{proof} %% AUFGABE 7-2c \item \begin{claim*} Sei $K=\kmplx$. Dann sind ${\mathbf{v}_{1}=\begin{svector} 1\\ \imageinh\\ 0\\\end{svector}}$, ${\mathbf{v}_{2}=\begin{svector} 1+\imageinh\\ -\imageinh\\ 1-2\imageinh\\\end{svector}}$, und ${\mathbf{v}_{3}=\begin{svector} \imageinh\\ 1-\imageinh\\ 2-\imageinh\\\end{svector}}$ über $K$ \fbox{nicht linear unabhängig}. \end{claim*} \begin{proof} Es reicht aus, den (Zeilen)rang der Matrix \begin{mathe}[mc]{rcl} A &:= &\begin{matrix}{ccc} 1 &1+\imageinh &\imageinh\\ \imageinh &-\imageinh &1-\imageinh\\ 0 &1-2\imageinh &2-\imageinh\\ \end{matrix} \end{mathe} zu untersuchen. Wir berechnen \begin{algorithm}[\rtab][\rtab] Reduktion der Matrix, $A$, mittels des Gaußverfahrens:\\ Zeilentransformation ${Z_{2} \leftsquigarrow \imageinh\cdot Z_{2}+Z_{1}}$ anwenden: \begin{mathe}[mc]{c} \begin{matrix}{ccc} 1 &1+\imageinh &\imageinh\\ 0 &2+\imageinh &1+2\imageinh\\ 0 &1-2\imageinh &2-\imageinh\\ \end{matrix}\\ \end{mathe} Zeilentransformation ${Z_{3} \leftsquigarrow \imageinh\cdot Z_{3}-Z_{2}}$ anwenden: \begin{mathe}[mc]{c} \begin{matrix}{ccc} \boxed{1} &1+\imageinh &\imageinh\\ 0 &\boxed{2+\imageinh} &1+2\imageinh\\ 0 &0 &0\\ \end{matrix}\\ \end{mathe} \end{algorithm} Der Zeilenstufenform entnimmt man, $\rank(A)=2$. Darum sind nur $2$ der Vektoren, und zwar $\{\mathbf{v}_{1},\mathbf{v}_{2}\}$, linear unabhängig. Der Vektor, $\mathbf{v}_{3}$, hingegen, hängt linear von diesen ab. \end{proof} Wir betrachten nun dieselbe Aufgabe, nur über $\reell$ statt $\kmplx$: \begin{claim*} Sei $K=\reell$. Dann sind ${\mathbf{v}_{1}=\begin{svector} 1\\ 0\\ 0\\ 1\\ 0\\ 0\\\end{svector}}$, ${\mathbf{v}_{2}=\begin{svector} 1\\ 1\\ 0\\ -1\\ 1\\ -2\\\end{svector}}$, und ${\mathbf{v}_{3}=\begin{svector} 0\\ 1\\ 1\\ -1\\ 2\\ -1\\\end{svector}}$ über $K$ \fbox{linear unabhängig}. \end{claim*} \begin{proof} Es reicht aus, den (Zeilen)rang der Matrix \begin{mathe}[mc]{rcl} A &:= &\begin{smatrix} 1 &1 &0\\ 0 &1 &1\\ 0 &0 &1\\ 1 &-1 &-1\\ 0 &1 &2\\ 0 &-2 &-1\\ \end{smatrix} \end{mathe} zu untersuchen. Wir berechnen \begin{algorithm}[\rtab][\rtab] Reduktion der Matrix, $A$, mittels des Gaußverfahrens:\\ Zeilentransformation ${Z_{4} \leftsquigarrow \imageinh\cdot Z_{1}-Z_{4}}$ anwenden: \begin{mathe}[mc]{c} \begin{smatrix} 1 &1 &0\\ 0 &1 &1\\ 0 &0 &1\\ 0 &2 &1\\ 0 &1 &2\\ 0 &-2 &-1\\ \end{smatrix}\\ \end{mathe} Zeilentransformation ${Z_{3} \leftrightsquigarrow Z_{6}}$ anwenden: \begin{mathe}[mc]{c} \begin{smatrix} 1 &1 &0\\ 0 &1 &1\\ 0 &-2 &-1\\ 0 &2 &1\\ 0 &1 &2\\ 0 &0 &1\\ \end{smatrix}\\ \end{mathe} Zeilentransformationen ${Z_{3} \leftsquigarrow 2\cdot Z_{2}+Z_{3}}$, ${Z_{4} \leftsquigarrow 2\cdot Z_{2}-Z_{4}}$, und ${Z_{5} \leftsquigarrow -1\cdot Z_{2}+Z_{5}}$ anwenden: \begin{mathe}[mc]{c} \begin{smatrix} 1 &1 &0\\ 0 &1 &1\\ 0 &0 &1\\ 0 &0 &1\\ 0 &0 &1\\ 0 &0 &1\\ \end{smatrix}\\ \end{mathe} Zeilentransformationen ${Z_{4} \leftsquigarrow Z_{4}-Z_{3}}$, ${Z_{5} \leftsquigarrow Z_{5}-Z_{3}}$, und ${Z_{6} \leftsquigarrow Z_{6}-Z_{3}}$ anwenden: \begin{mathe}[mc]{c} \begin{smatrix} 1 &1 &0\\ 0 &1 &1\\ 0 &0 &1\\ 0 &0 &0\\ 0 &0 &0\\ 0 &0 &0\\ \end{smatrix}\\ \end{mathe} \end{algorithm} Der Zeilenstufenform entnimmt man, $\rank(A)=3$. Darum sind alle $3$ Vektoren, $\{\mathbf{v}_{1},\mathbf{v}_{2},\mathbf{v}_{3}\}$, linear unabhängig. \end{proof} \end{enumerate} %% AUFGABE 7-3 \clearpage \let\altsectionname\sectionname \def\sectionname{Aufgabe} \section[Aufgabe 3]{} \label{ueb:7:ex:3} \let\sectionname\altsectionname Seien $K$ ein Körper und $V$ ein Vektorraum über $K$. Seien $n\in\ntrlpos$ und $\mathbf{v}_{i}\in V$ für $i\in\{1,2,\ldots,n\}$. \begin{enumerate}{\bfseries (a)} %% AUFGABE 7-3a \item \begin{claim*} Die folgende Aussage ist \fbox{gültig}:\\ Angenommen es existieren linear unabhängige Vektoren, $\mathbf{w}_{1},\mathbf{w}_{2},\ldots,\mathbf{w}_{n}\in V$ und Skalare $c_{i}\in K\ohne\{0\}$, so dass $\mathbf{v}_{i}=c_{i}\mathbf{w}_{i}$ für alle $i\in\{1,2,\ldots,n\}$. Dann bilden $\mathbf{v}_{1},\mathbf{v}_{2},\ldots,\mathbf{v}_{n}$ ein linear unabhängiges System. \end{claim*} \begin{proof} Wir zeigen dies direkt. Sei $\alpha_{i}\in K$ für $i\in\{1,2,\ldots,n\}$ und so dass $\sum_{i=1}^{n}\alpha_{i}\cdot\mathbf{v}_{i}=\zerovector$.\\ \textbf{Zu zeigen:} $\forall{i\in\{1,2,\ldots,n\}:~}\alpha_{i}=0$. Es gilt \begin{longmathe}[mc]{RCL} \sum_{i=1}^{n}\alpha_{i}\cdot\mathbf{v}_{i}=\zerovector &\Longleftrightarrow &\sum_{i=1}^{n}\alpha_{i}\cdot(c_{i}\mathbf{w}_{i})=\zerovector\\ &\Longleftrightarrow &\sum_{i=1}^{n}(\alpha_{i}c_{i})\cdot\mathbf{w}_{i}=\zerovector\\ &\Longrightarrow &\forall{i\in\{1,2,\ldots,n\}:~}\alpha_{i}c_{i}=0,\\ &&\text{da $\mathbf{w}_{1},\mathbf{w}_{2},\ldots,\mathbf{w}_{n}$ linear unabhängig}\\ &\Longleftrightarrow &\forall{i\in\{1,2,\ldots,n\}:~}\alpha_{i}=0\,\text{oder}\,c_{i}=0, \quad\text{da $K$ ein Körper ist}\\ &\Longleftrightarrow &\forall{i\in\{1,2,\ldots,n\}:~}\alpha_{i}=0, \quad\text{da $c_{i}\neq 0$ für alle $i$}.\\ \end{longmathe} Darum gilt $\alpha_{i}=0$ für alle $i\in\{1,2,\ldots,n\}$. Folglich sind $\mathbf{v}_{1},\mathbf{v}_{2},\ldots,\mathbf{v}_{n}$ linear unabhängig. \end{proof} %% AUFGABE 7-3b \item \begin{claim*} Die folgende Aussage ist \fbox{ungültig}:\\ Angenommen, $\mathbf{v}_{n}\notin\vectorspacespan(\mathbf{v}_{1},\mathbf{v}_{2},\ldots,\mathbf{v}_{n-1})$. Dann ist $\mathbf{v}_{1},\mathbf{v}_{2},\ldots,\mathbf{v}_{n}$ linear unabhängig. \end{claim*} \begin{proof} Folgendes ist ein Gegenbeispiel. Sei $n\geq 3$ beliebig. Sei $V=K^{2}$ und betrachte die Vektoren \begin{mathe}[mc]{rclqrcl} \mathbf{v}_{1}=\mathbf{v}_{2}=\ldots=\mathbf{v}_{n-1} &:= &\begin{svector} 1\\ 0\\\end{svector} &\mathbf{v}_{n} &:= &\begin{svector} 0\\ 1\\\end{svector}\\ \end{mathe} Dann gilt $\mathbf{v}_{n}\notin\vectorspacespan(\mathbf{v}_{1},\mathbf{v}_{2},\ldots,\mathbf{v}_{n-1})$, weil $\vectorspacespan(\mathbf{v}_{1},\mathbf{v}_{2},\ldots,\mathbf{v}_{n-1}) =\vectorspacespan(\mathbf{v}_{1}) =\{\begin{svector} t\\ 0\\\end{svector}\mid t\in K\}\notni \begin{svector} 0\\ 1\\\end{svector}$. Andererseits sind die $n-1\geq 2$ Vektoren, $\mathbf{v}_{1},\mathbf{v}_{2},\ldots,\mathbf{v}_{n}$ per Wahl nicht linear unabhängig (weil die alle gleich sind). Also sind die Vektoren, $\mathbf{v}_{1},\mathbf{v}_{2},\ldots,\mathbf{v}_{n}$ ebenfalls nicht linear unabhängig. Darum gilt die behauptete Implikation nicht im Allgemeinen. \end{proof} %% AUFGABE 7-3c \item \begin{claim*} Die folgende Aussage ist \fbox{ungültig}:\\ Das System $\mathbf{v}_{1},\mathbf{v}_{2},\ldots,\mathbf{v}_{n}$ ist genau dann linear unabhängig, wenn jedes echte Teilsystem linear unabhängig ist. \end{claim*} \begin{proof} Der $\Rightarrow$-Teil ist offensichtlich wahr. Es kann also nur die $\Leftarrow$-Richtung schiefgehen. Wir betrachten ein Gegenbeispiel mit $n=3$ Vektoren. Sei $V=K^{2}$ und betrachte \begin{mathe}[mc]{rclqrclqrcl} \mathbf{v}_{1} &:= &\begin{svector} 0\\ 1\\\end{svector}, &\mathbf{v}_{2} &:= &\begin{svector} 1\\ 0\\\end{svector}, &\mathbf{v}_{3} &:= &\begin{svector} 1\\ 1\\\end{svector}.\\ \end{mathe} Es ist einfach zu sehen, dass die echten Teilsysteme \begin{mathe}[mc]{cccccc} (\mathbf{v}_{1}), &(\mathbf{v}_{2}), &(\mathbf{v}_{3}), &(\mathbf{v}_{1},\mathbf{v}_{2}) &(\mathbf{v}_{1},\mathbf{v}_{3}) &(\mathbf{v}_{2},\mathbf{v}_{3})\\ \end{mathe} linear unabhängig sind. Aber (vor allem weil $V$ nur $2$-dimensional ist) $\mathbf{v}_{1},\mathbf{v}_{2},\mathbf{v}_{3}$ ist nicht linear unabhängig, da \begin{mathe}[mc]{rcl} 1\cdot\mathbf{v}_{1} +1\cdot\mathbf{v}_{2} +-1\cdot\mathbf{v}_{3} &= &\zerovector.\\ \end{mathe} Darum gilt die behauptete Implikation nicht im Allgemeinen. \end{proof} %% AUFGABE 7-3d \item \begin{claim*} Die folgende Aussage ist \fbox{gültig}:\\ Angenommen, $\mathbf{v}_{1},\mathbf{v}_{2},\ldots,\mathbf{v}_{n}$ sei linear unabhängig. Dann für alle $i\in\{1,2,\ldots,n\}\ohne\{1\}$ und $c\in K$ bilden $\mathbf{u},\mathbf{v}_{2},\ldots,\mathbf{v}_{n}$ ein linear unabhängiges System, wobei ${\mathbf{u}:=\mathbf{v}_{1}+c\mathbf{v}_{i}}$. \end{claim*} \begin{proof} Wir zeigen dies direkt. Sei $\alpha_{j}\in K$ für $j\in\{1,2,\ldots,n\}$ und so dass $\alpha_{1}\mathbf{u}+\sum_{j=2}^{n}\alpha_{j}\cdot\mathbf{v}_{j}=\zerovector$.\\ \textbf{Zu zeigen:} $\forall{j\in\{1,2,\ldots,n\}:~}\alpha_{j}=0$. Es gilt \begin{longmathe}[mc]{RCL} \alpha_{1}\mathbf{u}+\sum_{j=2}^{n}\alpha_{j}\cdot\mathbf{v}_{j}=\zerovector &\Longleftrightarrow &\alpha_{1}\mathbf{v}_{1} +\alpha_{1}c\mathbf{v}_{i} +\sum_{j=2}^{n}\alpha_{j}\cdot\mathbf{v}_{j}=\zerovector\\ &\Longleftrightarrow &c\alpha_{1}\mathbf{v}_{i} +\sum_{j=1}^{n}\alpha_{j}\cdot\mathbf{v}_{j}=\zerovector\\ &\Longleftrightarrow &\sum_{j=1}^{n}\beta_{j}\mathbf{v}_{j}=\zerovector,\\ &&\text{% wobei $\beta_{j}=\alpha_{j}$ für $j\in\{1,2,\ldots,n\}\ohne\{i\}$ und $\beta_{i}=c\alpha_{1}+\alpha_{i}$ }\\ &\Longrightarrow &\forall{j\in\{1,2,\ldots,n\}:~}\beta_{j}=0,\\ &&\text{da $\mathbf{v}_{1},\mathbf{v}_{2},\ldots,\mathbf{v}_{n}$ linear unabhängig}\\ &\Longleftrightarrow &c\alpha_{1}+\alpha_{i}=0 \,\text{und}\, \forall{j\in\{1,2,\ldots,n\}\ohne\{i\}:~}\alpha_{j}=0\\ &\Longrightarrow &c\cdot 0+\alpha_{i}=0 \,\text{und}\, \forall{j\in\{1,2,\ldots,n\}\ohne\{i\}:~}\alpha_{j}=0\\ &\Longleftrightarrow &\forall{j\in\{1,2,\ldots,n\}:~}\alpha_{j}=0\\ \end{longmathe} Darum gilt $\alpha_{i}=0$ für alle $i\in\{1,2,\ldots,n\}$. Folglich sind $\mathbf{u},\mathbf{v}_{2},\ldots,\mathbf{v}_{n}$ linear unabhängig. \end{proof} \end{enumerate} %% ******************************************************************************** %% FILE: body/uebung/ueb8.tex %% ******************************************************************************** \setcounternach{chapter}{8} \chapter[Woche 8]{Woche 8} \label{ueb:8} %% AUFGABE 8-1 \let\altsectionname\sectionname \def\sectionname{Aufgabe} \section[Aufgabe 1]{} \label{ueb:8:ex:1} \let\sectionname\altsectionname In dieser Aufgabe arbeiten wir im Vektorraum $V=\reell^{4}$. Seien \begin{mathe}[mc]{rcl} U_{1} &= &\{\mathbf{x}\in V\mid x_{1}+2x_{4}=x_{2}+2x_{3}\},\\ U_{2} &= &\{\mathbf{x}\in V\mid x_{1}=x_{2}+x_{3}+x_{4}\}.\ \end{mathe} Zu bestimmen sind Basen für $U_{1}$, $U_{2}$, $U_{1}\cap U_{2}$, und $U_{1}+U_{2}$. Wir beachten vorab \begin{longmathe}[mc]{RCLCL} \mathbf{x}\in U_{1} &\Longleftrightarrow &x_{1}-x_{2}-2x_{3}+2x_{4}=0 &\Longleftrightarrow &\underbrace{ \begin{matrix}{cccc} 1 &-1 &-2 &2\\ \end{matrix} }_{=:A_{1}} \mathbf{x}=\zerovector\\ \mathbf{x}\in U_{2} &\Longleftrightarrow &x_{1}-x_{2}-x_{3}-x_{4}=0 &\Longleftrightarrow &\underbrace{ \begin{matrix}{cccc} 1 &-1 &-1 &-1\\ \end{matrix} }_{=:A_{2}} \mathbf{x}=\zerovector\\ \mathbf{x}\in U_{1}\cap U_{2} &\Longleftrightarrow &\begin{array}[t]{0l} x_{1}-x_{2}-2x_{3}+2x_{4} = 0\\ \text{und}\,x_{1}-x_{2}-x_{3}-x_{4} = 0\\ \end{array} &\Longleftrightarrow &\underbrace{ \begin{matrix}{cccc} 1 &-1 &-2 &2\\ 1 &-1 &-1 &-1\\ \end{matrix} }_{=:A_{3}} \mathbf{x}=\zerovector\\ \end{longmathe} für alle $\mathbf{x}\in V$. Folglich ist $U_{1}$ die Menge der Nullvektoren von $A_{1}$, $U_{2}$ die Menge der Nullvektoren von $A_{2}$, $U_{1}\cap U_{2}$ die Menge der Nullvektoren von $A_{3}$. Um diese zu bestimmen, bringen wir diese Matrizen in Zeilenstufenform, und bestimmen mithilfe von \cite[Satz 5.3.8]{sinn2020} eine Basis des Lösungsraums. \begin{algorithm}[\rtab][\rtab] Zeilenstufenform für $A_{1}$: \begin{mathe}[mc]{rcl} A_{1} &= &\begin{matrix}{cccc} 1 &-1 &-2 &2\\ \end{matrix}\\ \end{mathe} Darum sind $x_{2}$, $x_{3}$, $x_{4}$ frei und $x_{1}$ wird durch diese bestimmt. Der Zeilenstufenform zufolge gilt \begin{mathe}[mc]{rcl} x_{1} &= &x_{2}+2x_{3}-2x_{4}.\\ \end{mathe} Mithilfe von \cite[Satz 5.3.8]{sinn2020} erhalten wir eine Basis des Lösungsraums, indem wir jeweils eine freie Unbekannte auf $1$ und alle anderen auf $0$ setzen: \begin{mathe}[mc]{rcl} x_{2}:=1,\,x_{3}:=0,\,x_{4}:=0 &\Longrightarrow &\mathbf{x}=\begin{svector} 1\\ 1\\ 0\\ 0\\\end{svector},\\ x_{2}:=0,\,x_{3}:=1,\,x_{4}:=0 &\Longrightarrow &\mathbf{x}=\begin{svector} 2\\ 0\\ 1\\ 0\\\end{svector},\\ x_{2}:=0,\,x_{3}:=0,\,x_{4}:=1 &\Longrightarrow &\mathbf{x}=\begin{svector} -2\\ 0\\ 0\\ 1\\\end{svector}.\\ \end{mathe} \end{algorithm} Darum gilt \begin{mathe}[mc]{rclcl} U_{1} &= &\{\mathbf{x}\in V\mid A_{1}\mathbf{x}=\zerovector\} &= &\vectorspacespan\underbrace{ \left\{ \begin{svector} 1\\ 1\\ 0\\ 0\\\end{svector}, \begin{svector} 2\\ 0\\ 1\\ 0\\\end{svector}, \begin{svector} -2\\ 0\\ 0\\ 1\\\end{svector} \right\} }_{=:B_{1}}\\ \end{mathe} und \fbox{$B_{1}$ bildet eine Basis für $U_{1}$}. \begin{algorithm}[\rtab][\rtab] Zeilenstufenform für $A_{2}$: \begin{mathe}[mc]{rcl} A_{2} &= &\begin{matrix}{cccc} 1 &-1 &-2 &2\\ \end{matrix}\\ \end{mathe} Darum sind $x_{2}$, $x_{3}$, $x_{4}$ frei und $x_{1}$ wird durch diese bestimmt. Der Zeilenstufenform zufolge gilt \begin{mathe}[mc]{rcl} x_{1} &= &x_{2}+x_{3}+x_{4}.\\ \end{mathe} Mithilfe von \cite[Satz 5.3.8]{sinn2020} erhalten wir eine Basis des Lösungsraums wie oben: \begin{mathe}[mc]{rcl} x_{2}:=1,\,x_{3}:=0,\,x_{4}:=0 &\Longrightarrow &\mathbf{x}=\begin{svector} 1\\ 1\\ 0\\ 0\\\end{svector},\\ x_{2}:=0,\,x_{3}:=1,\,x_{4}:=0 &\Longrightarrow &\mathbf{x}=\begin{svector} 1\\ 0\\ 1\\ 0\\\end{svector},\\ x_{2}:=0,\,x_{3}:=0,\,x_{4}:=1 &\Longrightarrow &\mathbf{x}=\begin{svector} 1\\ 0\\ 0\\ 1\\\end{svector}.\\ \end{mathe} \end{algorithm} Darum gilt \begin{mathe}[mc]{rclcl} U_{2} &= &\{\mathbf{x}\in V\mid A_{2}\mathbf{x}=\zerovector\} &= &\vectorspacespan\underbrace{ \left\{ \begin{svector} 1\\ 1\\ 0\\ 0\\\end{svector}, \begin{svector} 1\\ 0\\ 1\\ 0\\\end{svector}, \begin{svector} 1\\ 0\\ 0\\ 1\\\end{svector} \right\} }_{=:B_{2}}\\ \end{mathe} und \fbox{$B_{2}$ bildet eine Basis für $U_{2}$}. \begin{algorithm}[\rtab][\rtab] Zeilenstufenform für $A_{3}$ ($\text{Zeile}_{2} \leftsquigarrow \text{Zeile}_{2}-\text{Zeile}_{1}$): \begin{mathe}[mc]{rclcl} A_{3} &= &\begin{matrix}{cccc} 1 &-1 &-2 &2\\ 1 &-1 &-1 &-1\\ \end{matrix} &\rightsquigarrow &\begin{matrix}{cccc} 1 &-1 &-2 &2\\ 0 &0 &1 &-3\\ \end{matrix}\\ \end{mathe} Darum sind $x_{2}$, $x_{4}$, frei und $x_{1}$, $x_{3}$ werden durch diese bestimmt. Der Zeilenstufenform zufolge gilt \begin{mathe}[mc]{rcl} x_{3} &= &3x_{4}\\ x_{1} &= &x_{2}+2x_{3}-2x_{4} = x_{2}+2(3x_{4})-2x_{4} = x_{2}+4x_{4}.\\ \end{mathe} Mithilfe von \cite[Satz 5.3.8]{sinn2020} erhalten wir eine Basis des Lösungsraums wie oben: \begin{mathe}[mc]{rcl} x_{2}:=1,\,x_{4}:=0 &\Longrightarrow &\mathbf{x}=\begin{svector} 1\\ 1\\ 0\\ 0\\\end{svector},\\ x_{2}:=0,\,x_{4}:=1 &\Longrightarrow &\mathbf{x}=\begin{svector} 4\\ 0\\ 3\\ 1\\\end{svector}.\\ \end{mathe} gegeben. \end{algorithm} Darum gilt \begin{mathe}[mc]{rclcl} U_{1}\cap U_{2} &= &\{\mathbf{x}\in V\mid A_{3}\mathbf{x}=\zerovector\} &= &\vectorspacespan\underbrace{ \left\{ \begin{svector} 1\\ 1\\ 0\\ 0\\\end{svector}, \begin{svector} 4\\ 0\\ 3\\ 1\\\end{svector} \right\} }_{=:B_{3}}\\ \end{mathe} und \fbox{$B_{3}$ bildet eine Basis für $U_{1}\cap U_{2}$}. Es bleibt, nur noch eine Basis für $U_{1}+U_{2}$ zu bestimmen. \textbf{ANSATZ I.}\\ Mithilfe der oben berechneten Basen für $U_{1}$, $U_{2}$, wissen wir \begin{mathe}[mc]{rcl} U_{1}+U_{2} &= &\vectorspacespan\big\{ \begin{svector} 1\\ 1\\ 0\\ 0\\\end{svector}, \begin{svector} 2\\ 0\\ 1\\ 0\\\end{svector}, \begin{svector} -2\\ 0\\ 0\\ 1\\\end{svector} \big\} +\vectorspacespan\big\{ \begin{svector} 1\\ 1\\ 0\\ 0\\\end{svector}, \begin{svector} 1\\ 0\\ 1\\ 0\\\end{svector}, \begin{svector} 1\\ 0\\ 0\\ 1\\\end{svector} \big\}\\ &= &\vectorspacespan\big\{ \begin{svector} 1\\ 1\\ 0\\ 0\\\end{svector}, \begin{svector} 2\\ 0\\ 1\\ 0\\\end{svector}, \begin{svector} -2\\ 0\\ 0\\ 1\\\end{svector} \begin{svector} 1\\ 0\\ 1\\ 0\\\end{svector}, \begin{svector} 1\\ 0\\ 0\\ 1\\\end{svector} \big\}.\\ \end{mathe} Wir haben nun ein Erzeugendensystem bestimmt. Diese Menge muss auf eine \emph{maximale linear unabhängige Teilmenge} reduziert werden, um eine Basis daraus zu berechnen. Hierfür reicht es aus, die Vektoren in ein homogenes LGS überzuführen, die Matrix auf Zeilenstufenform zu reduzieren, um etwa durch die Spalten entsprechend den freien Unbekannten zu bestimmen, welche Spalten linear abhängig sind. \begin{algorithm}[\rtab][\rtab] Homogenes System: \begin{mathe}[mc]{rcl} \begin{matrix}{ccccc} 1 &2 &-2 &1 &1\\ 1 &0 &0 &0 &0\\ 0 &1 &0 &1 &0\\ 0 &0 &1 &0 &1\\ \end{matrix}\\ \end{mathe} Zeilenoperation ${Z_{2}\leftsquigarrow Z_{1}-Z_{2}}$ anwenden: \begin{mathe}[mc]{rcl} \begin{matrix}{ccccc} 1 &2 &-2 &1 &1\\ 0 &2 &-2 &1 &1\\ 0 &1 &0 &1 &0\\ 0 &0 &1 &0 &1\\ \end{matrix}\\ \end{mathe} Zeilenoperation ${Z_{3}\leftsquigarrow 2\cdot Z_{3} - Z_{2}}$ anwenden: \begin{mathe}[mc]{rcl} \begin{matrix}{ccccc} 1 &2 &-2 &1 &1\\ 0 &2 &-2 &1 &1\\ 0 &0 &2 &1 &-1\\ 0 &0 &1 &0 &1\\ \end{matrix}\\ \end{mathe} Zeilenoperation ${Z_{4}\leftsquigarrow Z_{3} - 2\cdot Z_{4}}$ anwenden: \begin{mathe}[mc]{rcl} \begin{matrix}{ccccc} 1 &2 &-2 &1 &1\\ 0 &2 &-2 &1 &1\\ 0 &0 &2 &1 &-1\\ 0 &0 &0 &1 &-3\\ \end{matrix}\\ \end{mathe} $\Longrightarrow$ nur $x_{5}$ frei. \end{algorithm} Also hängt die 5. Spalte von Spalten 1--4 ab, welche der Zeilenstufenform zufolge linear unabhängig sind. Folglich ist \begin{mathe}[mc]{c} \left\{ \begin{svector} 1\\ 1\\ 0\\ 0\\\end{svector}, \begin{svector} 2\\ 0\\ 1\\ 0\\\end{svector}, \begin{svector} -2\\ 0\\ 0\\ 1\\\end{svector} \begin{svector} 1\\ 0\\ 1\\ 0\\\end{svector} \right\}\\ \end{mathe} eine \fbox{Basis für $U_{1}+U_{2}$}. \textbf{ANSATZ II.}\\ Mithilfe der Dimensionsformel (siehe \cite[Satz~5.4.3~(2)]{sinn2020}) wissen wir \begin{mathe}[mc]{rcl} \dim(U_{1}+U_{2}) &= &\dim(U_{1}) + \dim(U_{2}) - \dim(U_{1}\cap U_{2})\\ &= &3+3-2 = 4 = \dim(V).\\ \end{mathe} Da $V$ endlich dimensional ist, und $\dim(U_{1}+U_{2})=\dim(V)$ für den linearen Unterraum $U_{1}+U_{2}\subseteq V$ gilt, gilt $U_{1}+U_{2}=V=\reell^{4}$ (Siehe \cite[Satz~5.4.3~(1)]{sinn2020}). Darum können wir bspw. die \uline{kanonische Basis} für $\reell^{4}$ \begin{mathe}[mc]{c} \left\{ \begin{svector} 1\\ 0\\ 0\\ 0\\\end{svector}, \begin{svector} 0\\ 1\\ 0\\ 0\\\end{svector}, \begin{svector} 0\\ 0\\ 1\\ 0\\\end{svector}, \begin{svector} 0\\ 0\\ 0\\ 1\\\end{svector} \right\}\\ \end{mathe} als \fbox{Basis für $U_{1}+U_{2}$} verwenden. \textbf{Bemerkung.} In diesem letzten Teil hatten wir Glück. Wenn sich $\dim(U_{1}+U_{2})0$, dann \begin{mathe}[mc]{rcl} x^{k} &= &x^{k}-c_{k}^{-1}\cdot\zerovector\\ &\eqcrefoverset{eq:1:\beweislabel}{=} &x^{k}-c_{k}^{-1}\sum_{i=0}^{d}c_{i}(x-1)^{i}\\ &= &x^{k}-\sum_{i=0}^{k}c_{k}^{-1}c_{i}(x-1)^{i} \quad\text{wegen Maximalität von $k$}\\ &= &\sum_{i=0}^{k-1} -c_{k}^{-1}c_{i}(x-1)^{i} +(x^{k}-(x-1)^{k}).\\ \end{mathe} Mithilfe der binomischen Formel ist es leicht zu sehen, dass der letzte Term ein Polynom vom Grade $] (SetAB); \draw (Set1) edge [->] (SetAC); \draw (Set1) edge [->] (SetBC); \draw (SetAB) edge [->] (SetA); \draw (SetAB) edge [->] (SetB); \draw (SetAC) edge [->] (SetA); \draw (SetAC) edge [->] (SetC); \draw (SetBC) edge [->] (SetB); \draw (SetBC) edge [->] (SetC); \draw (SetA) edge [->] (Set0); \draw (SetB) edge [->] (Set0); \draw (SetC) edge [->] (Set0); \end{tikzpicture}} \hraum Man sieht, dass dies einen \emph{Verband} und damit insbesondere eine Halbordnung bildet. \end{proof} \begin{proof}[Ansatz III][Ansatz III] Wir gehen die Axiome einer Halbordnung durch: \begin{kompaktenum}[\rtab][\rtab] \item[\uwave{{\itshape Reflexivität:}}] Sei $A\in\Pot(X)$ beliebig. \textbf{Zu zeigen:} $A\leq A$.\\ Offensichtlich gilt $X\ohne A\subseteq X\ohne A$.\\ Per Konstruktion gilt also $A\leq A$. \item[\uwave{{\itshape Antisymmetrie:}}] Seien $ A, A'\in\Pot(X)$ beliebig.\\ \textbf{Zu zeigen:} $A\leq A'$ und $A'\leq A$ $\Rightarrow$ $A=A'$.\\ Es gilt \begin{mathe}[mc]{rclql} A\leq A'\,\text{und}\, A'\leq A &\Longleftrightarrow &X\ohne A\subseteq X\ohne A' \,\text{und}\, X\ohne A'\subseteq X\ohne A &\text{(per Konstruktion)}\\ &\Longrightarrow &X\ohne A=X\ohne A' &\text{(per Definition von Mengengleichheit)}\\ &\Longrightarrow &A=A', &\text{da $A,A'\subseteq X$}.\\ \end{mathe} \item[\uwave{{\itshape Transitivität:}}] Seien $A, A',(a'',b'')\in\Pot(X)$ beliebig.\\ \textbf{Zu zeigen:} $A\leq A'$ und $A'\leq A''$ $\Rightarrow$ $A\leq A''$.\\ Es gilt \begin{mathe}[mc]{rcl} A\leq A'\,\text{und}\, A'\leq A'' &\Longleftrightarrow &X\ohne A\subseteq X\ohne A' \,\text{und}\, X\ohne A'\subseteq X\ohne A'' \,\text{(per Konstruktion)}\\ &\Longrightarrow &X\ohne A\subseteq X\ohne A''\\ &\Longleftrightarrow &A\leq A'' \,\text{(per Konstruktion)}.\\ \end{mathe} \end{kompaktenum} Darum erfüllt $(\Pot(X),\leq)$ die Axiome einer Halbordnung. \end{proof} %% SKA 4-4 \let\altsectionname\sectionname \def\sectionname{SKA} \section[Aufgabe 4]{} \label{ska:4:ex:4} \let\sectionname\altsectionname Betrachten wir die Halbordnung aus \cite[Beispiel 2.4.2(2)]{sinn2020}. Es sei also $C=\{a,b,c\}$ und die durch folgendes \emph{Hasse-Diagramm} dargestellte Ordnungsrelation auf $\Pot(C)$: \hraum {\footnotesize \begin{tikzpicture}[node distance=1cm, thick] \pgfmathsetmacro\habst{1.5} \pgfmathsetmacro\vabst{1} \node (Set1) at (0*\habst,3*\vabst) {$C$}; \node (SetAB) at (-1*\habst,2*\vabst) {$\{a,b\}$}; \node (SetAC) at (0*\habst,2*\vabst) {$\{a,c\}$}; \node (SetBC) at (1*\habst,2*\vabst) {$\{b,c\}$}; \node (SetA) at (-1*\habst,1*\vabst) {$\{a\}$}; \node (SetB) at (0*\habst,1*\vabst) {$\{b\}$}; \node (SetC) at (1*\habst,1*\vabst) {$\{c\}$}; \node (Set0) at (0*\habst,0*\vabst) {$\leer$}; \draw (Set0) edge [->] (SetA); \draw (Set0) edge [->] (SetB); \draw (Set0) edge [->] (SetC); \draw (SetA) edge [->] (SetAB); \draw (SetA) edge [->] (SetAC); \draw (SetB) edge [->] (SetAB); \draw (SetB) edge [->] (SetBC); \draw (SetC) edge [->] (SetAC); \draw (SetC) edge [->] (SetBC); \draw (SetAB) edge [->] (Set1); \draw (SetAC) edge [->] (Set1); \draw (SetBC) edge [->] (Set1); \end{tikzpicture}} \hraum Wenn wir das Element $\leer$ von $\Pot(C)$ entfernen sieht die Struktur folgendermaßen aus \hraum {\footnotesize \begin{tikzpicture}[node distance=1cm, thick] \pgfmathsetmacro\habst{1.5} \pgfmathsetmacro\vabst{1} \node (Set1) at (0*\habst,3*\vabst) {$C$}; \node (SetAB) at (-1*\habst,2*\vabst) {$\{a,b\}$}; \node (SetAC) at (0*\habst,2*\vabst) {$\{a,c\}$}; \node (SetBC) at (1*\habst,2*\vabst) {$\{b,c\}$}; \node (SetA) at (-1*\habst,1*\vabst) {$\{a\}$}; \node (SetB) at (0*\habst,1*\vabst) {$\{b\}$}; \node (SetC) at (1*\habst,1*\vabst) {$\{c\}$}; \draw (SetA) edge [->] (SetAB); \draw (SetA) edge [->] (SetAC); \draw (SetB) edge [->] (SetAB); \draw (SetB) edge [->] (SetBC); \draw (SetC) edge [->] (SetAC); \draw (SetC) edge [->] (SetBC); \draw (SetAB) edge [->] (Set1); \draw (SetAC) edge [->] (Set1); \draw (SetBC) edge [->] (Set1); \end{tikzpicture}} \hraum Offensichtlich hat $(\Pot(C)\ohne\{\leer\},\subseteq)$ kein kleinstes Element. Die Menge der minimalen Elementen ist durch $\{\{a\},\{b\},\{c\}\}$ gegeben. Also gibt es $3$ minimale Elemente. %% SKA 4-5 \let\altsectionname\sectionname \def\sectionname{SKA} \section[Aufgabe 5]{} \label{ska:4:ex:5} \let\sectionname\altsectionname Sei $W$ die Menge aller Wörter und $\Sigma$ die Menge aller Buchstaben. O.\,E. können wir annehmen, dass jedes Wort $w\in W$ der Länge $|w|\geq 2$ ist. (In Sprachen wie Englisch, Russisch, usw. ist dies nicht der Fall, aber wir könnten diese trivialen Wörter einfach ausschließen.) Betrachten wir die Relation $(W,\sim)$ gegeben durch \begin{mathe}[mc]{rcl} \eqtag[eq:1:ska:4:ex:5] w\sim w' &:\Longleftrightarrow &f(w)=f(w), \end{mathe} wobei $f:W\to\Sigma$ die Abbildung mit $f(w)=\text{1. Buchstabe in $w$}$ für alle $w\in W$ ist. Dann per Konstruktion \uline{reduziert} $f$ die Relation $(W,\sim)$ auf $(\Sigma,=)$. Aufgrund dessen und da $(\Sigma,=)$ eine Äquivalenzrelation ist, ist $(W,\sim)$ automatisch eine Äquivalenzrelation auch. Eigentlich spielt est keine Rolle, wie die Funktion, $f$, aussieht. Solange die Reduktion \eqcref{eq:1:ska:4:ex:5} gilt, bleibt $(W,\sim)$ eine Äquivalenzrelation. Dies gilt also insbesondere ebenfalls, wenn $f$ den zweitletzten Buchstaben von Wörtern berechnet. %% SKA 4-6 \let\altsectionname\sectionname \def\sectionname{SKA} \section[Aufgabe 6]{} \label{ska:4:ex:6} \let\sectionname\altsectionname \begin{enumerate}{\bfseries (a)} %% SKA 4-6a \item \begin{mathe}[tc]{rcl} \sum_{i=2}^{6}(-1)^{i}i^{2} &= &(-1)^{2}\cdot 2^{2} +(-1)^{3}\cdot 3^{2} +(-1)^{4}\cdot 4^{2} +(-1)^{5}\cdot 5^{2} +(-1)^{6}\cdot 6^{2}\\ &= &4-9+16-25+36 = 22\\ \end{mathe} %% SKA 4-6b \item \begin{mathe}[tc]{rcl} \prod_{j=1}^{4}(2j-1) &= &(2\cdot 1 - 1) +(2\cdot 2 - 1) +(2\cdot 3 - 1) +(2\cdot 4 - 1)\\ &= &1-3+5-7 = -4\\ \end{mathe} \end{enumerate} %% SKA 4-7 \let\altsectionname\sectionname \def\sectionname{SKA} \section[Aufgabe 7]{} \label{ska:4:ex:7} \let\sectionname\altsectionname \begin{claim} \makelabel{claim:main:ska:4:ex:7} Bezeichne mit $\Phi(n)$ die Aussage \begin{mathe}[mc]{rcl} \eqtag[eq:1:\beweislabel] \sum_{i=1}^{n}(-1)^{i}i^{2} &= &(-1)^{n}\frac{1}{2}n(n+1).\\ \end{mathe} Dann gilt $\forall{n\in\ntrlpos:~}\Phi(n)$. \end{claim} \begin{einzug}[\rtab][\rtab] \begin{proof} Wir zeigen \Cref{\beweislabel} stumpf per Induktion. \begin{kompaktenum}[\rtab][\rtab] \item[\uwave{{\bfseries Induktionsanfang:}}] Sei $n=1$. Dann \begin{mathe}[mc]{rcl} \sum_{i=1}^{n}(-1)^{i}i^{2} &= &(-1)^{1}1^{2} = -1\\ (-1)^{n}\frac{1}{2}n(n+1) &= &(-1)^{1}\frac{1}{2}\cdot 1\cdot (1+1) = -1\\ \end{mathe} Also gilt \eqcref{eq:1:\beweislabel}. Also gilt $\Phi(1)$. \item[\uwave{{\bfseries Induktionsvoraussetzung:}}] Sei $n>1$. Angenommen, $\Phi(n-1)$ gilt. \item[\uwave{{\bfseries Induktionsschritt:}}] \textbf{Zu zeigen:} $\Phi(n)$ gilt, d.\,h. Gleichung \eqcref{eq:1:\beweislabel} gilt.\\ Es gilt \begin{mathe}[mc]{rcl} \sum_{i=1}^{n}(-1)^{i}i^{2} &= &\sum_{i=1}^{n-1}(-1)^{i}i^{2} + (-1)^{n}n^{2}\\ &= &(-1)^{n-1}\frac{1}{2}(n-1)(n-1+1) + (-1)^{n}n^{2}\\ &&\text{wegen der IV}\\ &= &(-1)^{n}\cdot(-\frac{1}{2}n(n-1) + n^{2})\\ &= &(-1)^{n}\cdot(-\frac{1}{2}n^{2} + \frac{1}{2}n + n^{2})\\ &= &(-1)^{n}\cdot(\frac{1}{2}n^{2} + \frac{1}{2}n)\\ &= &(-1)^{n}\frac{1}{2}n(n+1).\\ \end{mathe} Also gilt \eqcref{eq:1:\beweislabel}. Also gilt $\Phi(n)$. \end{kompaktenum} Also gilt $\forall{n\in\ntrlpos:~}\Phi(n)$. \end{proof} \end{einzug} Für die Summe $\sum_{i=3}^{n}(-1)^{i}i^{2}$ ist der Ausdruck lediglich \begin{mathe}[mc]{rcl} \sum_{i=3}^{n}(-1)^{i}i^{2} &= &\sum_{i=1}^{n}(-1)^{i}i^{2}-(-1)^{1}\cdot 1-(-1)^{2}2^{2}\\ &= &(-1)^{n}\frac{1}{2}n(n+1)-3\\ \end{mathe} für alle $n\geq 3$. Sollten wir dies per Induktion beweisen wollen, brauchen wir lediglich im o.\,s. Beweis den \textbf{Induktionsanfang} auf $n=3$ zu ändern. Der Rest bleibt erhalten. \begin{rem} Man merkt, dass Induktion mit Deduzieren (»Ableiten«) nichts zu tun hat. Induktion ist schließlich nur ein Werkzeug, um Behauptungen zu \emph{verifizieren}. Sie verschafft uns aber keine Mittel, um \emph{auf die Behauptungen zu kommen}. In diesem konkreten Falle wurde Vorarbeit geleistet und \emph{direkt} argumentiert, um auf den Ausdruck in \eqcref{eq:1:\beweislabel} zu kommen. Ohne diese Arbeit wären wir auf diesen Ausdruck gar nicht gekommen. In dieser Vorarbeit steckt also die eigentliche mathematische Arbeit und dies bedarf etwas Kreativität, Intuition, usw. Häufig reicht diese Vorarbeit aber nur, um auf eine sinnvolle Behauptung zu kommen, und zum Schluss runden wir dies mit Induktion ab, um formal die behauptete Aussage zu bestätigen. Das ist die eigentliche Rolle von Induktion als Beweismittel. \end{rem} %% SKA 4-8 \let\altsectionname\sectionname \def\sectionname{SKA} \section[Aufgabe 8]{} \label{ska:4:ex:8} \let\sectionname\altsectionname \uwave{{\bfseries Kurzes Argument:}}\\ Wenn jede Farbe jeweils auf maximal $1$ Karte vorkommt, gibt es $\leq 4\cdot 1$ Karten. Aber $5$ Karten werden gewählt. \uwave{{\bfseries Ausführliches Argument:}}\\ Seien ${X:=\{\clubsuit, \diamondsuit, \heartsuit, \spadesuit\}}$ die Menge der Farben und ${Y:=\{1,2,3,4,5\}}$ die Indizes der Karten. Sei ${f:X\to\Pot(Y)}$ die Funktion, die der Wahl entspricht, d.\,h. \begin{mathe}[mc]{rcl} f(x) &= &\{y\in Y\mid\text{Karte $y$ hat Farbe $x$}\}\\ \end{mathe} für alle Farben $x\in X$. Nun, jede Karte, $y\in Y$, hat eine Farbe, sodass $y\in f(x)$ für ein $x\in X$. Also $Y\subseteq\bigcup_{x\in X}f(x)$. Und per Definition $f(x)\subseteq Y$ für alle $x\in X$. Darum $\bigcup_{x\in X}f(x)\subseteq Y$. Also \begin{mathe}[mc]{rcl} Y &= &\bigcup_{x\in X}f(x)\\ \end{mathe} Andererseits sind die Mengen $(f(x))_{x\in X}$ paarweise disjunkt, da jede Karte höchstens eine Farbe hat. Also ist $(f(x))_{x\in X}$ eine \emph{Partition} von $Y$. Darum \begin{mathe}[mc]{ll} &|Y| = |\bigcup_{x\in X}f(x)| = \sum_{x\in X}|f(x)| \leq |X|\cdot\max_{x\in X}|f(x)|\\ \Longrightarrow &\max_{x\in X}|f(x)| \geq |Y|/|X| = 5/4 > 1\\ \Longrightarrow &\exists{x\in X:~}|f(x)|>1\\ \Longrightarrow &\exists{x\in X:~}|f(x)|\geq 2\\ \end{mathe} Nach der Definition von $f$ heißt dies, es gibt eine Farbe, $x\in\{\clubsuit, \diamondsuit, \heartsuit, \spadesuit\}$, so dass $\geq 2$ der gezogenen Karten der Farbe $x$ sind. %% SKA 4-9 \let\altsectionname\sectionname \def\sectionname{SKA} \section[Aufgabe 9]{} \label{ska:4:ex:9} \let\sectionname\altsectionname \uwave{{\bfseries Kurzes Argument:}}\\ Wenn jeder Kalendartag jeweils von maximal $17$ Studierenden gefeiert wird, gibt es $\leq 366\cdot 17=6222$ Studierende. Aber es gibt $\geq 7000$ Studierende. \uwave{{\bfseries Ausführliches Argument:}}\\ Seien ${X=\{\text{1.~Jan},\,\text{2.~Jan},\,\ldots,\,\text{31.~Dez}\}}$ die Menge der Kalendartage und ${Y=\{x\mid x\,\text{ein/e Studierende/r an der Uni Leipzig}\}}$. Sei ${f:X\to\Pot(Y)}$ die Funktion, die der Wahl entspricht, d.\,h. \begin{mathe}[mc]{rcl} f(x) &= &\{y\in Y\mid\text{Studierende/r $y$ hat am Tag $x$ Geburtstag}\}\\ \end{mathe} für alle Kalendartage $x\in X$. Nun, jede/r Studierende/r, $y\in Y$, hat einen Geburtstag, sodass $y\in f(x)$ für ein $x\in X$. Also $Y\subseteq\bigcup_{x\in X}f(x)$. Und per Definition $f(x)\subseteq Y$ für alle $x\in X$. Darum $\bigcup_{x\in X}f(x)\subseteq Y$. Also \begin{mathe}[mc]{rcl} Y &= &\bigcup_{x\in X}f(x)\\ \end{mathe} Andererseits sind die Mengen $(f(x))_{x\in X}$ paarweise disjunkt, da jede/r Studierende/r höchstens einen Geburtstag hat. Also ist $(f(x))_{x\in X}$ eine \emph{Partition} von $Y$. Darum \begin{mathe}[mc]{ll} &|Y| = |\bigcup_{x\in X}f(x)| = \sum_{x\in X}|f(x)| \leq |X|\cdot\max_{x\in X}|f(x)|\\ \Longrightarrow &\max_{x\in X}|f(x)| \geq |Y|/|X| \geq 7000/366 > 19\\ \Longrightarrow &\exists{x\in X:~}|f(x)|>19\\ \Longrightarrow &\exists{x\in X:~}|f(x)|\geq 20\\ \end{mathe} Nach der Definition von $f$ heißt dies, es gibt einen Kalendartag, ${x\in\{\text{1.~Jan},\,\text{2.~Jan},\,\ldots,\,\text{31.~Dez}\}}$, so dass mindestens $20$ Studierende $x$ als Geburtstag feiern. Insbesondere gibt es $18$ Menschen, die den gleichen Geburtstag feiern. %% SKA 4-10 \let\altsectionname\sectionname \def\sectionname{SKA} \section[Aufgabe 10]{} \label{ska:4:ex:10} \let\sectionname\altsectionname \begin{claim} \makelabel{claim:main:ska:4:ex:10} Bezeichne mit $\Phi(n)$ die Aussage \begin{kompaktitem}[\rtab][\rtab] \item Für alle endlichen Mengen, $E_{1},E_{2},\ldots,E_{n}$, gilt $|\prod_{i=1}^{n}E_{i}|=\prod_{i=1}^{n}|E_{i}|$. \end{kompaktitem} Dann gilt $\forall{n\in\ntrlpos:~}\Phi(n)$. \end{claim} \begin{einzug}[\rtab][\rtab] \begin{proof} Wir zeigen dies per Induktion mit den Fällen $n\leq 2$ als Induktionsanfang. \begin{kompaktenum}[\rtab][\rtab] \item[\uwave{{\bfseries Induktionsanfang:}}] Sei $n=1$. Dann für alle Mengen, $E_{1}$ \begin{mathe}[mc]{rcccl} |\prod_{i=1}^{1}E_{i}| &= &|E_{1}| &= &\prod_{i=1}^{1}|E_{i}|\\ \end{mathe} Also gilt $\Phi(1)$. \item[] Sei $n=2$. Laut \Cref{lemm:1:ska:4:ex:10} (siehe unten) gilt für alle endlichen Mengen $E_{1},E_{2}$ \begin{mathe}[mc]{rcccccl} |\prod_{i=1}^{2}E_{i}| &= &|E_{1}\times E_{2}| &= &|E_{1}|\cdot|E_{2}| &= &\prod_{i=1}^{2}|E_{i}|.\\ \end{mathe} Also gilt $\Phi(2)$. \item[\uwave{{\bfseries Induktionsvoraussetzung:}}] Sei $n>2$. Angenommen, $\Phi(n-1)$ gilt. \item[\uwave{{\bfseries Induktionsschritt:}}] Seien $E_{1},E_{2},\ldots,E_{n}$ beliebige endliche Mengen.\\ \textbf{Zu zeigen:} $|\prod_{i=1}^{n}E_{i}|=\prod_{i=1}^{n}|E_{i}|$ gilt.\\ Es gilt \begin{mathe}[mc]{rclql} |\prod_{i=1}^{n}E_{i}| &= &|\prod_{i=1}^{n-1}E_{i}\times E_{n}|\\ &= &|\prod_{i=1}^{n-1}E_{i}|\cdot|E_{n}|, &\text{da $\Phi(2)$ gilt}\\ &= &\prod_{i=1}^{n-1}|E_{i}|\cdot|E_{n}| &\text{wegen der IV}\\ &= &\prod_{i=1}^{n}|E_{i}|.\\ \end{mathe} Also gilt $\Phi(n)$. \end{kompaktenum} Also gilt $\forall{n\in\ntrlpos:~}\Phi(n)$. \end{proof} \end{einzug} Wir müssen noch den Fall für $2$ Mengen beweisen. \begin{lemm} \makelabel{lemm:1:ska:4:ex:10} Seien $X$, $Y$ beliebige \uline{endliche} Mengen. Dann $|X\times Y|=|X|\cdot |Y|$. \end{lemm} \begin{einzug}[\rtab][\rtab] \begin{proof} Wir zeigen dies direkt. Seien $m:=|X|$ und $n:=|Y|$. Wegen Endlichkeit liegen $m,n$ in $\ntrlzero$. Falls $m=0$ oder $n=0$, so gilt $X=\leer$ oder $Y=\leer$ und damit \begin{mathe}[mc]{rcccccccl} |X\times Y| &= &|\leer| &= &0 &= &m\cdot n &= &|X|\cdot|Y|.\\ \end{mathe} Beschränken wir uns also auf den Famm $m,n>0$. Per Definition von Kardinalität (siehe \cite[\S{}3.3, S.54]{sinn2020}) existieren also Bijektionen \begin{mathe}[mc]{rcccl} f &: &\{0,1,\ldots,m-1\} &\to &X,\\ g &: &\{0,1,\ldots,n-1\} &\to &Y.\\ \end{mathe} (Wir fangen aus praktischen Gründen mit $0$ statt $1$ an.)\\ Definiere nun \begin{mathe}[mc]{rcccl} h &: &\{0,1,\ldots,mn-1\} &\mapsto &X\times Y\\ &: &k &\mapsto &(f(\modfn(k,m)), g([k/m])),\\ \end{mathe} wobei ${[\cdot]:\reell\to\intgr}$ die Gaußklammerfunktion, die reelle Zahlen \emph{abrundet}.\\ \textbf{Zu zeigen:} $h$ ist eine wohldefinierte Bijektion. \begin{kompaktenum}[\rtab][\rtab] \item[\uwave{{\bfseries Wohldefiniertheit:}}] Für alle $k\in\{0,1,\ldots,mn-1\}$ gilt $i:=\modfn(k,m)\in\{0,1,\ldots,m-1\}=\domain(f)$ und $j:=[k/m]\in\{0,1,\ldots,n-1\}=\domain(g)$, sodass $f(i)\in X$ und $g(j)\in Y$ und damit $h(k)=(f(i),g(j))\in X\times Y$. \item[\uwave{{\bfseries Injektivität:}}] Seien $k_{1},k_{2}\in\{0,1,\ldots,mn-1\}$ beliebig. \textbf{Zu zeigen:} $h(k_{1})=h(k_{2})\Rightarrow k_{1}=k_{2}$.\\ Nach \cite[Satz 3.4.2]{sinn2020} existieren (eindeutige) Werte ${q_{1},q_{2}\in\intgr}$ und ${r_{1},r_{2}\in\{0,1,\ldots,m-1\}}$, so dass \begin{mathe}[mc]{rcl} \eqtag[eq:1:\beweislabel] k_{1} &= &mq_{1}+r_{1},\\ k_{2} &= &mq_{2}+r_{2}.\\ \end{mathe} Daraus lässt sich ableiten, dass $\modfn(k_{1},m)=r_{1}$, $\modfn(k_{2},m)=r_{2}$, $[k_{1}/m]=q_{1}$, und $[k_{2}/m]=q_{2}$. Darum gilt \begin{mathe}[mc]{rcl} h(k_{1})=h(k_{2}) &\textoverset{Defn}{\Longleftrightarrow} &(f(r_{1}),g(q_{1}))=(f(r_{2}),g(q_{2}))\\ &\Longrightarrow &f(r_{1})=f(r_{2})\,\text{und}\,g(q_{1})=g(q_{2})\\ &\Longrightarrow &r_{1}=r_{2}\,\text{und}\,q_{1}=q_{2}\\ &&\text{da $f,g$ injektiv sind}\\ &\eqcrefoverset{eq:1:\beweislabel}{\Longrightarrow} &k_{1}=mq_{1}+r_{1}=mq_{2}+r_{2}=k_{2}.\\ \end{mathe} \item[\uwave{{\bfseries Surjektivität:}}] Sei $(x,y)\in X\times Y$. \textbf{Zu zeigen:} $(x,y)\in\range(h)$.\\ Wegen der Surjektivität von $f,g$ existieren nun $i\in\{0,1,\ldots,m-1\}$ und $j\in\{0,1,\ldots,n-1\}$, so dass $f(i)=x$ und $g(j)=y$.\\ Setze nun $k:=mj+i$.\\ Dann $\modfn(k,m)=i$ und $[k/m]=j$, sodass $h(k)=(f(i),g(j))=(x,y)$.\\ Also gilt $(x,y)\in\range(h)$. \end{kompaktenum} Darum ist $h$ ist eine wohldefinierte Bijektion, woraus sich per Definition von Kardinalität direkt ergibt, dass $|X\times Y|=mn=|X|\cdot|Y|$. \end{proof} \end{einzug} %% SKA 4-11 \let\altsectionname\sectionname \def\sectionname{SKA} \section[Aufgabe 11]{} \label{ska:4:ex:11} \let\sectionname\altsectionname Um ein Argument zurückzuweisen, reicht es häufig aus, das Argument einfach \emph{ausführlich} aufzuschreiben. Wir nehmen die Ausführung und formalisieren diese: \begin{claim*} Bezeichne mit $G(x)$, dass $x$ ein Goldfisch ist. Für $n\in\ntrlpos$ bezeichne mit $\Phi(n)$ folgende Aussage \begin{kompaktitem}[\rtab][\rtab] \item Für alle $n$-elementigen Mengen, $X$, von Fischen, wenn $\exists{x\in X:~}G(x)$, dann $\forall{x\in X:~}G(x)$. \end{kompaktitem} Dann $\forall{n\in\ntrlpos:~}\Phi(n)$ \end{claim*} \begin{proof}[ungültiges Argument] Dies wird per Induktion argumentiert. \begin{kompaktenum}[\rtab][\rtab] \item[\uwave{{\bfseries Induktionsanfang:}}] Betrachte eine $1$-elementige Menge, $X$, von Fischen.\\ Angenommen, ein $x_{0}\in X$ mit $G(x_{0})$ existiere.\\ Da $X$ nur dieses eine Element enthält, gilt offensichtlich $\forall{x\in X:~}G(x)$. \item[\uwave{{\bfseries Induktionsvoraussetzung:}}] Sei $n\in\ntrlpos$ mit $n\geq 1$. Angenommen, $\Phi(n)$ gilt. \item[\uwave{{\bfseries Induktionsschritt:}}] Sei $X$ eine $n+1$-elementige Menge von Fischen.\\ Angenommen, ein $x_{0}\in X$ mit $G(x_{0})$ existiere. \textbf{Zu zeigen:} Für alle $x\in X$ gilt $G(x)$.\\ Fixiere einen anderen Fisch $x_{1}\in X\ohne\{x_{0}\}$, was möglich ist, weil $|X|=n+1\geq 2$.\\ Setze $X_{0}:=X\ohne\{x_{1}\}$ und $X_{1}:=X\ohne\{x_{0}\}$.\\ Da $x_{1}\neq x_{0}$, sind $X_{0},X_{1}$ verschiedene $n$-elementige Mengen: \hraum {\footnotesize \begin{tikzpicture}[node distance=1cm, thick] \pgfmathsetmacro\habst{1.5} \pgfmathsetmacro\vabst{1.5} \pgfmathsetmacro\rad{1.5} \node (PtBL) at (-1.25*\habst,0*\vabst) {}; \node (PtTL) at (-1.25*\habst,2*\vabst) {}; \node (PtBR) at (+1.25*\habst,0*\vabst) {}; \node (PtTR) at (+1.25*\habst,2*\vabst) {}; \node (X0mid) at (-0.25*\habst,1*\vabst) {}; \node (X1mid) at (+0.25*\habst,1*\vabst) {}; \node[label=above:{$x_{0}$}] (x0) at (-1*\habst,1*\vabst) {$\bullet$}; \node[label=above:{$x_{1}$}] (x1) at (+1*\habst,1*\vabst) {$\bullet$}; \node[above left = 0.7*\rad and 0.7*\rad of X0mid] {$X_{0}$}; \node[above right = 0.7*\rad and 0.7*\rad of X1mid] {$X_{1}$}; \draw [thick, decoration={brace, mirror, raise=1*\vabst}, decorate] node [pos=0.5, anchor=north, yshift=-10pt] {$X$} (PtBL.south) -- (PtBR.south); \draw[pattern=north west lines] (X0mid) circle[radius=1*\rad]; \draw (X1mid) circle[radius=1*\rad]; \end{tikzpicture}} \hraum Fokussieren wir uns zunächst auf $X_{0}$ (die schattierte Teilmenge).\\ Da $X_{0}$ $n$-elementig ist und $x_{0}\in X_{0}$ und $G(x_{0})$, gilt per IV (\textdagger)~$\forall{x\in X_{0}:~}G(x)$. Wähle nun irgendeinen der Fische, $\tilde{x}\in X_{0}$ und setze $X':=X\ohne\{\tilde{x}\}$.\\ O.\,E. können wir $\tilde{x}:=x_{0}$ wählen, sodass $X'=X_{1}$ gilt.\\ Die Teilmenge $X_{1}$ ist nun eine $n$-elementige Menge mit mindestens $n-1$ Goldfischen.\\ \fbox{Also $\exists{x\in X_{1}:~}G(x)$.}\\ Per IV gilt also $\forall{x\in X_{1}:~}G(x)$.\\ Daraus und aus (\textdagger) folgt $\forall{x\in X:~}G(x)$, da ja $X=X_{0}\cup X_{1}$. Darum gilt $\Phi(n+1)$. \end{kompaktenum} Darum gilt $\forall{n\in\ntrlpos:~}\Phi(n)$. \end{proof} Das Problem mit diesem Argument steckt im Induktionsschritt an genau dieser Stelle: \begin{quote} \itshape Also $\exists{x\in X':~}G(x)$. \end{quote} Im ursprünglichen Text ist dies die problematische Stelle: \begin{quote} \itshape Jetzt können wir aber \uline{auch einen der Goldfische rausnehmen} und haben wieder ein Aquarium mit $n$ Fischen und \uline{mindestens einem Goldfisch}. \end{quote} Zurück aber zu unserer Formalisierung:\\ Wir haben etwas ausführlicher gezeigt, dass die Menge $X'$ mindestens $n-1$ Goldfische enthält. Wenn wir $\tilde{x}:=x_{0}$ wählen entspricht dies der Größe des Schnitts $X_{0}\cap X_{1}$. Das \uline{Diagramm} mag andeuten, dass dieser Schnitt nicht leer ist, aber das Diagramm täuscht. Wir brauchen $n\geq 2$, damit der Schnitt nicht leer ist. Aber im Induktionsschritt wird nur $n\geq 1$ vorausgesetzt. Kurzgesagt, das heißt das Induktionsargument ist faul, \fbox{weil der Schritt $1\rightsquigarrow 2$ implizit übersprungen wird}. %% ******************************************************************************** %% FILE: body/ska/ska5.tex %% ******************************************************************************** \setcounternach{chapter}{5} \chapter[Woche 5]{Woche 5} \label{ska:5} %% SKA 5-2 \let\altsectionname\sectionname \def\sectionname{SKA} \setcounternach{section}{2} \section[Aufgabe 2]{} \label{ska:5:ex:2} \let\sectionname\altsectionname Betrachtet sei die Teilbarkeitsrelation $(\intgr,\divides)$. Wir prüfen, welche Axiome erfüllt sind und beurteilen aufgrund dessen, ob es sich um eine Äquivalenzrelation, partielle Ordnung, Abbildung, usw. handelt. \begin{kompaktenum}[\rtab][\rtab] \item[\uwave{{\bfseries Reflexivität:}}] Sei $a\in\intgr$ beliebig. \textbf{Zu prüfen:} $a\divides a$?\\ Es gilt $a=1\cdot a$ und $1\in\intgr$.\\ Darum gilt $a\divides a$.\\ Also ist $(\intgr,\divides)$ \fbox{reflexiv}. \item[\uwave{{\bfseries Symmetrie:}}] Betrachte bspw. $2,10\in\intgr$.\\ Es gilt $2\divides 10$ aber $10\ndivides 2$.\\ Darum ist $(\intgr,\divides)$ \fbox{nicht symmetrisch}. \item[\uwave{{\bfseries Antisymmetrie:}}] Betrachte bspw. $2,-2\in\intgr$.\\ Es gelten $2\divides -2$ und $-2\divides 2$, aber $2\neq -2$.\\ Darum ist $(\intgr,\divides)$ \fbox{nicht antisymmetrisch}. \item[\uwave{{\bfseries Transitivität:}}] Seien, $a,b,c\in\intgr$. \textbf{Zu prüfen:} ($a\divides b$ und $b\divides c$) $\Rightarrow$ $a\divides c$?\\ Es gilt: \begin{mathe}[mc]{rcl} a\divides b\,\text{und}\,b\divides c &\Longleftrightarrow &\exists{k,j\in\intgr:~}c=kb,\,b=ja\\ &\Longrightarrow &\exists{k,j\in\intgr:~}c=(kj)a\\ &\Longrightarrow &\exists{m\in\intgr:~}c=ma\\ &\Longleftrightarrow &a\divides c.\\ \end{mathe} Also ist $(\intgr,\divides)$ \fbox{transitiv}. \item[\uwave{{\bfseries Totalität:}}] Betrachte bspw. $5,7\in\intgr$.\\ Dann $5\ndivides 7$, $7\ndivides 5$, und $5\neq 7$.\\ Darum ist $(\intgr,\divides)$ \fbox{nicht total}. \item[\uwave{{\bfseries Linkstotalität:}}] Sei $a\in\intgr$. \textbf{Zu prüfen:} $\exists{b\in\intgr:~}a\divides b$?\\ Wegen Reflexivität gilt nun $a\divides a$.\\ Also ist $(\intgr,\divides)$ \fbox{linkstotal}. \item[\uwave{{\bfseries Rechtseindeutigkeit:}}] Betrachte bspw. $2,10,100\in\intgr$.\\ Es gilt $2\divides 10$ und $2\divides 100$, aber $10\neq 100$.\\ Darum ist $(\intgr,\divides)$ \fbox{nicht rechtseindeutig}. \end{kompaktenum} Daraus folgt, dass $(\intgr,\divides)$ weder eine Äquivalenzrelation noch eine (lineare) Ordnungsrelation noch eine partielle Ordnungsrelation ist. Und es gibt keine Funktion ${f:\intgr\to\intgr}$, so dass $\graph(f)=\divides$. \textbf{Bemerkung:} Man kann aber zeigen, das die Beschränkungen $(\ntrlzero,\divides)$ und $(\ntrlpos,\divides)$ zusätzlich Antisymmetrie aufweisen, sodass diese partielle Ordnungsrelationen sind. %% SKA 5-3 \let\altsectionname\sectionname \def\sectionname{SKA} \setcounternach{section}{3} \section[Aufgabe 3]{} \label{ska:5:ex:3} \let\sectionname\altsectionname Seien $a,b\in\intgr$ mit $b>0$. Um $a$ durch $b$ (mit Rest) zu teilen, setzt man \begin{mathe}[mc]{rcl} q &:= &[a/b] \in\intgr\\ r &:= &a-b\cdot q\in\intgr.\\ \end{mathe} wobei ${[\cdot]:\reell\to\intgr}$ die Gaußklammerfunktion ist, die reelle Zahlen \emph{abrundet}. Per Definition gilt \begin{mathe}[mc]{rcccl} q &\leq &a/b &< &q+1\\ \end{mathe} also \begin{mathe}[mc]{rcccl} 0 &\leq &a-b\cdot q &< &b\\ \end{mathe} Darum $r\in\{0,1,\ldots,b-1\}$. Um dies aber \emph{per Hand} bzw. im Kopf zu machen, verwendet man iterative Algorithmen, die aus Schritten besteht: $a$ und $b$ durch »einfachere« Zahlen ersetzen; mit einfacheren Zahlen teilen; Nachjustieren. Welche Methode auch immer man anwendet hat dies mit \fbox{dem Existenz-Teil} des Beweises zu tun. %% SKA 5-4 \let\altsectionname\sectionname \def\sectionname{SKA} \setcounternach{section}{4} \section[Aufgabe 4]{} \label{ska:5:ex:4} \let\sectionname\altsectionname \begin{claim} \makelabel{claim:main:ska:5:ex:4} Es gilt \begin{kompaktenum}{\bfseries (i)}[\rtab][\rtab] \item\punktlabel{1} $\ggT(a,b)=\ggT(b,a)$; \item\punktlabel{2} $\ggT(a,b)=\ggT(|a|,|b|)$; \item\punktlabel{3} $\ggT(a,0)=\ggT(0,a)=|a|$, solange $a\neq 0$; \item\punktlabel{4} $\ggT(ca,cb)=|c|\ggT(a,b)$, solange $b,c\neq 0$; \end{kompaktenum} für $a,b,c\in\intgr$. \end{claim} \begin{einzug}[\rtab][\rtab] \begin{proof} \uline{\bfseries \punktcref{1}:} Es gilt $\ggT(a,b) =\max\{d\in\ntrlpos : d\divides a,b\} =\max\{d\in\ntrlpos : d\divides b,a\} =\ggT(b,a)$. \uline{\bfseries \punktcref{2}:} Sei $d,x\in\intgr$. Dann ist es einfach zu sehen, dass $d\divides x\Leftrightarrow d\divides|x|$.\\ Darum gilt $\ggT(a,b) =\max\{d\in\ntrlpos : d\divides a,b\} =\max\{d\in\ntrlpos : d\divides |a|,|b|\} =\ggT(|a|,|b|)$. \uline{\bfseries \punktcref{3}:} Laut \punktcref{1} reicht es aus \textbf{zu zeigen} $\ggT(a,0)=|a|$.\\ Es gilt $\ggT(a,0)=\max D$, wobei $D:=\{d\in\ntrlpos : d\divides a,0\}$.\\ (I) Setze $d_{0}:=|a|>0$. Offensichtlich gilt $d_{0}\divides a,0$.\\ (II) Für alle $d\in\ntrlpos$ gilt $d\divides a$ $\Rightarrow$ $|\frac{a}{d}|\geq 1$ (da $a,d\neq 0$) $\Rightarrow$ $d\leq|a|=d_{0}$.\\ Daraus ergibt sich, \begin{mathe}[mc]{rcccccl} d_{0} &\textoverset{(I)}{\in} &D &\subseteq &\{d\in\ntrlpos : d\divides a\} &\textoverset{(II)}{\subseteq} &\{d\in\ntrlpos : d\leq d_{0}\}\\ \end{mathe} woraus sich ergibt, dass $d_{0}\leq\max D\leq d_{0}$. Also $\ggT(a,0)=\max D=d_{0}=|a|$. \uline{\bfseries \punktcref{4}:} Setze $d_{1}:=\ggT(a,b)$ und $d_{2}:=\ggT(ca,cb)$. \textbf{Zu zeigen:} $d_{2}=|c|d_{1}$.\\ Wir zeigen dies durch zwei Ungleichungen.\\ Es gilt \begin{mathe}[mc]{rcl} d_{1}\divides a,b &\Longleftrightarrow &\exists{k,j\in\intgr:~}a=kd_{1}\,\text{und}\,b=jd_{1}\\ &\Longleftrightarrow &\exists{k,j\in\intgr:~}ca=kcd_{1}\,\text{und}\,cb=jcd_{1}\\ &\Longleftrightarrow &\exists{k,j\in\intgr:~}ca=k|c|d_{1}\,\text{und}\,cb=j|c|d_{1}\\ &&\text{da man z.\,B. $k$ durch $-k$ ersetzen kann}\\ &\Longleftrightarrow &|c|d_{1}\divides ca,cb\\ \end{mathe} Per Maximalität von $d_{2}$ unter den positiven Teilern, folgt \fbox{$|c|d_{1}\leq d_{2}$}.\\ Andererseits existieren nach dem \emph{Lemma von B\'ezout} $u,v\in\intgr$, so dass \begin{mathe}[mc]{rcl} d_{1}=\ggT(a,b) &= &ua+vb\\ \end{mathe} woraus sich ergibt, dass \begin{mathe}[mc]{rcl} \eqtag[eq:1:\beweislabel] \frac{|c|d_{1}}{d_{2}} &= &\underbrace{% \textstyle\pm u\frac{ca}{d_{2}}+\pm v\frac{cb}{d_{2}} }_{=:w}\\ \end{mathe} Da $d_{2}\divides ca,cb$ ist die rechte Seite von \eqcref{eq:1:\beweislabel} in $\intgr$. Und da $|c|,d_{1},d_{2}>0$, ist die linke Seite von \eqcref{eq:1:\beweislabel} strikt positiv, sodass $w\geq 1$ gilt. Darum \fbox{$|c|d_{1}=w\cdot d_{2}\geq 1\cdot d_{2}$}. \end{proof} \end{einzug} \textbf{Bemerkung.} Ohne das \emph{Lemma von B\'ezout} ist ein Beweis vom Letzten Punkt praktisch unmachbar. %% SKA 5-5 \let\altsectionname\sectionname \def\sectionname{SKA} \setcounternach{section}{5} \section[Aufgabe 5]{} \label{ska:5:ex:5} \let\sectionname\altsectionname Seien $a=57$ und $b=21$. Dann gilt $a=qb+r$, wobei $q=2$ und \fbox{$r=15$}. Es gilt $\ggT(a,b)=3$\footnote{% weil $r=3\cdot 5$ und $3,5\in\mathbb{P}$ und nur $3\divides 57$ gilt. } und $\ggT(b,r)=3$\footnote{% weil $r=3\cdot 5$ und $3,5\in\mathbb{P}$ und nur $3\divides 21$ gilt. } Also gilt $\ggT(a,b)=\ggT(b,\modfn(a,b))$, genau wie \cite[Lemma 3.4.5]{sinn2020} allgemein besagt. %% SKA 5-6 \let\altsectionname\sectionname \def\sectionname{SKA} \setcounternach{section}{6} \section[Aufgabe 6]{} \label{ska:5:ex:6} \let\sectionname\altsectionname Für jeden Fall berechnen wir $\ggT(a,b)$ mittels des Euklidischen Algorithmus (siehe \cite[Satz 3.4.7]{sinn2020}). \begin{longtable}[mc]{|cc|c|c|} \hline \hline $a$ &$b$ &Restberechnung (symbolisch) &Restberechnung (Werte)\\ \hline \endhead $1529$ &$170$ &$a = b\cdot q_{1} + r_{1}$ &$1529 = 170\cdot 8 + 169$\\ &&$b = r_{1}\cdot q_{2} + r_{2}$ &$170 = 169\cdot 1 + \boxed{\mathbf{1}}$\\ &&$r_{1} = r_{2}\cdot q_{3} + r_{3}$ &$169 = 1\cdot 169 + 0$\\ \hline $13758$ &$21$ &$a = b\cdot q_{1} + r_{1}$ &$13758 = 21\cdot 655 + \boxed{\mathbf{3}}$\\ &&$b = r_{1}\cdot q_{2} + r_{2}$ &$21 = 3\cdot 7 + 0$\\ \hline $210$ &$45$ &$a = b\cdot q_{1} + r_{1}$ &$210 = 45\cdot 4 + 30$\\ &&$b = r_{1}\cdot q_{2} + r_{2}$ &$45 = 30\cdot 1 + \boxed{\mathbf{15}}$\\ &&$r_{1} = r_{2}\cdot q_{3} + r_{3}$ &$30 = 15\cdot 2 + 0$\\ \hline $1209$ &$102$ &$a = b\cdot q_{1} + r_{1}$ &$1209 = 102\cdot 11 + 87$\\ &&$b = r_{1}\cdot q_{2} + r_{2}$ &$102 = 87\cdot 1 + 15$\\ &&$r_{1} = r_{2}\cdot q_{3} + r_{3}$ &$87 = 15\cdot 5 + 12$\\ &&$r_{2} = r_{3}\cdot q_{4} + r_{4}$ &$15 = 12\cdot 1 + \boxed{\mathbf{3}}$\\ &&$r_{3} = r_{4}\cdot q_{5} + r_{5}$ &$12 = 3\cdot 4 + 0$\\ \hline \hline \end{longtable} %% SKA 5-7 \let\altsectionname\sectionname \def\sectionname{SKA} \setcounternach{section}{7} \section[Aufgabe 7]{} \label{ska:5:ex:7} \let\sectionname\altsectionname Wir verwenden die Berechnungen aus der Tabelle in SKA \ref{ska:5:ex:6}. \begin{longtable}[mc]{|cc|c|c|} \hline \hline $a$ &$b$ &Rest (symbolisch) &Rest (Werte)\\ \hline \endhead $1529$ &$170$ &$r_{1} = a - 8\cdot b$ &$169 = 1\cdot a + -8\cdot b$\\ &&$r_{2} = b - 1\cdot r_{1}$ &$\boxed{1 = \mathbf{-1}\cdot a + \mathbf{9}\cdot b}$\\ \hline $13758$ &$21$ &$r_{1} = a - 655\cdot b$ &$\boxed{3 = \mathbf{1}\cdot a + \mathbf{-655}\cdot b}$\\ \hline $210$ &$45$ &$r_{1} = a - 4\cdot b$ &$30 = 1\cdot a + -4\cdot b$\\ &&$r_{2} = b - 1\cdot r_{1}$ &$\boxed{15 = \mathbf{-1}\cdot a + \mathbf{5}\cdot b}$\\ \hline $1209$ &$102$ &$r_{1} = a - 11\cdot b$ &$87 = 1\cdot a + -11\cdot b$\\ &&$r_{2} = b - 1\cdot r_{1}$ &$15 = -1\cdot a + 12\cdot b$\\ &&$r_{3} = r_{1} - 5\cdot r_{2}$ &$12 = 6\cdot a + -71\cdot b$\\ &&$r_{4} = r_{2} - 1\cdot r_{3}$ &$\boxed{3 = \mathbf{-7}\cdot a + \mathbf{83}\cdot b}$\\ \hline \hline \end{longtable} %% SKA 5-8 \let\altsectionname\sectionname \def\sectionname{SKA} \setcounternach{section}{8} \section[Aufgabe 8]{} \label{ska:5:ex:8} \let\sectionname\altsectionname Das Lemma von B\'ezout wird mittels des \fbox{Euklidischen Algorithmus} bewiesen.\\ Korollar 3.4.10 baut darauf und charakterisiert, wann zwei Zahlen teilerfremd sind.\\ Lemma 3.4.12 baut darauf und zeigt $\forall{i:~}b,a_{i}\,\text{teilerfremd}\Rightarrow b,\prod_{i=1}^{n}a\,\text{teilerfremd}$.\\ Satz 3.4.14 baut darauf und zeigt $p\divides\prod_{i=1}^{n}a_{i}\Rightarrow \exists{i:~}p\divides a_{i}$ für $p$ prim. Dieses letzte Ergebnis wird im Induktionsargument instrumentalisiert, um Primzerlegungen der Länge $k,l$ auf Primfaktorzerlegungen der Länge $k-1,l-1$ zu reduzieren, um das Induktionsargument voranzubringen. \begin{rem} In der Algebra gibt es zwei Begriffe, die bei gewöhnlichen Primzahlen, sich anwenden lassen: \emph{Irriduzibilität} und \emph{prim}. Die Definition in abstrakten Kontexten von \emph{prim} entspricht der Eigenschaft in \cite[Satz 3.4.14]{sinn2020}, während \emph{Irriduzibilität} eher sich auf die Teilbarkeit bezieht. Etwas »Zufälligerweise« handelt es sich bei $\intgr$ um eine Art von Struktur, in der diese zwei Konzepte zusammenfallen. Wie in fast allen technischen Bereichen sollte man auf solche »Zufälligkeiten« achten: Irgendwann befindet man sich in einer Situation, wo man feiner unterscheiden muss und es nicht mehr selbstverständlich ist, zwei Konzepte als identisch zu behandeln. \end{rem} %% SKA 5-10 \let\altsectionname\sectionname \def\sectionname{SKA} \setcounternach{section}{10} \section[Aufgabe 10]{} \label{ska:5:ex:10} \let\sectionname\altsectionname Siehe \cite[Satz 3.5.1]{sinn2020}. Hier eine Alternative:\\ Für $r=0$ setze man $q_{r}:=1$ und $p_{r}:=0$. Und für alle anderen rationalen Zahlen, $r\in\rtnl\ohne\{0\}$, wähle \begin{mathe}[mc]{rcl} q_{r} &:= &\min\overbrace{% \{n\in\ntrlpos\mid q_{r}\cdot r\in\intgr\} }^{D(r)}\in\ntrlpos\\ p_{r} &:= &q_{r}\cdot r\in\intgr.\\ \end{mathe} Da $r$ rational ist, ist $D(r)$ per Definition nicht leer. Darum ist die Wahl von $q_{r}$ und $p_{r}$ wohldefiniert und per Konstruktion gilt $p_{r}/q_{r}=r$. (Für $r=0$ gilt ebenfalls offensichtlich $p_{r}/q_{r}=r$.) Damit haben wir die Existenz einer kanonischen Darstellung begründet. Stimmt dies mit der Konstruktion im \cite[Satz 3.5.1]{sinn2020} überein? Für $r=0$ gilt offensichtlich $\ggT(p_{r},q_{r})=1$. Für $r\in\rtnl\ohne\{0\}$ gilt $d:=\ggT(p_{r},q_{r})=1$, denn sonst wäre $\frac{q_{r}}{d}$ eine positive natürliche Zahl in $D(r)$, da $\frac{q_{r}}{d}\cdot r=\frac{q_{r}\cdot r}{d}=\frac{p_{r}}{d}\in\intgr$, während $\frac{q_{r}}{d}0$. Angenommen, wir haben bereits $((a_{k},b_{k}))_{k=0}^{n-1}$ konstruiert. Aus $(\frac{a_{n}}{b_{n}})^{2}=p$ folgt nun $a_{n}^{2}=pb_{n}^{2}$. Daraus folgt ${p\divides a_{n}\cdot a_{n}}$ und damit gilt (vgl. \cite[Satz 3.4.14]{sinn2020}) ${p\divides a_{n}}$, weil $p$ prim ist. Da $b_{n}^{2}=p\cdot(\frac{a_{n}}{p})^{2}$ und da $\frac{a_{n}}{p}\in\intgr$, erhalten wir ebenfalls ${p\divides b_{n}\cdot b_{n}}$ und wiederum ${p\divides b_{n}}$. Setze also $a_{n+1}:=\frac{a_{n}}{p}$ und $b_{n+1}:=\frac{b_{n}}{p}$. Dann wie oben gezeigt wurde, gilt $a_{n+1}\in\intgr$ und $b_{n+1}\in\ntrlpos$. Offensichtlich gilt $(\frac{a_{n+1}}{b_{n+1}})^{2}=(\frac{a_{n}}{b_{n}})^{2}=p$. Und, da $p>1$, gilt $b_{n+1}] node [pos=0.5, above] {\footnotesize $f$} (SetY); \draw (SetY) edge [->] node [pos=0.5, above] {\footnotesize $g$} (SetZ); \end{tikzpicture} \hraum \begin{enumerate}{\bfseries (a)} %% QUIZ 4-a \item \begin{claim*} $g\circ f$ injektiv $\Rightarrow$ $f$ injektiv. \end{claim*} \begin{proof} Angenommen, $g\circ f$ sei injektiv. \textbf{Zu zeigen:} $f$ ist injektiv\\ \textbf{Zu zeigen:} Für alle $x_{1},x_{2}\in X$ gilt $f(x_{1})=f(x_{2})\Rightarrow x_{1}=x_{2}$.\\ Seien also $x_{1},x_{2}\in X$ beliebig. Es gilt: \begin{mathe}[mc]{rcl} f(x_{1}) = f(x_{2}) &\Longrightarrow &g(f(x_{1})) = g(f(x_{2}))\\ &\Longrightarrow &(g\circ f)(x_{1}) = (g\circ f)(x_{2})\\ &\Longrightarrow &x_{1} = x_{2}, \,\text{da $g\circ f$ injektiv}.\\ \end{mathe} Also ist $f$ injektiv. \end{proof} %% QUIZ 4-b \item \begin{claim*} $f,g$ injektiv $\Rightarrow$ $g\circ f$ injektiv. \end{claim*} \begin{proof} Angenommen, $f,g$ seien injektiv. \textbf{Zu zeigen:} $g\circ f$ ist injektiv\\ \textbf{Zu zeigen:} Für alle $x_{1},x_{2}\in X$ gilt $(g\circ f)(x_{1})=(g\circ f)(x_{2})\Rightarrow x_{1}=x_{2}$.\\ Seien also $x_{1},x_{2}\in X$ beliebig. Es gilt: \begin{mathe}[mc]{rcl} (g\circ f)(x_{1}) = (g\circ f)(x_{2}) &\Longrightarrow &g(f(x_{1})) = g(f(x_{2}))\\ &\Longrightarrow &f(x_{1}) = f(x_{2}), \,\text{da $g$ injektiv}\\ &\Longrightarrow &x_{1} = x_{2}, \,\text{da $f$ injektiv}.\\ \end{mathe} Also ist $g\circ f$ injektiv. \end{proof} \end{enumerate} %% ******************************************************************************** %% FILE: body/quizzes/quiz5.tex %% ******************************************************************************** \setcounternach{chapter}{5} \chapter[Woche 5]{Woche 5} \label{quiz:5} \begin{claim*} Seien $n\in\ntrlpos$ und $p\in\mathbb{P}$ mit $n